Questions from MedBullets Step3

Pataasin ang iyong marka sa homework at exams ngayon gamit ang Quizwiz!

tinea corporis: scaly erythem annular lesion. skin skin contact sports (wrestling), rx is

-azoles and terbinafine

. First-line treatment for new-onset urticaria is with a --

. First-line treatment for new-onset urticaria is with a second-generation antihistamine, such as cetirizine.

rx tropical sprue

...Tetracycline can be used to treat tropical sprue for 3-6 months. Symptoms of tropical sprue include steatorrhea, cramps, weight loss and signs of fat-soluble vitamin deficiencies. Endoscopy will show flattening of villi and inflammation of the lining of the small intestine with presence of lymphocytes in the biopsy of small intestine tissue.

hydrocele resolve in

1 year

2 cause acanthuses nigricans

1. DM, insulin res 2. gastric adenoCA

rx of asthma?

1. sup o2 2. saba, albuterol 3. steroids

blood cx for inf endocarditis needs blood cx -- hrs apart

12 hrs apart

rx for absence sz, 1st and 2nd

1st ethosux 2nd valproate

alzheimer's rx

1st line AC - rivastigmine 2nd line- memantine

At what HCG should you see uterine preg?

2000

alcoholic hallucinosis occurs when?

24 hrs after last drink DT is 48 hrs+

osteomalacia suspected what lab value to check?

25OHVitD

rx tourettes

2nd gen AP (risperidone)

KIDNEY SHIT: 3 weeks after strep, lump bumpy, dec C3

3 weeks after strep, lump bumpy, dec C3 Type 3 IC deposition: PSGN

Low-dose chest CT scans every year to screen for lung cancer in patients aged 55-80 years of age who have at least a -- pack-year smoking history and currently smoke or quit less than 15 years ago

30

Cystic fibrosis (CF) commonly presents with meconium ileus, characterized by bilious vomiting and failure to pass meconium. CF is an autosomal recessive disease and the most common lethal genetic mutation in the United States. Neonates with CF commonly present with failure to thrive, respiratory compromise, and meconium ileus. In meconium ileus, abnormally thick meconium results in bowel obstruction, perforation, or volvulus. CF is diagnosed by sweat chloride test or genetic testing.

3rd metatarsal fx: stress fx This patient presents with delayed passage of meconium, bilious emesis, unsuccessful disimpaction on digital rectal exam, and dilated loops of bowel on abdominal radiograph which suggests a diagnosis of meconium ileus. The best next step in management is enema with water-soluble contrast which may be both diagnostic and therapeutic. Meconium ileus presents with bilious emesis and failure to pass meconium in the first 48 hours of life. Abdominal radiograph typically shows dilated loops of small intestine proximal to the obstruction. Enema with water-soluble contrast (e.g., Gastrografin) is both diagnostic and therapeutic, as the contrast not only visualizes a microcolon but also relieves the obstruction via osmotic pull of water into the lumen of the colon. Surgery may be necessary if contrast enema fails. Figure A demonstrates the findings of meconium ileus on abdominal radiograph, including dilated loops of bowel and a "soap bubble" appearance of meconium mixed with bowel gas in the right lower quadrant. Illustration A demonstrates a microcolon on contrast enema, which is consistent with meconium ileus.

what deg burn is white leathery dry

3rd, excision and graft

how long does infantile colic take to resolve?

4 mi

if first deg relative has colon ca, do scope at ---

40, or 10 yrs before they got, whichever comes first.

how long do folate stores last

5 mo

2 yr old should have --- word vocab

50

predict post op FEV1 after pneumonectomy, have predicted fev1 of at least

800 ml

Nonreactive stress test defined as < __ mvmts in 2 hrs. If pt not at term, do

< 10 mvmts, do BPP. fetal breathing, body mvmts, fetal tone, AFV, and NST. BPP: BB TAN The NST is indicated in any situation where fetal demise or hypoxemia is suspected. Gestational diabetes is associated with an increased risk of fetal demise, and given that this patient complains of decreased fetal movement and has undetectable fetal heart tones, it is important to evaluate fetal well-being. The NST involves at least 20 minutes of continuous external fetal monitoring. A "reactive" NST indicates stable fetal status at the time of the test but is poorly predictive of future adverse outcomes. Generally, fetuses younger than 28 weeks do not exhibit significant heart rate accelerations, and moderate variability is considered reassuring. Between 28 and 32 weeks gestation, at least two accelerations of at least ten beats per minute lasting for at least ten seconds each should be seen. After 32 weeks, a reactive NST requires two 15-beat-per-minute accelerations lasting for at least 15 seconds (Illustration A). A nonreactive NST should prompt vibroacoustic stimulation in case non-reactivity is due to the fetal sleep cycle, or further testing such as a biophysical profile (BPP).

ARDS PCWP <

< 18 >18= FLUID OVERLOAD STATE Mortelliti and Manning review ARDS. Historically, the three criteria for ARDS were PWCP < 18 mmHg, bilateral infiltrates on chest radiography, and PaO2/FiO2 ratio < 200. Recently, the ARDS Definition Task Force validated a new definition of ARDS, which no longer require PCWP < 18 mmHg, but instead requires the following four features: acute onset (< 1 week), bilateral opacities, respiratory failure not fully explained by volume overload, and PaO2/FiO2 < 300 with minimum PEEP/CPAP of 5 cm H2O. Severity of ARDS, which was predictive of outcome, can now be graded on PaO2/FiO2 (mild < 300, moderate < 200 and severe < 100), eliminating the clinical term "acute lung injury" used previous for PaO2/FiO2 between 200 and 300.

< 30 breast mass do what, what about > 30?

<30 do breast US, likely Fibroadenoma >30 mammography

A -- -- -- randomizes participants into more than 1 independent variable, each with more than 1 possible intervention.

A factorial design study randomizes participants into more than 1 independent variable, each with more than 1 possible intervention.

A patient with a penetrating injury to his extremity in the setting of pulsatile bleeding has likely suffered an arterial injury and should be evaluated next with --

A patient with a penetrating injury to his extremity in the setting of pulsatile bleeding has likely suffered an arterial injury and should be evaluated next with coagulation studies and blood typing/crossmatch due to the possibility of surgical treatment

A patient with symptoms of fever, mental status change, muscle rigidity, and hypertension, with an elevated CK in the setting of antipsychotic medications, is likely suffering from ---. He should be treated with --- along with cessation of the current antipsychotic. Neuroleptic malignant syndrome is a rare but potentially fatal side effect of antipsychotic medications. It presents with symptoms such as fever, mental status change, muscle rigidity, autonomic instability, dystonia, and tremor. Labs typically show an elevated CK level along with hyperkalemia and leukocytosis. It is potentially fatal and should be managed aggressively.

A patient with symptoms of fever, mental status change, muscle rigidity, and hypertension, with an elevated CK in the setting of antipsychotic medications, is likely suffering from neuroleptic malignant syndrome (NMS). He should be treated with dantrolene along with cessation of the current antipsychotic. Neuroleptic malignant syndrome is a rare but potentially fatal side effect of antipsychotic medications. It presents with symptoms such as fever, mental status change, muscle rigidity, autonomic instability, dystonia, and tremor. Labs typically show an elevated CK level along with hyperkalemia and leukocytosis. It is potentially fatal and should be managed aggressively. Muench et al. discuss the adverse effects of antipsychotic medications. They state that NMS typically occurs more often after high-potency first generation antipsychotics compared to lower potency second generation agents. They recommend immediate treatment of NMS with a muscle relaxant such as dantrolene with or without a dopamine agonist such as bromocriptine along with the discontinuation of the offending agent.

A -- cyst presents with pain in the knee that is typically localized to the posterior aspect. A palpable mass is present in the medial and posterior aspect of the knee which represents herniated synovial fluid. This condition is associated with anything that damages the joint. This can include osteoarthrtis, RA, or trauma (in particular meniscal tears). Rheumatoid arthritis is considered a -- hypersensitivity reaction.

A popliteal cyst presents with pain in the knee that is typically localized to the posterior aspect. A palpable mass is present in the medial and posterior aspect of the knee which represents herniated synovial fluid. This condition is associated with anything that damages the joint. This can include osteoarthrtis, rheumatoid arthritis, or trauma (in particular meniscal tears). Rheumatoid arthritis is considered a type IV hypersensitivity reaction.

A -- -- is a complication that can occur after a cardiac catheterization and is diagnosed with a CT scan.

A retroperitoneal hematoma is a complication that can occur after a cardiac catheterization and is diagnosed with a CT scan.

A solid mass confirmed by ultrasound imaging in a patient < 30 years should have the histology evaluated by biopsy, in this case-- -- ---is the most accurate choice.

A solid mass confirmed by ultrasound imaging in a patient < 30 years should have the histology evaluated by biopsy, in this case image-guided core biopsy is the most accurate choice.

cohort study

A type of epidemiologic study where a group of exposed individuals (individuals who have been exposed to the potential risk factor) and a group of non-exposed individuals are followed over time to determine the incidence of disease

A young child presenting with a barking cough and inspiratory stridor is most likely suffering from laryngotracheobronchitis, more commonly called croup. The most common organism is Parainfluenza virus and it should be treated with -- and --

A young child presenting with a barking cough and inspiratory stridor is most likely suffering from laryngotracheobronchitis, more commonly called croup. The most common organism is Parainfluenza virus and it should be treated with corticosteroids and racemic epinehprine.

Postoperative hypocalcemia is common following successful parathyroidectomy. If hypocalcemia is asymptomatic, no treatment is necessary. Symptoms of hypocalcemia generally appear 2 to 4 days following surgery and are initially treated with IV calcium gluconate, or, if symptoms are mild, oral calcium such as calcium lactate, calcium carbonate, or calcium gluconate. For symptoms that occur earlier than two days or are asymmetric, one should be suspicious of other causes.

AAA repair can obstruct blood flow to the colon causing ischemic colitis/mesenteric ischemia.

elevated ferritin =

ACD, give pRBC or sideroblastic anemia, rx Pyridoxine b6

methylphenidate rx

ADHD and narcolepsy

early diastolic blowing, decrescendo murmur heard best at the left sternal borde

AR

paradoxically split s2

AS

aortic stenosis and -- together have poor prognosis

AS and CHF

this OD causes tinnitus

ASA- mixed res alk and met acid

Fixed split S2

ASD

all women aged starting --- should have mammograms every 1-2 years. It is controversial whether screening mammography should begin as early as age 40. Women with first-degree relatives with breast cancer should begin screening 10 years before the age of the family member's diagnosis.

According to the United States Preventive Services Task Force, all women aged 50-74 should have mammograms every 1-2 years. It is controversial whether screening mammography should begin as early as age 40. Women with first-degree relatives with breast cancer should begin screening 10 years before the age of the family member's diagnosis.

Achalasia is worked up with a barium swallow followed by -- to rule out malignancy prior to performing a myotomy.

Achalasia is worked up with a barium swallow followed by endoscopy to rule out malignancy prior to performing a myotomy.

--- ---- deficiency presents with poor muscle tone, feeding difficulties, macroglossia, and hypertrophic cardiomyopathy.

Acid maltase deficiency presents with poor muscle tone, feeding difficulties, macroglossia, and hypertrophic cardiomyopathy. pompe ds- HOCM, floppy baby

-- --- is a premalignant lesion that can present in sun-exposed areas with thickened transparent or yellow scales.

Actinic keratosis is a premalignant lesion that can present in sun-exposed areas with thickened transparent or yellow scales.

Subepithelial deposits with spike and dome appearance on EM is seen in what disease? What is the cause?

Acute PSGN

Acute uncomplicated diverticulitis merits conservative outpatient management with bowel rest and empiric antibiotics against gram-negatives and anaerobes.

Acute uncomplicated diverticulitis merits conservative outpatient management with bowel rest and empiric antibiotics against gram-negatives and anaerobes.

All breastfed infants should receive supplementation with --, and premature breastfed infants should also receive supplementation with -- due to the increased risk of iron-deficiency anemia.

All breastfed infants should receive supplementation with vitamin D, and premature breastfed infants should also receive supplementation with iron due to the increased risk of iron-deficiency anemia.

All pregnant women should receive the --vaccine. Live vaccinations such as varicella, intranasal flu vaccine, measles/mumps/rubella, smallpox, and human papillomavirus are contraindicated.

All pregnant women should receive the intramuscular flu vaccine. Live vaccinations such as varicella, intranasal flu vaccine, measles/mumps/rubella, smallpox, and human papillomavirus are contraindicated.

All premature infants are at risk of -- --. As the condition is often asymptomatic, screening for IVH via cranial (transfontanel) ultrasound should occur by day 7 of life.

All premature infants are at risk of intraventricular hemorrhage (IVH). As the condition is often asymptomatic, screening for IVH via cranial (transfontanel) ultrasound should occur by day 7 of life.

An elevated alpha-fetoprotein (AFP) followed by an US that is unrevealing for the cause of AFP elevation (accurate gestational age; single, viable intrauterine pregnancy; no obvious fetal deformities) is an indication for --- during the second trimester.

An elevated alpha-fetoprotein (AFP) followed by an US that is unrevealing for the cause of AFP elevation (accurate gestational age; single, viable intrauterine pregnancy; no obvious fetal deformities) is an indication for amniocentesis during the second trimester.

An increase in the cut-off value of a test leads to an increase in the number of false negatives. In other words, the higher the threshold for diagnosis, the more individuals who actually have diabetes mellitus there will be who falsely test negative for the disease.

An increase in the cut-off value of a test leads to an increase in the number of false negatives. In other words, the higher the threshold for diagnosis, the more individuals who actually have diabetes mellitus there will be who falsely test negative for the disease.

An otherwise healthy child presenting with a large pink, scaly "herald patch" and many surrounding pink papules over the abdomen and back likely has --- ---. The exanthem is self-limited and will typically resolve without intervention within 3-4 weeks.

An otherwise healthy child presenting with a large pink, scaly "herald patch" and many surrounding pink papules over the abdomen and back likely has pityriasis rosea. The exanthem is self-limited and will typically resolve without intervention within 3-4 weeks. Pityriasis rosea is a common, acute exanthem, commonly affecting children and young adults. While the exact etiology of the exanthem is unknown, HHV6 and HHV7 are considered possible viral candidates. It is characterized by an initial herald patch, followed by the development of a diffuse papulosquamous rash. The secondary exanthem is often described as appearing in a "Christmas tree" pattern over the back, as see in Illustration A. There is no confirmatory diagnostic test. While self-resolving, topical or systemic steroids and antihistamines often are used to relieve pruritis. Stulberg et al provide a general overview of pityriasis rosea. The differential for patients who present with a large, annular lesion and surrounding papulosquamous exanthem includes nummular eczema, secondary syphilis, tinea corporis, pityriasis lichenoides, guttate psoriasis, viral exanthem, lichen planus, and medication reaction. The rash of pityriasis rosea typically lasts about five weeks and resolves by eight weeks in more than 80 percent of patients.

Androstenedione, DHEA, and testosterone would be elevated in a hormonally active -- tumor. no change in estrogen and altos

Androstenedione, DHEA, and testosterone would be elevated in a hormonally active adrenal tumor.

Painless BRN per rectum no diverticulosis or itis

Angiodysplasia, do cautery AVM

Ankylosing spondylitis can cause inflammation and -- that predispose the patient to -- --

Ankylosing spondylitis can cause inflammation and osteoporosis/osteopenia that predispose the patient to vertebral fractures.

most spec AB for RA?

Anti CCP

-- -- antibodies is associated with pemphigus vulgaris. These patients will present with fragile/flaccid blisters and involvement of the oral cavity is more common. On biopsy, intraepidermal cleavage is seen. Immunoflourescence will show IgG in a fish-net pattern.

Anti-desmosome antibodies is associated with pemphigus vulgaris. These patients will present with fragile/flaccid blisters and involvement of the oral cavity is more common. On biopsy, intraepidermal cleavage is seen. Immunoflourescence will show IgG in a fish-net pattern.

Amenorrhea and milky discharge from the breast can be caused by hypothyroidism-induced prolactin release, and an initial TSH level should be obtained.

Aortic regurgitation is associated with a wide pulse pressure, fingernail pulsations, head bobbing, and a femoral artery murmur.

Arrest of labor is defined as no cervical change in - or more hours with adequate contractions or --or more hours with inadequate contractions. adequate ctx are 2-3 min

Arrest of labor is defined as no cervical change in 4 or more hours with adequate contractions or 6 or more hours with inadequate contractions.

-- can cause laboratory abnormalities such as thrombocytosis and Howell-Jolly bodies (on peripheral smear) due to decreased splenic clearance.

Asplenia can cause laboratory abnormalities such as thrombocytosis and Howell-Jolly bodies (on peripheral smear) due to decreased splenic clearance.

-- -- -- presents with gradual, painless loss of central vision with yellow drusen deposits seen on fundoscopic exam.

Atrophic ("dry") macular degeneration presents with gradual, painless loss of central vision with yellow drusen deposits seen on fundoscopic exam. Exudative macular degeneration presents with rapid, painless loss of central vision. Fundoscopic exam shows proliferation of blood vessels and hemorrhage.

fragile x assoc w

Autism

Waardenburg Syndrome

Autosomal dominant, or assoc w/ advanced paternal age. Short palpebral fissures, white forelock and SN deafness. Waardenburg's syndrome is classically associated with hypopigmentation of hair and skin, with the characteristic white forelock being the most distinctive feature. Additionally, congenital unilateral or bilateral sensorineural hearing loss is associated with Waardenburg's syndrome.

rx sx hyperthyroid

BB

rx HOCM

BB or CCB

which meds exam psoriasis

BB, NSAIDS, ACEI, Li, Malaria drugs

best way to dx MM

BM Bx

se mycophenolate

BM Suppression

Bacterial vaginosis should be treated with metronidazole, and patients prescribed oral medication should -- --- so as not to have a disulfiram-like reaction.

Bacterial vaginosis should be treated with metronidazole, and patients prescribed oral medication should avoid alcohol so as not to have a disulfiram-like reaction. flagyl and EtOH can cause disulfiram rxn

Beta-thalassemia major presents with severe --- ,-- anemia.

Beta-thalassemia major presents with severe microcytic, hypochromic anemia. inc hb f and a2

Bidirectional ventricular tachycardia is a sign of severe -- toxicity, which should be managed with -- antibodies.

Bidirectional ventricular tachycardia is a sign of severe digoxin toxicity, which should be managed with anti-digoxin antibodies.

-- is a broad-based budding yeast that can disseminate in immunocompromised hosts causing skin lesions, osteolytic bone lesions, and lung nodules.

Blastomyces is a broad-based budding yeast that can disseminate in immunocompromised hosts causing skin lesions, osteolytic bone lesions, and lung nodules Answer 2: Coccidiomyocosis is caused by the fungus Coccidioides immitis. It is endemic in the Southwest, and patients present with mild pneumonia, meningitis, or disseminated disease to the skin and bone. Biopsy would show spherules containing endospores. Answer 3: Histoplasmosis is caused by the fungus Histoplasma capsulatum. It is endemic in the Mississippi and Ohio River valleys, and is found in bird and bat droppings. Patients present with pneumonia, hilar lymphadenopathy, and ulcerated tongue lesions. On histopathology, oval yeast cells would be seen within macrophages.

Bloody urethral discharge occurring in an older man is -- carcinoma (UC) until proven otherwise.

Bloody urethral discharge occurring in an older man is urethral carcinoma (UC) until proven otherwise.

Bone marrow transplant recipients are immunocompromised and at elevated risk for infection. When a bone marrow transplant patient develops a respiratory tract infection, -- -- should be suspected.

Bone marrow transplant recipients are immunocompromised and at elevated risk for infection. When a bone marrow transplant patient develops a respiratory tract infection, invasive aspergillosis should be suspected.

breast feed jaundice breast milk jaundice explain each

Breast milk jaundice occurs because the infant liver is not mature enough to process lipids. It presents between the 4th and 7th day of life. If breast feeding stops, bilirubin levels fall rapidly; however, if breast feeding continues, bilirubin levels will fall slowly. Distinct from breast milk jaundice, breastfeeding jaundice is the most common cause of neonatal jaundice in the first week of life and occurs in 10% of births. It is characterized by insufficient feeding, which leads to fewer bowel movements and, therefore, decreased bilirubin clearance. The infant in this vignette has been breastfeeding normally.

Broad spectrum antibiotics can destroy the normal flora leading to decreased endogenous vitamin K production and result in ---.

Broad spectrum antibiotics can destroy the normal flora leading to decreased endogenous vitamin K production and bleeding.

biggest risk for pp endometritis?

C sectionThis elderly man with well-controlled diabetes presents with a worsened visual acuity on exam. In the setting of a normal fundoscopic exam, he likely has presbyopia.

elderly immunosuppressed man, possible meningitis, do what

CAVS ceftriax amp vanc steroids

specific phobia rx

CBT

TPN causes less secretion of ---

CCK CCK usually stimulates the gallbladder to contract in response to fat, protein, and carbohydrates within the digestive tract. Gallbladder contraction causes delivery of bile into the small intestine to digest the food that is passing through. With TPN, there is no enteral stimulus to stimulate release of CCK, which leads to gallbladder dysmotility. As a result, particulate sediment from bile (biliary sludge) collects in the gallbladder and can cause biliary colic.

hypothyroid myopathy has elevated

CK level

smudge cells = dx w

CLL dx w For cytometry

fatigue, early satiety, low LAP = rx?

CML rx imatinib

HA, dizzy, polycythemia, dx?

CO poisoning. Pt will be using heater indoors.

resp acidosis causes

COPD exam, Asthma

DLCO is decreased in

COPD, D in COPD DLCO DLCO NORMAL IN ASTHMA

Cell-free DNA testing is a non-invasive and highly sensitive/specific diagnostic test for fetal aneuploidy that can be completed at 10 weeks gestation.

CSF findings in aseptic meningitis show an elevated cell count (though typically < 300/mm^3 in contrast to bacterial meningitis), a normal or elevated protein, and a normal glucose. CSF should be sent for Gram stain and culture to confirm the diagnosis and rule out bacterial meningitis. Once the diagnosis has been confirmed, supportive therapy is all that is needed for viral meningitis.

in suspected child abuse do what first

CT head

so dissection needs

CTA

HA, diplopia, fever, perturb edema, from skin infxn on nose

CV Thrombosis, can also cause loss sensation in V distributions

Calcium-containing kidney stones are radiopaque and can be prevented with increased fluid consumption, -- administration, and low salt and protein intake.

Calcium-containing kidney stones are radiopaque and can be prevented with increased fluid consumption, thiazide diuretic administration, and low salt and protein intake. thiazides inc ca reabs

sinusitis,OCP, smoking, hypercoag state, severe pain sudden HA, diplopia, periorbital edema, dx rx?

Can sinus thrombosis, rx w broad spec AB and surgical drainage.

tia patient, do what

Carotid US

fever, jaundice, RUQ pain

Charcot's triad ascending cholangitis, do ERCP

Insecticide poisoning:

Cholinergic, secretions everywhere! In AC poisoning its opp, dry flushed skin, confusion, urinary retention.

---- ---- is a syndrome consisting of lower urinary tract symptoms and pelvic pain. It is a distinct entity from chronic bacterial prostatitis. Prominent symptoms include pain that can involve the lower abdomen, penis, testicles, or prostate area. There is often concurrent bladder irritation or obstruction that leads to voiding difficulty. Urinalysis can reveal pyuria, but not always. It can be associated with irritable bowel syndrome and fibromyalgia. Front-line treatment for chronic prostatitis includes alpha blockers and fluoroquinolone antibiotics.

Chronic prostatitis is a syndrome consisting of lower urinary tract symptoms and pelvic pain. It is a distinct entity from chronic bacterial prostatitis. Prominent symptoms include pain that can involve the lower abdomen, penis, testicles, or prostate area. There is often concurrent bladder irritation or obstruction that leads to voiding difficulty. Urinalysis can reveal pyuria, but not always. It can be associated with irritable bowel syndrome and fibromyalgia. Front-line treatment for chronic prostatitis includes alpha blockers and fluoroquinolone antibiotics.

ca levels when CYP inducers?

Classic P450 inducers include barbiturates, St. John's Wort, carbamazepine, rifampin, alcohol (chronic), phenytoin, griseofulvin, phenobarbital, and sulfonylureas. When the P450 system is induced, this can lead to increased breakdown of vitamin D and hypocalcemia. Hypocalcemia can present with carpopedal spasm when a patient's blood pressure is taken and spasm of the patient's facial muscle when the masseter is tapped. breaks down Vitae's D and causes dec Ca

Turners murmur

Coarctation aorta: continuous flow murmur interscap region

--- acts to decrease neutrophil motility by inhibiting microtubule polymerization through tubulin binding;

Colchicine acts to decrease neutrophil motility by inhibiting microtubule polymerization through tubulin binding;

Early decels mean:

Compression of the fetal head resulting from uterine contraction . vasovagal response to altered cerebral blood flow.

Consumption of an acid/alkali should be managed with initial stabilization followed by -- --- --

Consumption of an acid/alkali should be managed with initial stabilization followed by imaging with a radiograph, a gastrografin swallow, and endoscopy.

how to dx addisons?

Cosyntropin acth stim test

--- is a remnant of -- pouch that can present with signs of increased intracranial pressure (vomiting and headache) and compress the optic chasm causing bitemporal hemianopsia.

Craniopharyngioma is a remnant of Rathke's pouch that can present with signs of increased intracranial pressure (vomiting and headache) and compress the optic chasm causing bitemporal hemianopsia.

Craniopharyngiomas occur in the remnants of -- --, which is along the nasopharynx to the diencephalon. This tumor can compress the optic chiasm, resulting in a bitemporal hemianopsia.

Craniopharyngiomas occur in the remnants of Rathke pouch, which is along the nasopharynx to the diencephalon. This tumor can compress the optic chiasm, resulting in a bitemporal hemianopsia.

-- -- --- can result in ulnar-distributed symptoms starting at the elbow including loss of sensation over the ulnar side of the forearm, loss of sensation over the medial two digits (pointer and ring finger), and decreased grip strength.

Cubital tunnel compression can result in ulnar-distributed symptoms starting at the elbow including loss of sensation over the ulnar side of the forearm, loss of sensation over the medial two digits (pointer and ring finger), and decreased grip strength.

Cyclic vomiting syndrome (CVS) is an idiopathic disorder that typically presents in children 4-7 years old, with the classic history described in the question stem. It is associated with a family (or personal) history of --. Supportive therapy may be required during acute episodes, and some evidence suggests the use of antimigraine medications may be beneficial as prophylaxis. Most children outgrow the disorder by their teenage years.

Cyclic vomiting syndrome (CVS) is an idiopathic disorder that typically presents in children 4-7 years old, with the classic history described in the question stem. It is associated with a family (or personal) history of migraines. Supportive therapy may be required during acute episodes, and some evidence suggests the use of antimigraine medications may be beneficial as prophylaxis. Most children outgrow the disorder by their teenage years.

meconium ileum assoc w

Cystic Fibrosis, where they have no vas def

t shaped uterus from:

DES in utero, clear cell adenoCA

TBI has what effect on kidneys?

DI, urine will have low spec grab, and low osmolality, but there is high plasma osm

noncompressible veins of lower extrem is what

DVT. new onset means start heparin

Definitive diagnosis involves pulmonary angiography or spiral CT, but these are potentially time-consuming and may result in significant delay in treatment. Thus, if suspicion for PE is high and there are no contraindications, --- should be initiated while workup is completed.

Definitive diagnosis involves pulmonary angiography or spiral CT, but these are potentially time-consuming and may result in significant delay in treatment. Thus, if suspicion for PE is high and there are no contraindications, anticoagulation should be initiated while workup is completed.

Delusional disorders can be treated with --- --.

Delusional disorders can be treated with atypical antipsychotics.

Diarrhea within 6 hours of ingesting food is most likely caused by preformed toxins by the organism, -- -- . This patient should be treated with supportive measures such as intravenous fluids and antiemetics.

Diarrhea within 6 hours of ingesting food is most likely caused by preformed toxins by the organism, Staphylococcus aureus. This patient should be treated with supportive measures such as intravenous fluids and antiemetics.

equivocal fast exam do what next

Dx peritoneal lavage

--- is the most common malignant germ cell neoplasm; it is often seen in Turner's syndrome and presents with elevated hCG and LDH

Dysgerminoma is the most common malignant germ cell neoplasm; it is often seen in Turner's syndrome and presents with elevated hCG and LDH

rash after amox=

EBV mono

-- -- occurs secondary to P. aeruginosa and presents with hemorrhagic pustules and erythema that can progress to a necrotic ulcer.

Ecthyma gangrenosum occurs secondary to P. aeruginosa and presents with hemorrhagic pustules and erythema that can progress to a necrotic ulcer. This patient is presenting with physical exam findings suggestive of ecthyma gangrenosum. Ecthyma gangrenosum presents with hemorrhagic pustules and erythema that can progress to become a necrotic ulcer. This pathology occurs secondary to a Pseudomonas aeruginosa infection typically in a predisposed patient. Risk factors in a patient's history for this pathology include a immunocompromised state, hematologic malignancies, burns, malnutrition, chemotherapy, and immunosuppressive therapy. Treatment for this pathology should include anti-pseudomonal antibiotics such as piperacillin-tazobactam in conjunction with gentamicin.

IUGR, microcephaly, rocker bottom feet, clenched fists overlap fingers, VSD, prominent occiput

Edwards, trisomy 18 microphthalmia is patau 13.

ESR and CK in PMR?

Elev ESR, normal CK

pcos inc risk which CA

Endometrial CA

PCOS increases risk of what cancer?

Endometrial cancer Oligo-/anovulation and less progesterone than normal -> more estrogen without progesterone -> endometrial hyperplasia with breakthrough bleeding -> endometrial cancer

-- --- presents with asthma, cough, renal abnormalities, and peripheral eosinophilia and disease activity correlates with p-ANCA levels.

Eosinophilic granulomatosis with polyangiitis (Churg-Strauss syndrome) presents with asthma, cough, renal abnormalities, and peripheral eosinophilia and disease activity correlates with p-ANCA levels.

what is a woman risk factor for cholecystitis?

Estradiol

-- is a tumor marker for granulosa cell ovarian tumors.

Estrogen is a tumor marker for granulosa cell ovarian tumors. The patient is presenting with evidence of hyperestrogenism (e.g., spider angiomata, swollen breasts, and menorrhagia) with a thickened endometrial stripe and an ovarian mass, which is suspicious for a granulosa cell ovarian tumor. Estrogen is a tumor marker associated with granulosa cell tumors. Granulosa cell tumors arise from the sex cord stroma. They produce estrogen, which causes symptoms of hyperestrogenism/hyperestinism. Adults may present with menorrhagia, postmenopausal bleeding, swollen/tender breasts, and spider angiomata. Ultrasound will show an ovarian mass and may reveal ascites and endometrial hyperplasia (also caused by the high levels of estrogen).

SCD pts need ---

FOLATE. B9 The most likely diagnosis in this patient is sickle cell disease (SCD). The best supplement to give this patient is vitamin B9 (folate). This patient presents with fatigue, conjunctival pallor, and normocytic anemia with appropriate reticulocytosis, which are classic findings found in SCD. Management of SCD has multiple components. Despite the normal MCV, observational studies of patients with SCD have shown a high rate of folate deficiency, which is why all patients should receive folate supplementation. Due to the high incidence of asplenia or functional hyposplenism, patients should receive additional vaccinations against S. pneumoniae and N. meningitidis as well as prophylactic penicillin until 5 years of age. Most patients should also receive hydroxyurea to increase the production of fetal hemoglobin to reduce the risk of acute crises. Of note, although this patient presents with reticulocytosis, patients with SCD may also present with a low reticulocyte count in an aplastic crisis, which is most commonly caused by infection with parvovirus B19.

elev FSH, amenorrhea, dx?

FSH is high and ovaries aren't responding bc pt has TS! 45, X

-- -- -- -- should be suspected in any pediatric patient who has repeat infections that are atypical or not expected epidemiologically.

Factitious disorder by proxy should be suspected in any pediatric patient who has repeat infections that are atypical or not expected epidemiologically.

Hypochondriasis, or illness anxiety disorder, is persistent fear and anxiety over having or getting diseases without any or minimal symptoms.

Factitious disorder is a condition in which a person makes herself or himself ill or exaggerates symptoms in order to gain attention or sympathy - this patient did not intentionally make herself sick. The patient is most likely suffering from somatic symptom disorder - a condition where a patient experiences physical symptoms suggestive of injury or illness that cannot be explained by a medical condition, an alternative mental disorder, or the use of substances. somatic: something there we don't know what tho.

hartnup disease

Failed absorption of tryptophan in intestine/kidneys, that causes pellagra from niacin deficiency Tx: high protein diet, nicotinic diet

in HRS vs other conditions, what is lower?

FeNa

Febrile non-hemolytic transfusion reaction occurs secondary to cytokines that are release by leukocytes from a blood transfusion. Patients present within 1-6 hours of transfusion with symptoms of fever, chills, and malaise without signs or symptoms of hemolysis. To avoid this complication, --- (removal of leukocytes from tranfused products) should be performed on the blood products prior to transfusion.

Febrile non-hemolytic transfusion reaction occurs secondary to cytokines that are release by leukocytes from a blood transfusion. Patients present within 1-6 hours of transfusion with symptoms of fever, chills, and malaise without signs or symptoms of hemolysis. To avoid this complication, leukoreduction (removal of leukocytes from tranfused products) should be performed on the blood products prior to transfusion. Acute hemolytic transfusion reaction (ABO incompatibility) presents with severe symptoms and signs of hemolysis which occur during a transfusion.

RA Splenomegaly Neutropenia What syndrome?

Felty syndrome- NAS

Females in their 4th decade of life can experience decreased fertility secondary to ---

Females in their 4th decade of life can experience decreased fertility secondary to a decreased ovarian reserve.

which ssri has no weight gain

Fluoxetine

Folic acid is frequently depleted in alcoholic patients and is a cofactor for the enzyme --- ----

Folic acid is frequently depleted in alcoholic patients and is a cofactor for the enzyme methionine synthase.

-- is a reduced form of folic acid used to "rescue" patients from the macrocytic anemia caused by methotrexate.

Folinic acid (leucovorin) is a reduced form of folic acid used to "rescue" patients from the macrocytic anemia caused by methotrexate.

Furuncles/carbuncles should be treated with ---

Furuncles/carbuncles should be treated with incision and drainage as well as anti-staphylococcal antibiotics (dicloxacillin and cefadroxil).

aerobic g+ cocci in chains

GAS

mcc organism in nec fasc

GAS

heartburn, belching, epigastric pain

GERD

--- is a protozoan parasite that is a common cause of persistent enteric disease among international travelers and hikers in the United States. It is transmitted by water, food, or the fecal-oral route. Patients will present after an incubation period of at least 1 week. The disease often becomes chronic when untreated. Symptoms include persistent, foul-smelling steatorrhea, weight loss, abdominal cramping, flatulence, belching, and possibly acquired LACTOSE intolerance. Tinidazole and metronidazole can be used as treatment. Both have an associated disulfiram-like reaction, so alcohol should be avoided. FLAGYL: DISULFIRAM

Giardia lamblia is a protozoan parasite that is a common cause of persistent enteric disease among international travelers and hikers in the United States. It is transmitted by water, food, or the fecal-oral route. Patients will present after an incubation period of at least 1 week. The disease often becomes chronic when untreated. Symptoms include persistent, foul-smelling steatorrhea, weight loss, abdominal cramping, flatulence, belching, and possibly acquired lactose intolerance. Tinidazole and metronidazole can be used as treatment. Both have an associated disulfiram-like reaction, so alcohol should be avoided.

Given that the patient is a poor surgical candidate and that radiation therapy is highly effective for vaginal squamous cell carcinoma, -- therapy is the most appropriate next step in management.

Given that the patient is a poor surgical candidate and that radiation therapy is highly effective for vaginal squamous cell carcinoma, radiation therapy is the most appropriate next step in management.

-- -- are benign neoplasms of the --- body, which regulates body temperature and present with tenderness and pain that is worse with cold temperatures.

Glomus tumors are benign neoplasms of the glomus body, which regulates body temperature and present with tenderness and pain that is worse with cold temperatures.

all preg women should get tested for

HIV syphhyllis hep b

mcc ITP

HIV & Hep C

common cause of ITP:

HIV,Hep C

pt has dep c cirrhosis, ascites, recent varices, now poor urine output and elev cr unresponsive to fluid=

HRS: prog renal failure, low urine Na, hyponatremia, oliguria. only rx is liver transplant.Use vaptans, and vasopressin antag to respolve hyponat and ascites.

in esrd, epo causes

HTN

OCP SE

HTN, HA

antimicrosomal ab

Hashimoto's

Hepatic encephalopathy is a decline in mental status secondary to liver failure. It occurs when the liver is unable to clear the blood of toxic metabolites, especially ammonia. There are multiple factors that may cause a patient with liver failure to become encephalopathic -- one important one is --. Other precipitating factors include alkalosis, the use of sedative or CNS-altering drugs, infection, and hypovolemia (e.g. from a GI bleed). Loop diuretics, such as this patient's furosemide, predispose to hypokalemia.

Hepatic encephalopathy is a decline in mental status secondary to liver failure. It occurs when the liver is unable to clear the blood of toxic metabolites, especially ammonia. There are multiple factors that may cause a patient with liver failure to become encephalopathic -- one important one is hypokalemia. Other precipitating factors include alkalosis, the use of sedative or CNS-altering drugs, infection, and hypovolemia (e.g. from a GI bleed). Loop diuretics, such as this patient's furosemide, predispose to hypokalemia.

breast CA, post menopausal women Her2 Neu rx ER+ rx

Her 2 Neu Trastuzamab ER+ anastrozole, tamoxifen

-- --- is a pruritic blistering dermatologic disorder unique to pregnancy. It is not associated with past or present herpes infection. The onset is generally later in pregnancy (second trimester or later). The distribution involves the abdomen or extremities but may be generalized. It is thought to be an autoimmune condition triggered by pregnancy, with antibodies against hemidesmosomal proteins. It is associated with other autoimmune diseases such as Hashimoto's thyroiditis, Grave's disease, or pernicious anemia.

Herpes gestationis is a pruritic blistering dermatologic disorder unique to pregnancy. It is not associated with past or present herpes infection. The onset is generally later in pregnancy (second trimester or later). The distribution involves the abdomen or extremities but may be generalized. It is thought to be an autoimmune condition triggered by pregnancy, with antibodies against hemidesmosomal proteins. It is associated with other autoimmune diseases such as Hashimoto's thyroiditis, Grave's disease, or pernicious anemia. its centered around umbilicus and spreads to abdomen

-- -- is a viral infection of the hand caused by either type 1 or type 2 herpes simplex virus (HSV). The classic presentation is a painful, ulcerative lesion on one or more fingers.

Herpetic whitlow is a viral infection of the hand caused by either type 1 or type 2 herpes simplex virus (HSV). The classic presentation is a painful, ulcerative lesion on one or more fingers.

High baby Hct w neonatal polycythemia rubra Vera rx

Hydration partial exchange transfusion

test for lactose intolerance

Hydrogen breath test +

-- presents with anxiety, palpitations, weight loss, hyperhidrosis, and sometimes hypercalcemia.

Hyperthyroidism presents with anxiety, palpitations, weight loss, hyperhidrosis, and sometimes hypercalcemia.

multiple blood transfusions can cause what Ca abnormal

Hypocalcemia, causing hyperactive reflexes

Neumann pick reflexia

Hyporeflexia

EPO for anemia CKD, risk of dev

IDA, renewwerd eryropoeises depletes Fe stores in bone marrow, so dec MCV, inc RDW, dec fear, dec Trans sat. Before beginning treatment for anemia of chronic disease in chronic kidney disease (CKD), all patients should be evaluated for iron deficiency anemia, which would present with low MCV, increased RDW, decreased ferritin, and decreased transferrin saturation. Patients should continue to receive maintenance iron supplementation throughout therapy, as the renewed erythropoiesis depletes iron stores in the bone marrow.

inital rx all rhin

IN steroids

lateral knee tender dx

IT band syn

lyme meningitis rx?

IV Ceftriaxone

rx asa tox

IV Na Bicarb NAC for acet toe

neutropenic fever rx

IV ceftazidime

rx acute dystonia

IV diphenhydramine or benztropine

pid and TOA rx?

IV doxy and IV cefoxitin

rx malig otitis externa, if CN involvement and droop?

IV fluroquinolone

eye sx = methanol: rx?

IV fomepizole, or etOH

PCP (jirovecii) pt allergy to sulfa (bactrim included i guess), rx w

IV pentamidine and steroids for severe cases

rx acute MS flare

IV solumedrol

HIV pts, Pneumococcal polysac vaccine induces ---- response

In HIV patients, pneumococcal polysaccharide vaccine is effective (it induces a T-cell independent B-cell response) and indicated (it prevents the serious morbidity or mortality that may occur with pneumonia in HIV). The other vaccines listed are live vaccines and therefore contraindicated in HIV-positive patients with a CD4 count <200 cells/mm^3. The pneumococcal polysaccharide vaccine is recommended in all adults over 65 and in younger patients with comorbid conditions that increase the risk of pneumococcal infection. In healthy adults, 1 dose is given after age 65. In patients with HIV, a 1-time revaccination is given 5 years after the initial dose. The vaccine contains capsular polysaccharides of the 23 most common types of pneumococcus and induces a T-cell independent B-cell response. The vaccine has been shown to be effective against invasive pneumococcal disease such as meningitis and bacteremia; its efficacy against pneumonia is controversial.

In a fair-skinned child with a Marfanoid habitus and mental retardation or developmental delay, suspect --- . rx --.

In a fair-skinned child with a Marfanoid habitus and mental retardation or developmental delay, suspect homocystinuria. If the disease is due to a decreased affinity of cystathionine beta synthase for vitamin B6, high-dose vitamin B6 can improve long-term survival. Homocystinuia is an autosomal recessive disease of methionine metabolism resulting in the build-up of homocystine in blood and urine. Patients are normal at birth, but then develop failure-to-thrive and developmental delay. Signs and symptoms include fair skin, blue eyes, mental retardation, osteoporosis, inferior lens dislocation (Figure A), Marfanoid habitus, seizures, and premature atherosclerosis with predisposition to thromboembolic events. The disease can be caused by one of three defects of homocystine metabolism, which causes inconsistency in the effectiveness of high dose vitamin B6 therapy.

In a patient with Crohn's disease a palpable mass is concerning for an abdominal -- and -- is the diagnostic test of choice.

In a patient with Crohn's disease a palpable mass is concerning for an abdominal abscess and computed tomography (CT) is the diagnostic test of choice.

In a patient with HIV who is non-adherent to medical therapy, presenting with focal neurologic symptoms, and a contrast CT notable for a ring-enhancing lesion, raises suspicion for ----. Given this patient's classic presentation and ring-enhancing lesion found on brain imaging, the next best step is to treat the toxoplasmosis with pyrimethamine-sulfadiazine.

In a patient with HIV who is non-adherent to medical therapy, presenting with focal neurologic symptoms, and a contrast CT notable for a ring-enhancing lesion, raises suspicion for CNS toxoplasmosis. Given this patient's classic presentation and ring-enhancing lesion found on brain imaging, the next best step is to treat the toxoplasmosis with pyrimethamine-sulfadiazine.

In a patient with a new diagnosis of hypertension, the physician should order a --

In a patient with a new diagnosis of hypertension, the physician should order a urinalysis for signs of long-term kidney damage as the result of elevated blood pressure.

In a patient with a provoked DVT associated with a transient risk factor (i.e., surgery), the minimum recommended anticoagulation duration is -- months.

In a patient with a provoked DVT associated with a transient risk factor (i.e., surgery), the minimum recommended anticoagulation duration is three months. The duration of warfarin therapy for DVT depends on the patient's risk of developing a subsequent DVT. Reversible or transient risk factors include trauma, recent surgery, prolonged immobilization, and certain medication including OCPs. Nontransient factors include hypercoagulability disorders, malignancy, and past history of VTE. For a first DVT and for DVTs associated with transient risk factors, the recommended anticoagulation duration is specifically 3 months. If not, treatment period is considered "extended" (i.e. indefinite).

The patient is presenting with an imaging-confirmed thyroid nodule and low TSH, which raises the suspicion for a hyperfunctioning thyroid nodule. The next step in management would be thyroid scintigraphy. Initial diagnostic evaluation of a thyroid nodule after history and physical exam includes a serum TSH level and a thyroid ultrasound. If the serum TSH concentration is below normal, this indicates overt or subclinical hyperthyroidism, and a thyroid scintigraphy should be performed next. Scintigraphy/radionuclide scan utilizes either radioisotopes of iodine or technetium-99m pertechnetate (the former is preferred when available) to demonstrate thyroid function. Hyperfunctioning nodules are rarely cancerous, therefore if a nodule is hyperfunctioning (hot) on radionuclide imaging, this does not require fine needle aspiration (FNA). However, some clinicians may elect to perform FNA if the ultrasound shows signs suspicious for cancer.

In a patient with a thyroid nodule confirmed by ultrasound and a low TSH, a radionuclide thyroid scan is the next step in management.

In a patient with very high suspicion for C. difficile but whose toxin PCR comes back negative, a limited sigmoidoscopy or colonoscopy should be performed to confirm diagnosis by observing --- ---This patient presents with short stature, low hairline, and a broad chest, which suggests a diagnosis of Turner syndrome (45, X). Patients with Turner syndrome typically have absent Barr bodies on buccal smear.

In a patient with very high suspicion for C. difficile but whose toxin PCR comes back negative, a limited sigmoidoscopy or colonoscopy should be performed to confirm diagnosis by observing pseudomembranous colitis.

In adults with cystic fibrosis, ---- is the most common cause of pneumonia.

In adults with cystic fibrosis, Pseudomonas aeruginosa is the most common cause of pneumonia. s aureus in children

In patients who are in shock or have compromised cardiac output, initiation of-- -- can result in a profound decrease in preload and worsen systemic perfusion.

In patients who are in shock or have compromised cardiac output, initiation of positive pressure mechanical ventilation can result in a profound decrease in preload and worsen systemic perfusion. cpap and bipap dec AFL, wall stress, improve oxygenation, Cardiogenic pulmonary edema can be addressed with intravenous diuresis and noninvasive ventilation. Central positive airway pressure (CPAP) and BiPAP both decrease afterload, decrease wall stress, improve oxygenation, and improve dyspnea - they have been shown to lower in-hospital mortality. In a naive patient, BiPAP is typically started at 10/5 cm H2O. As with intubated ventilation, positive pressure from noninvasive ventilation increases intrathoracic pressure, decreases venous return, and decreases stroke volume and cardiac output. This patient's PIP settings are aggressively high, and should be decreased. so + pressure inc IT pressure, dec preload venous return and CO.

In patients with breast cancer, the --- intrauterine device and condoms are appropriate contraceptive options due to the lack of hormones.

In patients with breast cancer, the copper intrauterine device and condoms are appropriate contraceptive options due to the lack of hormones.

In physiologic jaundice of the newborn, total bilirubin usually remains < -- mg/dL, is benign, and is managed by observation. its mostly indirect BR

In physiologic jaundice of the newborn, total bilirubin usually remains < 12 mg/dL, is benign, and is managed by observation.

In the diagnostic workup of hypercortisolism, -- -- -- -- should be pursued if laboratory data supports high cortisol/ACTH levels but no radiographic source can be found.

In the diagnostic workup of hypercortisolism, inferior petrosal sinus sampling should be pursued if laboratory data supports high cortisol/ACTH levels but no radiographic source can be found.

Her symptoms of neck flexion, clenched jaw, and upward deviation of the eye on convergence are classic for an -- -- , one of the manifestations of acute dystonia. Others include spasmodic torticollis, in which the neck is involuntarily turned to one side, and oromandibular dystonia. Acute dystonia is one type of the extrapyramidal symptoms (EPS) caused by dopamine antagonism. In this patient's case, the most likely culprit is metoclopramide, and the most appropriate management is discontinuation of the offending drug and administration of --- or another anticholinergic medication.

In this case the patient's adverse event is the key to understanding the pharmacology underlying her symptoms. Her symptoms of neck flexion, clenched jaw, and upward deviation of the eye on convergence are classic for an oculogyric crisis, one of the manifestations of acute dystonia. Others include spasmodic torticollis, in which the neck is involuntarily turned to one side, and oromandibular dystonia. Acute dystonia is one type of the extrapyramidal symptoms (EPS) caused by dopamine antagonism. In this patient's case, the most likely culprit is metoclopramide, and the most appropriate management is discontinuation of the offending drug and administration of benztropine or another anticholinergic medication.

pt infertile, T shaped uterus:

In utero exposure DES

Increasing the cut-off point of a test leads to an increase in test --- and a decrease in --- .

Increasing the cut-off point of a test leads to an increase in test specificity and a decrease in sensitivity. so more spec means inc FN, dec FP

Infectious mononucleosis can be complicated by a variety of hematologic abnormalities, including cold autoimmune hemolytic anemia (AIHA), thrombocytopenia, or aplastic anemia. Cold AIHA can present with livedo reticularis or acrocyanosis upon exposure to the cold, and it can be diagnosed by peripheral smear or the positive direct (Coombs) test for the presence of bound complement on red blood cells. The treatment for moderate to severe disease is --.

Infectious mononucleosis can be complicated by a variety of hematologic abnormalities, including cold autoimmune hemolytic anemia (AIHA), thrombocytopenia, or aplastic anemia. Cold AIHA can present with livedo reticularis or acrocyanosis upon exposure to the cold, and it can be diagnosed by peripheral smear or the positive direct (Coombs) test for the presence of bound complement on red blood cells. The treatment for moderate to severe disease is rituximab.

----- ---- --- present in children as grade I/II, systolic murmurs that decrease with standing, and require only parental reassurance.

Innocent cardiac murmurs present in children as grade I/II, systolic murmurs that decrease with standing, and require only parental reassurance.

Intense symptoms of depression and guilt that persist beyond 12 months following the death of a loved one is indicative of -- --- --- -.

Intense symptoms of depression and guilt that persist beyond 12 months following the death of a loved one is indicative of persistent complex bereavement disorder. Not only do symptoms persist beyond 12 months, but they are generally severe in nature and interfere with daily functioning.

Intracranial abscesses are well-recognized complications of rhinosinusitis and occur most commonly due to what organism?

Intracranial abscesses are well-recognized complications of rhinosinusitis and occur most commonly due to streptococcal species. staph when penetrating head trauma

rx keloids

Intralesional steroids

bicarb up, K does what

K up low H = low K KH

4-10, painless limp, pelvic muscle atrophy, shortening affected limb, hip stiffness.

LCP Slipped capital femoral epiphysis (SCFE) presents with a similar waddling gait, and decreased range of motion but is more common between the ages 11-15 and is often seen in obese children. Osgood-Schlatter disease is partial avulsion of the tibial apophysis typically found in adolescents who are involved in athletic activity.

rx NPH

LPs

Worsening inspir stridor improves w prone pos

Laryngomalaci

--- are a common cause of dating errors in pregnancy, as they may misleadingly increase the fundal height. Ultrasound should be performed to confirm the gestational age based on fetal biometry.

Leiomyomata are a common cause of dating errors in pregnancy, as they may misleadingly increase the fundal height. Ultrasound should be performed to confirm the gestational age based on fetal biometry. Answer 1: Molar pregnancy, particularly a complete hydatiform mole, is a possible cause of a size greater than dates uterus due to abnormally proliferating placental tissue. However, this patient's -hCG is appropriate for a 12-week pregnancy, where in complete molar pregnancies, levels often surpass 100,000 mIU/mL. In addition, ß-hCG's cross-reactivity with the thyroid hormone receptor causes low TSH with signs of hyperthyroidism, both of which are absent in this patient. She also has not had hyperemesis gravidarum resulting in weight loss, which may occur in molar pregnancies as well.

dec libido, test nodule, mass, dx? this makes what? LH levels?

Leydig cell tumor. This tumor classically produces testosterone, which is then aromatized into estrogen, resulting in suppression of luteinizing hormone. Excess estrogen in a male can arise from an estrogen-producing tumor, or from a testosterone-producing tumor with peripheral aromatization to estrogen. Other symptoms of excess estrogen may include infertility, testicular atrophy, and feminization of hair patterns. In a patient with a testicular mass that is palpated on exam and confirmed on ultrasound, the most likely diagnosis is a Leydig cell tumor, a sex cord stromal tumor. Because of the excess estrogen, luteinizing hormone and follicle stimulating hormone will be low as a result of negative feedback.

Lung -- classically presents with hemoptysis and weight loss in a non-smoker with a peripheral mass seen on chest radiograph.

Lung adenocarcinoma classically presents with hemoptysis and weight loss in a non-smoker with a peripheral mass seen on chest radiograph.

Pregnant pt high testosterone w Pepcid nass

Luteoma

GCS Score

M6V5E4

hodgekins lymphoma assoc w

MCD

hodgekin lymphtoma assoc w

MCD. rx steorids

Pyloric Sten, NONBILIOUS PS IS NB!!!!! BILIOUS VOMIT US GASLESS ABD IS

MIDGUET VOLVULUS, DO UPPER GI SERIES

hep b and hep c tram track

MPGN Proliferate the tram tracks!!!

"tram tracking" dec C3

MPGN proliferate the tram tracks! dense deposit in type 2. Hep C assoc w MPGN, mix ess cryo, PCT

young male, low back pain worse in am, relieved w exercise

MRI SI joint, its ankylosing spondylitis

ADPKD assoc w

MVP

ADPKD assoc w what heart murmur

MVP

which murmur dec with squatting

MVP HOCM less common than MVP

Management of early Parkinson's that is dominated by motor symptoms usually involves an anticholinergic medication like -- (or ---). As the disease progresses to involve other motor symptoms (bradykinesia, postural instability) and cognitive symptoms, management includes medications that increase the levels of dopamine in the central nervous system like levodopa and carbidopa.

Management of early Parkinson's that is dominated by motor symptoms usually involves an anticholinergic medication like trihexyphenidyl (or benztropine). As the disease progresses to involve other motor symptoms (bradykinesia, postural instability) and cognitive symptoms, management includes medications that increase the levels of dopamine in the central nervous system like levodopa and carbidopa.

The newborn is presenting with vaginal bleeding, white vaginal discharge, breast hypertrophy, and swollen labia, which suggests the physiologic effects of maternal estrogen withdrawal. Maternal estrogen can cause temporary changes in newborns such as breast enlargement, labial swelling, and physiologic leukorrhea. As the newborn experiences maternal estrogen withdrawal, newborn girls may also have uterine withdrawal bleeding. All changes should slowly resolve within the first 2 months of life.

Maternal estrogen in newborns can cause breast hypertrophy, swollen labia, physiologic leukorrhea, and uterine withdrawal bleeding.

-- typically present in children as a cerebellar midline mass that results in a non-obstructive hydrocephalus due to compression of the 4th ventricle and truncal ataxia.

Medulloblastomas typically present in children as a cerebellar midline mass that results in a non-obstructive hydrocephalus due to compression of the 4th ventricle and truncal ataxia.

hep B, hep C, spike and dome

Membranous Gn all members have spike and dome necklace

Spike and dome on EM

Membranous glomerulonephritis

vertigo, tinnitus, hearing loss,

Meniere's disease: low freq SN hear loss Medical treatment for Meniere's typically involves dietary modifications to reduce salt intake, diuretics, and medications for symptomatic relief of vertigo.

-- --- is distinguished from PolyArtNod by the presence of microscopic vessel involvement, especially skin, lung, and glomerular vessels. 70% of patients are ANCA positive

Microscopic polyangiitis is distinguished from PAN by the presence of microscopic vessel involvement, especially skin, lung, and glomerular vessels. 70% of patients are ANCA positive

Podocyte Effacement and fatty cast (Electron Microscopy), no changes on light microscopy seen w/

Minimal Change (Lipoid nephrosis) disease

Palpable purpura, GN, arthralgia, HSM, perish neuro, dec Comp, Hep C

Mixed Ess Cryo

Acute mania Rx

Mood stab and atypical AP so Li and Olanzapine

primary amenorrhea, no uterus, normal karyotype and testos=

Mullerian agenesis

adderrall inc what NT?

NE

premature baby, change feeding tol, lethargy, abd dis, hematochezia:

NEC: pneumatosis intestinalis: gas bubbles in bowel.

vitamin B12 deficiency secondary to the fish tape worm Diphyllobothrium latum. Folate can correct an anemia secondary to vitamin B12 deficiency, but there will still be progression of neurologic complications without correction of B12 levels as well.

NSAID use and aspirin can predispose individuals to gastritis and GI bleeding leading to iron deficiency anemia.

rx for Reiters syn

NSAIDS

--- presents with a classic tetrad of daytime sleepiness, hypnagogic and/or hypnopompic hallucinations, sleep paralysis, and cataplexy.

Narcolepsy presents with a classic tetrad of daytime sleepiness, hypnagogic and/or hypnopompic hallucinations, sleep paralysis, and cataplexy (brief sudden loss muscle tone due to rapid induction REM sleep).

-- presents with colicky flank pain and hematuria, and the best diagnostic test in a pregnant patient is ultrasound.

Nephrolithiasis presents with colicky flank pain and hematuria, and the best diagnostic test in a pregnant patient is ultrasound.

--- --- --- occurs secondary to antipsychotic use and presents with delirium, high fever, muscle rigidity, and autonomic instability.

Neuroleptic malignant syndrome occurs secondary to antipsychotic use and presents with delirium, high fever, muscle rigidity, and autonomic instability. The anesthetic version is malignant hyperthermia.

New-onset ascites identified in patients should have a paracentesis to assess cell count, total protein, albumin, and bacterial culture and sensitivity. A serum-ascites albumin gradient is greater than or equal to 1.1 g/dL confirms the diagnosis of ---.

New-onset ascites identified in patients should have a paracentesis to assess cell count, total protein, albumin, and bacterial culture and sensitivity. A serum-ascites albumin gradient is greater than or equal to 1.1 g/dL confirms the diagnosis of cirrhotic portal hypertension ascites or heart failure cirrhosis. SAAG > 1.1 = portal HTN. Causes are cirrhosis, large liver mets, HF, Budd Chiari <1.1: perit CA, infxn, Nephrotic syndrome, pancreatic ascites

def of sphingomyelinase

Niemann-Pick SPLENOMEG CHERRY RED MACULA Tay sach (GM2 ganglioside), dec hexosaminidase A. also has crm, but no HSM, or cerv LAD.

Non-displaced scaphoid fractures are managed with a thumb spica cast and displaced fractures are managed with open reduction and internal fixation.

Non-displaced scaphoid fractures are managed with a thumb spica cast and displaced fractures are managed with open reduction and internal fixation.

Norepinephrine is a potent α1 agonist, which causes increased SVR, reflex bradycardia, but also can cause ---- ischemia.

Norepinephrine is a potent α1 agonist, which causes increased SVR, reflex bradycardia, but also can cause peripheral (digital) ischemia.

hepatic adenomas risk factors

OCP anabolic steroids

case control uses cohort uses

Odds Ratio CC OR Cohort RR

Open angle glaucoma is characterized by peripheral, then central PAINLESS vision loss, and is associated with increasing age.

Open angle glaucoma is characterized by peripheral, then central PAINLESS vision loss, and is associated with increasing age.

-- (qualitative defect) presents with pseudofractures, muscle pain, and lab abnormalities (hypocalcemia, hypophosphatemia, increased PTH, increased alkaline phosphatase levels) in contrast to osteoporosis (quantitative defect without lab abnormalities). High PTH and Alk Phos dec Ca and Phos

Osteomalacia (qualitative defect) presents with pseudofractures, muscle pain, and lab abnormalities (hypocalcemia, hypophosphatemia, increased PTH, increased alkaline phosphatase levels) in contrast to osteoporosis (quantitative defect without lab abnormalities).

This patient is presenting with diffuse musculoskeletal symptoms in the setting of laboratory abnormalities (hypocalcemia) suggesting a diagnosis of osteomalacia. Patients with celiac disease often do not absorb fat-soluble vitamins (including vitamin D) and can be IgA deficient which can lead to anaphylaxis when receiving a transfusion. Osteomalacia presents with diffuse musculoskeletal tenderness/pain, difficulty walking, and other non-specific musculoskeletal findings. In contrast to osteoporosis, osteomalacia presents with laboratory abnormalities including a decreased calcium, phosphorus, and vitamin D level, and an increased alkaline phosphatase and PTH level. Anything that causes fat malabsorption can cause decreased absorption of fat-soluble vitamins including vitamin D. Celiac disease is a common cause of fat malabsorption and is associated with decreased IgA levels. If an IgA deficient patient receives a transfusion from a donor with normal levels of IgA, they can experience anaphylaxis.

Osteomalacia presents with diffuse musculoskeletal symptoms and laboratory abnormalities (low calcium/phosphorus and high PTH/alkaline phosphatase) and is associated with fat malabsorption (such as in celiac disease, Crohn disease, and pancreatitis).

-- is associated with sickle cell disease and presents with frequent urination due to an inability to concentrate urine.

Other symptoms associated with SCD include fatigue, dactylitis, splenomegaly, priapism, and bone pain.

--- presents with conductive hearing loss and typically occurs in young female patients. Bone conduction would be louder in the Rinne test, and the Weber test would localize to the affected ear.

Otosclerosis presents with conductive hearing loss and typically occurs in young female patients. Bone conduction would be louder in the Rinne test, and the Weber test would localize to the affected ear. Otosclerosis is a hereditary disorder that results in conductive hearing loss, which will show BC > AC of the affected ear on Rinne testing, and sound lateralizing to the affected ear on Weber testing.

scleroderma related to

PBC, intranet biliary obs. fatigue prurits, jaundice.

Non-suppurative destructive cholangitis and progressive loss of intrahepatic bile ducts are the hallmark of ---.

PBC.

new onset LBBB w chest pain

PCI, likely LAD occlusion, masking MI findings.

pt has syphillis, now rx?

PCN 3 shots f/u titers 6 mo

dark urine, skin rash hands, inc EtOH use, fragile skin photons, dx rx?

PCT, rx w hydroxychloroquine

rx 3rd deg heart block is

PM

increase ESR, muscle stiffness= dx and rx?

PMR, steroids aldolase in polydermatomyositis when profound muscle weakness present.

Periductal portal tract fibrosis and segmental stenosis of extrahepatic and intrahepatic bile ducts 37% (28/76)

PSC

enlarged, hypercellular glomeruli seen in

PSGN

rx of graves pregnancy

PTU 1st trimester ESPCIALLY NOT METHIMAZOLE IN 1ST TRIMESTER 1p 2m 3m

pertussis coughing can cause what

PTX

bone pain, elev alk phos, normal Ca =

Pagets ds bone Pagets disease of the bone presents with bone pain in the setting of an elevated alkaline phosphatase and a normal calcium/phosphate. The diagnosis can be supported with a high urine hydroxyproline. Radiography will reveal osteolytic/sclerotic lesions, and bone scan will demonstrate increased radiotracer uptake. Complications of Pagets disease of the bone include hearing loss (due to temporal bone enlargement and loss of bone mineral density in the cochlear capsule), headache, spinal stenosis, radiculopathy, leg bowing, fracture, osteosarcoma, and giant cell tumor of the bone. a

MVA trauma complication

Panc pseudocyst from trauma

------ classically presents with diffuse anterior knee pain after overuse or prolonged sitting; compression of the patella during knee extension will reproduce the pain.

Patellofemoral pain syndrome classically presents with diffuse anterior knee pain after overuse or prolonged sitting; compression of the patella during knee extension will reproduce the pain. Note worse w prolonged sitting or climbing stairs.

Patients on immunosuppresive therapy after a transplant should be vaccinated (pneumovax, hepatitis A and B, influenza, and tetanus-diphtheria) and started on TMP-SMX.

Patients on immunosuppresive therapy after a transplant should be vaccinated (pneumovax, hepatitis A and B, influenza, and tetanus-diphtheria) and started on TMP-SMX. This patient is presenting status post renal transplant and is currently on immunosuppressive therapy. This patient should be started on TMP-SMX as prophylaxis against Pneumocystis jirovecci pneumonia (PJP). After a transplant procedure, patients are started on immunosuppressive therapy in order to avoid rejection of the transplanted organ. The most important interventions for a patient status post transplant procedure are to receive all of their vaccinations and to receive TMP-SMX as prophylaxis against PJP. Other appropriate but less dire measures include giving ganciclovir as prophylaxis against cytomegalovirus (CMV).

Patients presenting with asthma attacks in the setting of an increasing PaCO2 should be -- as their respiratory muscles are fatiguing.

Patients presenting with asthma attacks in the setting of an increasing PaCO2 should be intubated as their respiratory muscles are fatiguing.

Patients undergoing AAA repair are at risk of -- -- post-operatively.

Patients undergoing AAA repair are at risk of ischemic bowel post-operatively. This is most likely seen due to decreased collateral circulation through the mesenteric system. Depending on the location of the aneurysm, the inferior mesenteric artery is often tied off during this operation and ischemia can ensue if insufficient collateral circulation is absent. In the event of significant vascular disease, emboli can also be sent downstream, occluding the distal-most mesenteric vessels. Pain out of proportion to physical exam is the classic finding of mesenteric ischemia.

Patients with DiGeorge syndrome are at risk for --, which commonly presents with -- --

Patients with DiGeorge syndrome are at risk for hypocalcemia, which commonly presents with neuromuscular irritability (or tetany).

Given his recent cystoscopy in the setting of recurrent UTIs, the most likely source of his bacteremia and, ultimately, SBE, is his urinary tract, making enterococci the most likely organism.

Patients with a history of panic attacks may become agoraphobic after experiencing an attack in a place from which it would be difficult to escape, like a plane.

Patients with a pheochromocytoma should be medically managed prior to surgery with an irreversible alpha-blocker, such as -- , followed by a beta-blocker.

Patients with a pheochromocytoma should be medically managed prior to surgery with an irreversible alpha-blocker, such as phenoxybenzamine, followed by a beta-blocker.

PAD rx? 2 meds

Patients with peripheral arterial disease with vascular risk factors should be started on both antiplatelet therapy with aspirin or clopidogrel and lipid-lowering therapy with a statin. cilostazol not approp as initial med regimen can use if fails exercise therapy

Patients with subacute bacterial endocarditis (SBE) with Streptococcus viridans highly susceptible to penicillin should be treated with IV -- G or IV --

Patients with subacute bacterial endocarditis (SBE) with Streptococcus viridans highly susceptible to penicillin should be treated with IV penicillin G or IV ceftriaxone,

Pelvic trauma is best managed with IV fluids and a pelvic binder to keep the pelvis in place. If the bleeding does not stop, the next best step in management is-- -

Pelvic trauma is best managed with IV fluids and a pelvic binder to keep the pelvis in place. If the bleeding does not stop, the next best step in management is IR vessel embolization.

Antichol OD rx w

Physostigmine Atropine is Anticholinergic and rx chol tox

Pilonidal cysts occur secondary to poor hygiene/exposure to sweat and present as acute abscesses that should be treated with----

Pilonidal cysts occur secondary to poor hygiene/exposure to sweat and present as acute abscesses that should be treated with incision and drainage.

no vag exam on preg pt with ---

Placenta previa occurs when the placenta forms over the cervical os, thus making a natural delivery extremely dangerous. These patients must be delivered by Cesarean section (C-section). Patients who have had a previous pregnancy delivered by C-section are at an increased risk of placenta previa in subsequent pregnancies. Ultrasound must first be performed in this patient to determine the location of the placenta, as a vaginal exam has the potential of affecting the placenta and pregnancy.

Rx of Goodpastures The best initial step in management for Goodpasture's disease is-- -- -- antibody levels.

Plasma exchange last Corticosteroids cyclophosphamide The best initial step in management for Goodpasture's disease is type IV collagen (basement membrane) antibody levels. This patient most likely has Goodpasture disease, in which anti-GBM antibodies attach to the basement membrane in the lungs and kidneys, resulting in hemoptysis and kidney failure. Goodpasture disease is an autoimmune type 2 sensitivity reaction. Lung symptoms usually precede kidney symptoms and include hemoptysis, cough, shortness of breath, and chest pain. Kidney symptoms usually include blood and protein in the urine, unexplained edema, elevated blood pressures. Diagnosis is confirmed with kidney biopsy showing crescentic glomerulonephritis pathology with linear anti-GBM antibodies (Illustration A).

SC patient get what type vaccine

Polysaccharide vaccine w PCN ppx

This infant is presenting with symptoms of Potter sequence, which includes limb deformities, low-set ears, a flattened nose, and pulmonary hypoplasia. This syndrome typically occurs secondary to autosomal recessive polycystic kidney disease, characterized by cystic dilation of the collecting ducts in the kidney.

Potter sequence occurs secondary to renal agenesis, renal malformation, or obstructive uropathy. The lack of urine production causes the fetus to get "smashed up" against the uterus due to a lack of amniotic fluid, causing facial abnormalities, limb abnormalities, and pulmonary hypoplasia. Mortality typically occurs secondary to the pulmonary complications in these patients. Renal aplasia, autosomal recessive polycystic kidney disease, and obstructive uropathy can cause Potter sequence.

Prader-Willi syndrome commonly presents in older children and adolescents with hyperphagia, intellectual disability, and ---.

Prader-Willi syndrome commonly presents in older children and adolescents with hyperphagia, intellectual disability, and hypogonadism.

The patient is presenting at age 6 with acne, breast development, enlarged areolae, pubic hair, advanced bone age, and elevated FSH and LH levels, which is consistent with central precocious puberty. Head CT should be obtained to rule out a secondary cause.

Precocious puberty is defined as the development of secondary sex characteristics before age 9 in boys and age 8 in girls. Girl may develop adrenarche (axillary hair and acne), pubarche (pubic hair), and thelarche (breast development), while boys may develop testicular enlargement and pubic/axillary hair growth. Patients will also have accelerated bone growth and advanced bone age. Central precocious puberty is due to early activation of the hypothalamic-pituitary axis; therefore, FSH and LH are elevated. Central precocious puberty may be idiopathic or secondary to obesity or CNS tumors. Imaging of the head with CT or magnetic resonance imaging (MRI) should be performed to rule out secondary causes.

Preeclampsia greatly increases the risk of ---- due to uteroplacental insufficiency.

Preeclampsia greatly increases the risk of growth retardation due to uteroplacental insufficiency.

how to dec ovarian CA risk, dec ovulation w

Preg OCP Breastfeed

Pregnant women with Sjögren's syndrome are at greater risk of fetal -- --

Pregnant women with Sjögren's syndrome are at greater risk of fetal heart block. antiRho,La

---- results in bilateral sensorineural hearing loss that can result from cochclear hair cell degeneration.

Presbycusis results in bilateral sensorineural hearing loss that can result from cochclear hair cell degeneration. Presbycusis results in bilateral sensorineural hearing loss that can result from cochclear hair cell degeneration. On physical exam, when a tuning fork is placed in the middle of the patient's forehead, sound is appreciated equally on both ears. When a tuning fork is placed by the external auditory canal and subsequently on the mastoid process, air conduction is greater than bone conduction. Air>bone=sn hear loss

This elderly man with well-controlled diabetes presents with a worsened visual acuity on exam. In the setting of a normal fundoscopic exam, he likely has presbyopia.

Presbyopia is age-related decline in proximal vision resulting from loss of elasticity of the lens. Presbyopia is an age-related change in the elasticity of the lens zonules. The zonules connect to the lens periphery and suspend the lens to the surrounding ciliary muscle. With near vision, the ciliary body contracts like a sphincter causing the zonules to relax and the lens to get "rounder." This rounding of the lens increases its refractive ability and allows focusing of near objects. With aging and onset of presbyopia, the lens hardens and loses its ability to round out.

Patients with MS have a number of different disease courses including: relapsing-remitting, primarily or secondarily progressive, or relapsing progressive (Illustration A). Relapsing-remitting MS is thought to be the most benign and is associated with female gender, age of onset < 40, and optic neuritis or sensory symptoms as the first presenting symptoms.

Primary hyperaldosteronism (Conn's syndrome) presents with hypertension that is refractory to treatment as well as hypokalemia and metabolic alkalosis.

-- -- on EKG is the most specific indicator of toxicity in cases of TCA overdose, and the best initial step in management is to administer sodium bicarbonate.

QRS widening on EKG is the most specific indicator of toxicity in cases of TCA overdose, and the best initial step in management is to administer sodium bicarbonate.

UTI in < 2 yr old, or recurrent UTI, do what?

RBUS

hematuria, flank pain, palpable flank mass:

RCC This patient's presentation of unexplained hematuria and flank mass in a smoker over the age of 50 is suggestive of renal cell carcinoma. A CT scan with contrast is the imaging procedure of choice to diagnose and stage this cancer.

all rh ( ) pts should get rhogam

RH -

how to dx acute appendicitis in peds pts

RLQ US

A crescent-shaped nephro path, assoc w goodpastures

RPGN

SLE, crescentic, loop wire

RPGN loop and diffuse diffuse loops rapidly make good (passture) crescents SLE and Goodpasture make Rapid shit

inability to secrete protons in distal tubule: alpha intercalated cells, can be caused by Li and ampho B

RTA 1 replete bicarb

prox tubule can't reabsorb bicarb, piss out bicarb Fanconi Syn

RTA 2 rx thiazide

aldos def or resistance Li ampho B, SLE are causes

RTA 4 rx fludrocortisone

Which RTA has high urine pH?

RTA I

Radioactive iodine ablation with prophylactic glucocorticoids would be an appropriate treatment option for someone with moderate to severe --- as thyroid destruction during ablation can cause worsening of symptoms.

Radioactive iodine ablation with prophylactic glucocorticoids would be an appropriate treatment option for someone with moderate to severe exophthalmos as thyroid destruction during ablation can cause worsening of symptoms. otherwise you don't give steroids before RAI

Restlessness, confusion, tremulousness, psychosis, and generalized seizure are all symptoms consistent with withdrawal from ---.

Restlessness, confusion, tremulousness, psychosis, and generalized seizure are all symptoms consistent with withdrawal from benzodiazepines.

Low-voltage QRS

Restrictive cardiomyopathy presents as congestive heart failure. A classic finding is low-voltage EKG with diminished amplitude. Hereditary hemochromatosis is a reversible cause of restrictive cardiomyopathy and is diagnosed by measuring serum transferrin saturation. Restrictive cardiomyopathy may be idiopathic or result from infiltrative diseases (e.g. amyloidosis), metabolic storage diseases (e.g. hemochromatosis), or endomyocardial fibrosis. It causes diastolic dysfunction, which results in significant physical findings of heart failure out of proportion to the echocardiographic findings of systolic heart failure. The volume of the left ventricle is normal, whereas the ventricular walls may be normal or symmetrically thickened. On physical exam, signs of right heart failure predominate and include JVD, bilateral ankle edema, and tender hepatomegaly. Signs of left sided heart failure, such as pleural effusions and bibasilar rales, may also be present. Chest radiograph and EKG findings are somewhat nonspecific as described in the question stem. MRI is the preferred imaging modality, as it can rule out pericardial disease and identify iron overloaded tissues.

-- --- --- presents with a sudden, monoocular, painless loss of vision with optic disk swelling, dilated veins, cotton wool spots, and retinal hemorrhages.

Retinal vein occlusion presents with a sudden, monoocular, painless loss of vision with optic disk swelling, dilated veins, cotton wool spots, and retinal hemorrhages.

Rheumatoid arthritis is a risk factor for -- and prophylaxis includes --.

Rheumatoid arthritis is a risk factor for osteoporosis and prophylaxis includes bisphosphonates.

Routine screening of cervical length by transvaginal ultrasound at --- weeks gestation is the standard of care for women with previous cervical procedures.

Routine screening of cervical length by transvaginal ultrasound at 18-24 weeks gestation is the standard of care for women with previous cervical procedures for cerv insuff.

high pitch bowel sound tympanitic bs

SBO

tympanic to percussion

SBO

hypoactive bs, abd distension, diffuse tenderness: dx?

SBO- rebound tend

mucocut ulcers, erosions, target shaped lesions 1 week after sulfa drug= ?

SJS

Sacral bruising is an indicator of child abuse and should not be confused with benign -- spots.

Sacral bruising is an indicator of child abuse and should not be confused with benign Mongolian spots.

This patient's middle cerebral artery (MCA) infarction, history of recurrent epistaxis, iron deficiency anemia, telangiectasias of the lips, and a family history of intracerebral and gastrointestinal bleeding is concerning for hereditary hemorrhagic telangiectasias (HHT). He likely has a pulmonary arteriovenous malformation (AVM) that resulted in an MCA stroke from a paradoxical embolus. HHT, also known as Osler-Weber-Rendu syndrome, is an autosomal dominant disorder characterized by recurrent epistaxis, telangiectasias, and vascular malformations. Pulmonary AVMs create right-to-left shunts that do not allow the pulmonary arteries to be oxygenated and a path for paradoxical emboli that may result in cerebral complications (e.g., embolic stroke, transient ischemic attacks, and brain abscesses). The hypoxemia results in secondary polycythemia, which is seen in this patient. These pulmonary AVMs are typically clinically silent. There also seems to be an increased prevalence of migraine headaches in patients with pulmonary AVMs.

Scleroderma renal crisis (SRC) presents with a sudden decrease in renal function in the setting of scleroderma (difficulty using her hands, swollen extremities, and skin without any wrinkles). This patient is presenting with hypertension, hyperkalemia, increased BUN, and increased creatinine suggesting the diagnosis of SRC. In the setting of acute renal failure, lithium should be discontinued as it is cleared by the kidneys and can quickly reach toxic levels. Valproic acid is an appropriate alternative to lithium in such circumstances. Of note, valproic acid does not have the same mortality benefit that lithium does in treating bipolar disorder.

-- --- particularly following rhinoplasty, presents with a whistling noise heard during respiration. The nasal septum derives its blood supply from the the surrounding mucosa. If the mucosa is compromised during nasal surgery, the septum can undergo necrosis and result in a perforation. Perforations in the setting of nasal surgery are usually caused by either a septal hematoma or septal abscess.

Septal perforation, particularly following rhinoplasty, presents with a whistling noise heard during respiration. The nasal septum derives its blood supply from the the surrounding mucosa. If the mucosa is compromised during nasal surgery, the septum can undergo necrosis and result in a perforation. Perforations in the setting of nasal surgery are usually caused by either a septal hematoma or septal abscess.

This patient is presenting with fever, joint pain, and an urticarial rash following administration of amoxicillin, suggesting the diagnosis of serum sickness-like reaction, which is managed by removal of the offending agent and symptomatic treatment with an anti-histamine (diphenhydramine). Serum sickness and serum sickness-like reactions are type III hypersensitivity reactions that classically occur 5-10 days following exposure to rituximab, infliximab, beta-lactams, sulfa drugs, and anti-venoms. Symptoms include fever, arthralgia, lymphadenopathy, an urticarial rash, and possible renal involvement. It is often a self-limited reaction. Management involves removal of the offending agent and symptomatic treatments such as anti-histamines or steroids.

Serum sickness and serum sickness-like reactions should be managed by removal of the offending agent (e.g., beta lactams and sulfa drugs) and with symptomatic treatment.

This patient presents with fatigue, conjunctival pallor, and normocytic anemia with appropriate reticulocytosis, which are classic findings found in SCD. Management of SCD has multiple components. Despite the normal MCV, observational studies of patients with SCD have shown a high rate of folate deficiency, which is why all patients should receive folate supplementation. Due to the high incidence of asplenia or functional hyposplenism, patients should receive additional vaccinations against S. pneumoniae and N. meningitidis as well as prophylactic penicillin until 5 years of age. Most patients should also receive hydroxyurea to increase the production of fetal hemoglobin to reduce the risk of acute crises. Of note, although this patient presents with reticulocytosis, patients with SCD may also present with a low reticulocyte count in an aplastic crisis, which is most commonly caused by infection with parvovirus B19.

Sickle cell disease usually causes a chronic, normocytic hemolytic anemia with appropriate reticulocytosis. Folate deficiency is common, and patients should receive daily folate supplementation. This patient is presenting with painful, symmetric swelling of his hands suggestive of a diagnosis of dactylitis. Dactylitis can often be the first presenting symptom of sickle cell disease (altered red blood cell morphology). Sickle cell disease occurs due to a genetic mutation that causes the production of HbSS. These abnormal red blood cells can sickle in times of stress or during dehydration. Patients can present with pain crises, acute chest syndrome, stroke, or splenic infarction. When sickling occurs in the digits, patients can experience painful, symmetric swelling of their hands known as dactylitis. This can often be the first presenting symptom in a sickle cell patient if they do not have sickle cell disease in their medical history.

Sideroblastic anemias are caused by defects in heme synthesis or mitochondrial function and can be either congenital or acquired. The classic findings are ringed sideroblasts on bone marrow aspirate and --- --- on peripheral smear.

Sideroblastic anemias are caused by defects in heme synthesis or mitochondrial function and can be either congenital or acquired. The classic findings are ringed sideroblasts on bone marrow aspirate and Pappenheimer bodies on peripheral smear. Usually from Copper def, OH abuse, INZ.

Lastly, the finding of a dilated common bile duct without evidence of stones is a very common finding in --- and is a buzzword for the condition.

Sphinc Of Oddi dysfn, rx w sphincterotomy This patient is presenting with classic symptoms of acute cholecystitis with ultrasound findings demonstrating absence of gallstones. In this setting, sphincter of Oddi dysfunction should be suspected, and ERCP with manometry should be performed to confirm the diagnosis. This patient fits the classic, "fat, female, fertile and forty" presentation suggesting acute cholecystitis. Her lab values (elevated AST, ALT, alkaline phosphatase, and bilirubin) could also support a diagnosis of a stone impacted in the common bile duct. That being said, the right upper quadrant ultrasound did not show gallstones. Her MRCP also did not reveal any abnormalities ruling out a possible diagnosis of acalculous cholecystitis. In addition, her lab values and clinical presentation remain the same 24 hours later. In light of these findings, dysfunction of the Sphincter of Oddi should be suspected. To make this diagnosis, the best test is ERCP with manometry of the sphincter of Oddi. Ultimately, a sphincterotomy will be needed once the diagnosis is confirmed.

Squamous cell carcinoma is generally treated with surgery. -- is an alternative treatment.

Squamous cell carcinoma is generally treated with surgery. Radiation is an alternative treatment.

-- -- -- presents in pediatric patients with a fever, tachycardia, a scarlatiniform rash with a positive Nikolsky sign, and mucosal sparing.

Staphylococcal scalded skin syndrome presents in pediatric patients with a fever, tachycardia, a scarlatiniform rash with a positive Nikolsky sign, and mucosal sparing.

-- -- presents with reddish plaques or nodules, neutrophilia and fever. It may be associated with an underlying malignancy.

Sweet's syndrome presents with reddish plaques or nodules, neutrophilia and fever. It may be associated with an underlying malignancy.

SOAP BRAIN MD mnemonic for SLE stands for?

Systemic lupus erythematous (SLE) is a systemic autoimmune disease that presents with characteristic multi-organ symptoms. The acronym SOAP BRAIN MD is helpful for remembering the associated findings: Serositis (i.e., pleuritis, pericarditis), Oral ulcers, Arthritis, Photosensitivity, Blood (i.e., pancytopenia), Renal (i.e., proteinuria), ANA, Immunologic (i.e., anti-dsDNA), Neurologic (i.e., psychiatric disorders, seizures, and headaches), Malar rash, and Discoid rash. Pancytopenia (i.e., anemia, leukopenia, and thrombocytopenia) is caused by immune-mediated destruction. Antibodies associated with SLE cause immune-mediated bone marrow failure and peripheral cell destruction.

pleural fluid reveals lymphocytic leukocytosis dx is?

TB

growth delay, fallen off curve obtain?

TSH

-- -- (stress cardiomyopathy) presents after an acute stressor with chest pain, dyspnea, ST-elevation on ECG, a globular heart on chest radiograph, and minor elevations in troponins.

Takotsubo cardiomyopathy (stress cardiomyopathy) presents after an acute stressor with chest pain, dyspnea, ST-elevation on ECG, a globular heart on chest radiograph, and minor elevations in troponins.

--- --- --- (compression of the tibial nerve as it passes through the tarsal tunnel) presents with pain, numbness, tingling, and aching along the plantar surface of the foot whereas -- --- presents with pain along the plantar surface of the foot that improves with rest.

Tarsal tunnel syndrome (compression of the tibial nerve as it passes through the tarsal tunnel) presents with pain, numbness, tingling, and aching along the plantar surface of the foot whereas plantar fasciitis presents with pain along the plantar surface of the foot that improves with rest.

-- -- presents with hypotension, tachycardia, unilateral breath sounds, and JVD, and the best initial step in management is needle decompression in the second intercostal space.

Tension pneumothorax presents with hypotension, tachycardia, unilateral breath sounds, and JVD, and the best initial step in management is needle decompression in the second intercostal space.

The best acute therapy for hereditary angioedema is fresh frozen plasma and ----, and the best chronic therapy is an ---.

The best acute therapy for hereditary angioedema is fresh frozen plasma and ecallantide, and the best chronic therapy is an androgen (danazol).

The best initial diagnostic test for ankylosing spondylitis is a -- --, while the most accurate diagnostic test is an--

The best initial diagnostic test for ankylosing spondylitis is a plain radiograph, while the most accurate diagnostic test is an MRI of the SI joint.

The best initial step in management for bacterial endocarditis is ---

The best initial step in management for bacterial endocarditis is blood cultures followed by broad-spectrum antibiotics (typically vancomycin and gentamicin).

The best initial step in management for congenital clubfoot (talipes equinovarus) is ----. Its where ft point inward.

The best initial step in management for congenital clubfoot (talipes equinovarus) is stretching and serial casting.

The best initial step in management of new torticollis is neck-- to rule out acquired causes including retropharyngeal abscess and atlantoaxial subluxation.

The best initial step in management of new torticollis is neck radiograph to rule out acquired causes including retropharyngeal abscess and atlantoaxial subluxation.

The best initial therapy for hereditary spherocytosis is vitamin --, and refractory cases should be managed with --.

The best initial therapy for hereditary spherocytosis is vitamin B9 (folate), and refractory cases should be managed with splenectomy.

The best medical therapy for a stable patient with atrial fibrillation with borderline hypotension is --.

The best medical therapy for a stable patient with atrial fibrillation with borderline hypotension is digoxin.

The best next step for proteinuria found incidentally in a child is to repeat the urine dipstick, ---

The best next step for proteinuria found incidentally in a child is to repeat the urine dipstick, often "splitting the urine," performing one in the morning and in the afternoon to see if it comes on toward the end of the day.

The boy in this vignette has a supracondylar humeral fracture. Due to the proximity to the brachial artery, a dreaded complication of this injury is -- of the hand, resulting in Volkmann ischemic contracture.

The boy in this vignette has a supracondylar humeral fracture. Due to the proximity to the brachial artery, a dreaded complication of this injury is ischemia of the hand, resulting in Volkmann ischemic contracture.

The boy in this vignette most likely has a neuroblastoma, which presents classically under age four. A work up would have involved a CT scan of the abdomen showing -- & --- of the ----.

The boy in this vignette most likely has a neuroblastoma, which presents classically under age four. A work up would have involved a CT scan of the abdomen showing calcifications and hemorrhages of the adrenal medulla.

The clinical presentation is consistent with --- ---, which is associated with hyperviscosity syndrome (e.g. blurring or loss of vision, headache, ataxia, dementia, stroke, or coma) and a monoclonal IgM spike on serum protein electrophoresis.

The clinical presentation is consistent with Waldenstrom's macroglobulinemia, which is associated with hyperviscosity syndrome (e.g. blurring or loss of vision, headache, ataxia, dementia, stroke, or coma) and a monoclonal IgM spike on serum protein electrophoresis. Answer 1: IgM monoclonal gammopathy of undetermined significance (MGUS) is defined by serum IgM concentration < 3.0 g/dL and the absence of anemia or other systemic symptoms. This can be a precursor state to multiple myeloma or Waldenstrom's macroglobulinemia. Answer 2: Multiple myeloma is a malignancy involving plasmacytes in the bone marrow, characterized by lytic bone lesions, formation of immunoglobulins (usually IgG and IgA), and renal failure.

The clinical presentation is consistent with Wilson's disease, which is associated with -- 24-hour urinary copper excretion and a -- serum ceruloplasmin level. there is -- Copper

The clinical presentation is consistent with Wilson's disease, which is associated with high 24-hour urinary copper excretion and a low serum ceruloplasmin level.

The diagnosis of PMR is one of exclusion as there is no definitive test or pathognomic finding for the disease. Characteristics that suggest PMR include age >50 years old, proximal and bilateral morning stiffness, elevated ESR, and response to glucocorticoids. Those with atypical symptoms are often times difficult to diagnose.

The diagnosis of PMR is one of exclusion as there is no definitive test or pathognomic finding for the disease. Characteristics that suggest PMR include age >50 years old, proximal and bilateral morning stiffness, elevated ESR, and response to glucocorticoids. Those with atypical symptoms are often times difficult to diagnose.

The finding of a "triple bubble" sign is consistent with --- ----, which presents similarly to duodenal atresia with bilious vomiting early in life

The finding of a "triple bubble" sign is consistent with jejunal atresia, which presents similarly to duodenal atresia with bilious vomiting early in life

The history, exam and laboratory testing in this vignette support a diagnosis of idiopathic central precocious puberty. This condition is treated with --- to suppress hypothalamic-pituitary axis. Precocious puberty is defined as puberty that begins before age 8 in girls and before age 9 in boys. It is estimated that precocious puberty affects between 1 in 5000 and 1 in 10,000 children and a high female to male ratio. The causes of precocious puberty are broadly subdivided according to a functional classification based on gonadotrophin dependence and gonadotrophin non-dependence. Central precocious puberty is diagnosed by pubertal levels of LH that increase with GnRH stimulation, and is termed idiopathic when no obvious pathology is responsible for the elevated central hormones (e.g. tumor).

The history, exam and laboratory testing in this vignette support a diagnosis of idiopathic central precocious puberty. This condition is treated with GnRH agonists (leuprolide) to suppress hypothalamic-pituitary axis. Precocious puberty is defined as puberty that begins before age 8 in girls and before age 9 in boys. It is estimated that precocious puberty affects between 1 in 5000 and 1 in 10,000 children and a high female to male ratio. The causes of precocious puberty are broadly subdivided according to a functional classification based on gonadotrophin dependence and gonadotrophin non-dependence. Central precocious puberty is diagnosed by pubertal levels of LH that increase with GnRH stimulation, and is termed idiopathic when no obvious pathology is responsible for the elevated central hormones (e.g. tumor).

The initial management of acute sinusitis is -----, with antibiotics given only if there is no clinical improvement during observation or temperature exceeds 38.3 C.

The initial management of acute sinusitis is to observe for 7 days, with antibiotics given only if there is no clinical improvement during observation or temperature exceeds 38.3 C.

Duke criteria (major: positive blood cultures, evidence of endocardial involvement, minor: predisposing heart condition or history of IV drug use, fever, vascular phenomena, immunologic phenomena, blood cultures not meeting major criteria, echocardiographic findings not included in major findings). The number of blood cultures that must be positive to meet this criterion depends on which microorganism is involved. The evidence of endocardial involvement can be seen on transthoracic echocardiography (TTE) or transesophageal echocardiography (TEE) as an oscillating intracardiac mass on a valve, an abscess, new partial dehiscence of a prosthetic valve, or new valvular regurgitation. Definite endocarditis is defined by the presence of 2 major criteria, 1 major and 3 minor criteria, or 5 minor criteria. Possible endocarditis is defined by the presence of 1 major and 1 minor criteria or 3 minor criteria. Definite criteria can also be established by culture or biopsy of a vegetation or intracardiac abscess. Figure A demonstrates Osler's nodes on the patient's hands. Osler's nodes are painful, erythematous lesions found on the palms and soles caused by immune complex deposition. Illustration A demonstrates Janeway lesions, one of the vascular phenomena included in the modified Duke criteria. Janeway lesions are non-tender, erythematous macules or nodular lesions found on the palms and soles. They are caused by septic emboli which form microabscesses. Illustration B demonstrates septic emboli to the lungs on chest radiograph, another finding meeting the criteria for vascular phenomena in the modified Duke criteria. Illustration C demonstrates Roth's spots. Roth's spots are retinal hemorrhages caused by immune complex-mediated vasculitis and satisfy the immunologic phenomena criteria of the modified Duke criteria.

The modified Duke criteria outline the criteria necessary to make a diagnosis of infective endocarditis. Major criteria include positive blood cultures and evidence of endocardial involvement. Minor criteria include predisposing risk factors, fever, vascular phenomena, immunologic phenomena, and microbiological evidence not meeting major criteria. Making a definite diagnosis requires satisfying 2 major criteria, 1 major and 3 minor criteria, or 5 minor criteria.

The most accurate method of dating a pregnancy in the first trimester is ---- A chalazion is the result of meibomian gland obstruction and if persistent or recurrent may be a sign of meibomian gland carcinoma or sebaceous cell carcinoma. The nodule forms on the conjunctival surface of the eyelid and is the result of granulomatous inflammation. This condition is self-limiting and does not require treatment. The differential includes a stye, which is an infection of the sebaceous glands of Zeis at the base of the eyelashes, or an infection of the apocrine sweat glands of Moll. This infection is on the external aspect of the eyelid while a chalazion is on the internal surface. EXCISE THE CHALAZION BC COULD BE SEB CC, MEIMOB GLAND CA

The most accurate method of dating a pregnancy in the first trimester is ultrasound measurement of the crown-rump length. Chalazion presents with a hard and painless lid nodule. This pathology occurs secondary to granulomatous inflammation of the Meibomian gland. The lesion can be treated with simple excision; however, recurrent lesions are concerning for Meibomian cell carcinoma. The most important differential diagnosis to consider in these patients is a hordeolum which presents with a firm and painful lid nodule (in contrast to painless).

The most accurate test for pulmonary embolism in the setting of acute kidney injury or chronic renal failure is a --- ---- ---

The most accurate test for pulmonary embolism in the setting of acute kidney injury or chronic renal failure is a ventilation perfusion scan (V/Q scan). COPY

The most common cause of cardiac arrest after a myocardial infarction is -- --

The most common cause of cardiac arrest after a myocardial infarction is ventricular fibrillation.

The most common cause of lobar hemorrhage in the elderly is ---

The most common cause of lobar hemorrhage in the elderly is deposition of β-amyloid in blood vessel walls.

The most common contraindications to breastfeeding include --

The most common contraindications to breastfeeding include HIV, active tuberculosis, open sores from herpes simplex virus, and galactosemia in the infant.

The most effective form of emergency contraception is---

The most effective form of emergency contraception is the copper IUD.

The patient has a history of depression and is presenting with decreased motor activity, waxy flexibility, lack of response to external stimuli, and mutism, suggesting the diagnosis of --. --- should initially be treated with --

The patient has a history of depression and is presenting with decreased motor activity, waxy flexibility, lack of response to external stimuli, and mutism, suggesting the diagnosis of catatonia. Catatonia should initially be treated with benzodiazepines like lorazepam.

The patient has a markedly elevated beta-hcG following pathology that confirmed a --- ---which is suspicious for gestational trophoblastic disease (GTD). GTD is associated with-- --- -- that present as bilaterally enlarged, multicystic ovaries and can cause hyperandrogenism. Theca lutein cysts are caused by hyperplasia of the theca interna cells stimulated by excessive circulating gonadotropins such as beta-hCG. Therefore, they are associated with conditions that increase gonadotropins including GTD, multifetal pregnancy, polycystic ovarian syndrome, and ovulation induction. They present as bilateral, muticystic ovaries, and patients may have signs of hyperandrogenism (i.e., hirsutism, acne and masculine appearance).

The patient has a markedly elevated beta-hcG following pathology that confirmed a hydatidiform mole, which is suspicious for gestational trophoblastic disease (GTD). GTD is associated with theca lutein cysts that present as bilaterally enlarged, multicystic ovaries and can cause hyperandrogenism. Theca lutein cysts are caused by hyperplasia of the theca interna cells stimulated by excessive circulating gonadotropins such as beta-hCG. Therefore, they are associated with conditions that increase gonadotropins including GTD, multifetal pregnancy, polycystic ovarian syndrome, and ovulation induction. They present as bilateral, muticystic ovaries, and patients may have signs of hyperandrogenism (i.e., hirsutism, acne and masculine appearance).

The patient has metastatic prostate cancer, which is usually initially treated with androgen depletion and luteinizing hormone releasing agonists (e.g. leuprolide) as first line agents. However, in patients with severe ureteral obstruction and painful vertebral metastases, an -- (e.g. --) is given prior to beginning LHRH agonist therapy.

The patient has metastatic prostate cancer, which is usually initially treated with androgen depletion and luteinizing hormone releasing agonists (e.g. leuprolide) as first line agents. However, in patients with severe ureteral obstruction and painful vertebral metastases, an antiandrogen (e.g. flutamide or bicalutamide) is given prior to beginning LHRH agonist therapy. Although early prostate cancer may be asymptomatic, late prostate cancer causes obstructive urinary symptoms and most commonly metastasizes to the bones. LHRH agonists (e.g. leuprolide) bind to LHRH receptors in the anterior pituitary and initially promote the release of LH and FSH, resulting in an undesired increase in testosterone. However, after continuous agonism, the LHRH receptors downregulate, causing a decrease in LH release and ultimately produce the desired decrease in testosterone. Patients presenting with painful metastases and ureteral obstruction should receive a direct anti-androgen first.

In a trauma patient with asymmetric upper extremity blood pressure and pulses and a widen superior mediastinum on chest x-ray, suspect aortic dissection. dx?

The patient in the above vignette most likely has a traumatic thoracic aortic dissection. In a trauma patient with asymmetric upper extremity blood pressure and pulses and a widen superior mediastinum on chest x-ray, suspect aortic dissection.

The patient in this vignette is most likely suffering from anorexia nervosa (AN). Patients with body weight less than --% of average should be hospitalized.

The patient in this vignette is most likely suffering from anorexia nervosa (AN). Patients with body weight less than 75% of average should be hospitalized.

The patient in this vignette is suffering from an asthma exacerbation. The most frequent allergen that causes allergic asthma exacerbations is the -- -- --

The patient in this vignette is suffering from an asthma exacerbation. The most frequent allergen that causes allergic asthma exacerbations is the house dust mite.

The patient in this vignette most likely suffers from acalculous cholecystitis (ACC). After the diagnosis is made with a right upper quadrant ultrasound, emergent -- -- should be performed.

The patient in this vignette most likely suffers from acalculous cholecystitis (ACC). After the diagnosis is made with a right upper quadrant ultrasound, emergent percutaneous cholecystostomy should be performed.

The patient in this vignette presents with a painful lesion in the proximal tibia with pain that is worse in the evening and resolves with a non-steroidal anti-inflammatory drug (NSAID). This presentation is suggestive of -- --

The patient in this vignette presents with a painful lesion in the proximal tibia with pain that is worse in the evening and resolves with a non-steroidal anti-inflammatory drug (NSAID). This presentation is suggestive of osteoid osteoma.

The patient is presenting with continuous, painless urinary leakage and pooling of fluid in the vagina following bilateral tubal ligation, which is consistent with a -- --. -- --nstilled into the bladder can confirm the diagnosis.

The patient is presenting with continuous, painless urinary leakage and pooling of fluid in the vagina following bilateral tubal ligation, which is consistent with a vesicovaginal fistula. Methylene blue instilled into the bladder can confirm the diagnosis.

The patient is presenting with deafness and long QT syndrome that progressed to torsades de pointes, suggesting the diagnosis of Jervell and Lange-Nielsen syndrome. The next best step in management is to --

The patient is presenting with deafness and long QT syndrome that progressed to torsades de pointes, suggesting the diagnosis of Jervell and Lange-Nielsen syndrome. The next best step in management is to replete potassium and magnesium and then start a beta-blocker.

hypopigmented lesions occurring in summer that suggest -- -- summer and back.. think

The patient is presenting with hypopigmented lesions occurring in summer that suggest tinea versicolor. Tinea versicolor can be caused by Malassezia furfur or globosa. This patient is presenting with lesions suggestive of tinea versicolor, which is diagnosed with a KOH prep demonstrating mycelium strands and numerous spores. Tinea versicolor occurs secondary to a Malassezia globosa/furfur infection in patients who live in hot and humid weather. Symptoms include hypopigmented or hyperpigmented lesions with minimal other symptoms (though may be itchy) that typically occur on the trunk, an area of perspiration. The diagnostic test of choice is a KOH prep demonstrating mycelium strands and numerous spores ("spaghetti and meatballs"). they are hypo pigmented, white looking.

The patient is presenting with multiple, asymptomatic, hypopigmented lesions on the trunk and a family history significant for bilateral deafness, which suggests the diagnosis of ----Patients with NFII characteristically develop --- --- ---

The patient is presenting with multiple, asymptomatic, hypopigmented lesions on the trunk and a family history significant for bilateral deafness, which suggests the diagnosis of neurofibromatosis type II (NFII). Patients with NFII characteristically develop bilateral vestibular schwannomas.

The patient is presenting with nausea, early satiety, occasional vomiting, postprandial fullness and bloating with a history of long-term uncontrolled diabetes, suggesting the diagnosis of diabetic gastroparesis. --- is an appropriate pharmacologic treatment.

The patient is presenting with nausea, early satiety, occasional vomiting, postprandial fullness and bloating with a history of long-term uncontrolled diabetes, suggesting the diagnosis of diabetic gastroparesis. Erythromycin is an appropriate pharmacologic treatment.

The patient is presenting with recurrent spontaneous abortions, a history of thrombosis, and a prolonged aPTT, which suggests the diagnosis of antiphospholipid syndrome (APLS). APLS is associated with a false-positive VDRL. A false-positive VDRL/RPR can be associated with APLS, as the anticardiolipin antibodies and lupus anticoagulant can cause positive reactions in confirmatory tests for syphilis serology. For patients who are found to have antiphospholipid antibodies, but are asymptomatic, treatment is not required. These patients should still, however, minimize other factors that may increase clotting risk, such as smoking and oral contraceptives. Those patients who have had a thrombotic episode must be on indefinite anticoagulation (warfarin). Women with APLS can have successful pregnancies, despite the increased risk for miscarriage.

The patient is presenting with recurrent spontaneous abortions, a history of thrombosis, and a prolonged aPTT, which suggests the diagnosis of antiphospholipid syndrome (APLS). APLS is associated with a false-positive VDRL. Lupus anticoagulant will cause prolongation of PTT but lends itself to clotting, not bleeding. Lupus anticoagulant is a subtype of anti-phospholipid syndromes, which also include anti-cardiolipin antibody. Lupus anticoagulant is an immunoglobulin that binds to phospholipids and proteins associated with the cell membrane. This binding interferes with phospholipids that induce in vitro coagulation during the PTT laboratory test. The term lupus anticoagulant is actually a misnomer since the protein is associated with a thrombophilic state and can present with recurrent arterial or venous thromboses or spontaneous abortions in women. PT, bleeding time, platelet count, and D-dimer are normal, while PTT is prolonged. PT tests the function of factors I, II, V, VII, and X (the extrinsic pathway), while PTT tests the function of all factors except VII and XIII (the intrinsic pathway).

The patient is presenting with vaginal bleeding and a soft, enlarged uterus suspicious for -- --, which should first be managed with -- & -- Uterine atony is the most common cause of postpartum hemorrhage. Risk factors for postpartum hemorrhage include prolonged or induced labor, multiple gestation, polyhydramnios, operative delivery, grand multiparity, macrosomic infant, and chorioaminionitis. It occurs when the uterus fails to contract after the placenta separates. The uterus on examination is distended and soft. The first step in management is bimanual uterine massage and oxytocin, which both aim to increase uterine contraction.

The patient is presenting with vaginal bleeding and a soft, enlarged uterus suspicious for uterine atony, which should first be managed with uterine massage and oxytocin. Uterine atony is the most common cause of postpartum hemorrhage. Risk factors for postpartum hemorrhage include prolonged or induced labor, multiple gestation, polyhydramnios, operative delivery, grand multiparity, macrosomic infant, and chorioaminionitis. It occurs when the uterus fails to contract after the placenta separates. The uterus on examination is distended and soft. The first step in management is bimanual uterine massage and oxytocin, which both aim to increase uterine contraction.

The 8 yo patient presents with -- -- --, idiopathic avascular necrosis of the femoral capital epiphysis.

The patient presents with Legg-Calve-Perthes disease, idiopathic avascular necrosis of the femoral capital epiphysis. This patient's presentation is suggestive of transient synovitis of the hip, the most common cause of hip pain in childhood. A hip radiograph is indicated to rule out Legg-Calve-Perthes disease. Transient synovitis of the hip is most commonly seen in males between the ages of 5-10. The etiology of this condition is unknown; suggested causes include an autoimmune reaction after a viral infection, allergic reaction, or trauma. It is a diagnosis of exclusion and other more serious conditions must be ruled out. A normal WBC count and ESR help differentiate transient synovitis from septic arthritis. A radiograph will appear normal in transient synovitis of the hip; however, in Legg-Calve-Perthes disease--an avascular necrosis of the femoral head--X-ray demonstrates a flattened or fragmented femoral head. Typical treatment is rest and NSAID therapy, which often improves the symptoms associated with transient synovitis of the hip.

The patient presents with a firm, smooth, hyperpigmented nodule that arose from a site of trauma, which is consistent with a -- COPY

The patient presents with a firm, smooth, hyperpigmented nodule that arose from a site of trauma, which is consistent with a dermatofibroma. COPY

The patient presents with arthritis, peripheral edema, history of diabetes, and hyperpigmentation, which suggests the diagnosis of --.

The patient presents with arthritis, peripheral edema, history of diabetes, and hyperpigmentation, which suggests the diagnosis of hemochromatosis. Hemochromatosis has an increased risk of hepatocellular carcinoma, which is associated with alpha fetoprotein (AFP).

The patient presents with non-blanching, violaceous papules/plaques on his bilateral lower extremities that show neutrophils, fibrinogen, and immune complexes (i.e., C3, IgG, and IgM) on skin biopsy. This is consistent with --- ---.

The patient presents with non-blanching, violaceous papules/plaques on his bilateral lower extremities that show neutrophils, fibrinogen, and immune complexes (i.e., C3, IgG, and IgM) on skin biopsy. This is consistent with leukocytoclastic vasculitis. The patient presents with non-blanching, violaceous papules/plaques on his bilateral lower extremities that show neutrophils, fibrinogen, and immune complexes (i.e., C3, IgG, and IgM) on skin biopsy. This is consistent with leukocytoclastic vasculitis. HSP has IgA deposition.

The patient presents with scleral icterus, jaundice, and normal liver function tests except for a direct hyperbilirubinemia. If on pathology the liver appeared normal, this presentation would be most consistent with -- --

The patient presents with scleral icterus, jaundice, and normal liver function tests except for a direct hyperbilirubinemia. If on pathology the liver appeared normal, this presentation would be most consistent with Rotor syndrome. Rotor syndrome is an autosomal recessive disorder that causes congenital hyperbilirubinemia. Patients are usually asymptomatic, but medications that are toxic to the liver as well as oral contraceptives can induce hyperbilirubinemia leading to jaundice and scleral icterus. Rotor syndrome is caused by a defect in the storage capacity of conjugated (direct) bilirubin, which results in a mild to moderate direct hyperbilirubinemia with normal liver function tests. The condition is benign with no need for liver biopsy, as the liver would appear normal on pathology.

The patient suffers from symptoms of opioid withdrawal. --- is an opioid agent used to stabilize patients suffering from withdrawal.

The patient suffers from symptoms of opioid withdrawal. Methadone is an opioid agent used to stabilize patients suffering from withdrawal. Individuals who abruptly withdraw from opioid use may see symptoms begin within 24 hours. Flu-like symptoms may be present, including nausea, diarrhea and stomach cramps, and patients often complain of joint and muscle aches. Though symptoms may be severe, they are not life-threatening. First-line stabilization agent is methadone.

The patient's non-tender, indurated mandibular mass with yellowish drainage following a dental procedure is suggestive of --- which is best treated with high-dose ---.

The patient's non-tender, indurated mandibular mass with yellowish drainage following a dental procedure is suggestive of cervicofacial actinomycosis, which is best treated with high-dose penicillin.

The patient's presentation of post-infectious painful enlargement of the thyroid and hyperthyroidism are consistent with ----

The patient's presentation of post-infectious painful enlargement of the thyroid and hyperthyroidism are consistent with subacute granulomatous thyroiditis, also called de Quervain's thyroiditis. riedel is hard and painless.

The polysaccharide pneumococcal vaccine induces a -- only response to produce moderate affinity antibodies.

The polysaccharide pneumococcal vaccine induces a B-cell only response to produce moderate affinity antibodies.

The single best indication for placement of an ICD in an adult is an ejection fraction less than--

The single best indication for placement of an ICD in an adult is an ejection fraction less than 35%.

The synovial aspirate from this patient with a recurrence of gouty arthritis is most likely to yield cloudy yellow fluid with--- to --- WBC (--% neutrophils), needle-shaped negatively birefringent crystals, and no bacteria, unless the joint is superinfected.

The synovial aspirate from this patient with a recurrence of gouty arthritis is most likely to yield cloudy yellow fluid with 2000-50000 WBC (70% neutrophils), needle-shaped negatively birefringent crystals, and no bacteria, unless the joint is superinfected.

The treatment of a stress fracture is --

The treatment of a stress fracture is casting and reduced weight bearing.

The treatment of choice for a first episode of uncomplicated diverticulitis is bowel rest, antibiotics (--), and a liquid diet.

The treatment of choice for a first episode of uncomplicated diverticulitis is bowel rest, antibiotics (trimethoprim-sulfamethoxazole), and a liquid diet.

The treatment of choice for amyotrophic lateral sclerosis, which presents with both upper and lower motor neuron signs, is --.

The treatment of choice for amyotrophic lateral sclerosis, which presents with both upper and lower motor neuron signs, is riluzole.

The treatment of choice in pregnancy for Graves' disease that is refractory to propranolol and propylthiouracil is ----. The two most common causes of hyperthyroidism in pregnancy are Graves' disease and hCG-mediated hyperthyroidism, which is a mild hyperthyroidism with suppressed TSH and elevated free T4 caused by peaking levels of hCG at the end of the first trimester.

The treatment of choice in pregnancy for Graves' disease that is refractory to propranolol and propylthiouracil is thyroidectomy. The two most common causes of hyperthyroidism in pregnancy are Graves' disease and hCG-mediated hyperthyroidism, which is a mild hyperthyroidism with suppressed TSH and elevated free T4 caused by peaking levels of hCG at the end of the first trimester. No RAI in preg!

Asymptomatic bacteriuria usually resolves spontaneously in elderly patients without antibiotic therapy.

These patients DO require treatment for asymptomatic bacteriuria: - Pregnant - Urologic intervention - Hip arthroplasty

This 2-month-old patient presents with jaundice, dark urine, mild transaminitis, and conjugated hyperbilirubinemia, which suggests the diagnosis of -- ---. there is obs of --- ---

This 2-month-old patient presents with jaundice, dark urine, mild transaminitis, and conjugated hyperbilirubinemia, which suggests the diagnosis of biliary atresia. Biliary atresia is characterized by obstruction of the extrahepatic biliary tree. Biliary atresia is a progressive, obliterative disease of the extrahepatic biliary tree. It presents in the first 8 weeks of life with jaundice, acholic stools, dark urine, hepatomegaly, mild transaminitis, and conjugated hyperbilirubinemia. The definitive diagnosis is made by cholangiogram, and treatment usually involves hepatoportoenterostomy (Kasai procedure), in which the duodenum is directly anastomosed to the liver. Most patients also eventually require liver transplant.

This 3-day old infant is experiencing jaundice due to insufficient nutritional intake, known as -- -- -- This type of indirect hyperbilirubinemia occurs when inadequate intake leads to infrequent bowel movements, leading to decreased excretion of bilirubin from the body.

This 3-day old infant is experiencing jaundice due to insufficient nutritional intake, known as "breastfeeding failure jaundice". This type of indirect hyperbilirubinemia occurs when inadequate intake leads to infrequent bowel movements, leading to decreased excretion of bilirubin from the body. Neonates are physiologically predisposed to develop jaundice. Breastfeeding failure jaundice presents in the first week of life and is caused by inadequate nutritional intake, which leads to dehydration, an insufficient number of bowel movements, and decreased excretion of bilirubin. Neonates are expected to lose 5-7% of their birth weight within the first week of life, but a greater than 10% weight loss is abnormal. The treatment is increased dietary intake. In contrast, breast milk jaundice begins after days 4-7 of life. The cause of breast milk jaundice is not completely understood, but it is thought to be related to high levels of B-glucuronidase and lipase within breast milk. No specific intervention is needed for breast milk jaundice, and continued breast milk feeding is recommended.

hemophilia pts joints have

This child has hemophilia A. Patients with hemophilia A and B are at risk of recurrent hemarthrosis, with subsequent hemophilic arthropathy, marked by joints with iron deposits and synovial thickening with fibrosis.

This child is presenting with acute abdominal distress and currant jelly stools The finding of a "target sign" on ultrasonography is highly sensitive and specific for -- , therefore a -- --- is very effective at diagnosing and reducing intussuscepted sections of bowel. Intussusception occurs when a proximal loop of bowel telescopes and invaginates into a distal segment, which results in compression of the proximal bowel's mesentery and subsequent vascular compromise. Intussusception often occurs when lead points, or segments of bowel which protrude into the lumen of the bowel, advance into adjacent segments. In children, lead points can result from mesenteric adenitis, hypertrophied Peyer's patches, or Meckel's diverticulae. Patients will present with colicky abdominal pain, and investigation with ultrasonography can demonstrate the two segments of bowel invaginated in one another (Illustration A and B)

This child is presenting with acute abdominal distress and currant jelly stools The finding of a "target sign" on ultrasonography is highly sensitive and specific for intussusception, therefore a pneumatic enema is very effective at diagnosing and reducing intussuscepted sections of bowel. Intussusception occurs when a proximal loop of bowel telescopes and invaginates into a distal segment, which results in compression of the proximal bowel's mesentery and subsequent vascular compromise. Intussusception often occurs when lead points, or segments of bowel which protrude into the lumen of the bowel, advance into adjacent segments. In children, lead points can result from mesenteric adenitis, hypertrophied Peyer's patches, or Meckel's diverticulae. Patients will present with colicky abdominal pain, and investigation with ultrasonography can demonstrate the two segments of bowel invaginated in one another (Illustration A and B)

This child most likely has -- ---, as evidenced by poor language development, and inability to hear directions or his name when called.

This child most likely has hearing impairment, as evidenced by poor language development, and inability to hear directions or his name when called.

acute gallstone pancreatitis. According to Ranson's criteria, an initial white blood cell count greater than-- /uL is associated with increased mortality.

This clinical vignette is consistent with acute gallstone pancreatitis. According to Ranson's criteria, an initial white blood cell count greater than 18,000/uL is associated with increased mortality.

This elderly patient with a recent fall while on anticoagulation is presenting with a gradual onset of headache and confusion, which is suspicious for a -- hematoma.

This elderly patient with a recent fall while on anticoagulation is presenting with a gradual onset of headache and confusion, which is suspicious for a subdural hematoma.

This infant has failed conservative management of congenital talipes equinovarus, also known as clubfoot. The next step is surgery, ideally performed between -- & --- months of age though some guidelines state that waiting until 12 months of age can lead to better outcomes. COPY

This infant has failed conservative management of congenital talipes equinovarus, also known as clubfoot. The next step is surgery, ideally performed between 3 and 6 months of age though some guidelines state that waiting until 12 months of age can lead to better outcomes. COPY

This infant is ruddy and polycythemic, likely secondary to maternal -- during pregnancy. COPY

This infant is ruddy and polycythemic, likely secondary to maternal hyperglycemia during pregnancy. COPY

This infant presents with vomiting, bloody stools, and poor weight gain, which suggests a diagnosis of -- --. The best next step in management is modification of the mother's diet to eliminate cow's milk and soy.

This infant presents with vomiting, bloody stools, and poor weight gain, which suggests a diagnosis of milk-protein allergy. The best next step in management is modification of the mother's diet to eliminate cow's milk and soy.

This infant with a history of eczema, fever, irritability, malaise, and a disseminated monomorphic vesicular rash most likely has -- --, for which the treatment is IV --.

This infant with a history of eczema, fever, irritability, malaise, and a disseminated monomorphic vesicular rash most likely has eczema herpeticum, for which the treatment is IV acyclovir.

This infant's clinical picture is consistent with a diagnosis of neonatal jaundice. A rising serum total bilirubin at a rate of greater than -- mg/dL/24hr is indicative of pathologic jaundice and warrants further investigation.

This infant's clinical picture is consistent with a diagnosis of neonatal jaundice. A rising serum total bilirubin at a rate of greater than 5 mg/dL/24hr is indicative of pathologic jaundice and warrants further investigation.

This newborn's seizures, cardiorespiratory failure, abnormal neurological exam, and bulging fontanelle, in the setting of prematurity, is concerning for an -- --. -- -- the best initial step in management.

This newborn's seizures, cardiorespiratory failure, abnormal neurological exam, and bulging fontanelle, in the setting of prematurity, is concerning for an intraventricular hemorrhage (IVH). Cranial ultrasonography is the best initial step in management.

This obese, middle-aged female presenting with localized pain over the anteromedial portion of the proximal tibia is most likely suffering from --- ---- ---

This obese, middle-aged female presenting with localized pain over the anteromedial portion of the proximal tibia is most likely suffering from pes anserine bursitis. Pes anserine bursitis is most commonly seen in overweight, middle-aged females, diabetics, patients with concomitant knee arthritis, and young, active individuals. The pes anserine bursa lies superficial to the superficial MCL and deep to the pes anserinus. Overuse causes excessive friction between the bursa and the overlying pes anserinus, leading to inflammation. Management should include rest, NSAIDs, stretching, and physical therapy. A corticosteroid injection may be indicated for refractory cases.

This patient experienced amaurosis fugax, a painless loss of vision in one eye. The most common cause of amaurosis fugax is a cholesterol plaque emboli from a --- artery plaque.

This patient experienced amaurosis fugax, a painless loss of vision in one eye. The most common cause of amaurosis fugax is a cholesterol plaque emboli from a carotid artery plaque.

S. aureus pneumonia is a common complication of -- pneumonia.

This patient has Staphylococcus aureus pneumonia. S. aureus pneumonia is a common complication of influenza pneumonia. aspiration is anaerobes

This patient has a rash consistent with candidal intertrigo. The best next step in management is ---

This patient has a rash consistent with candidal intertrigo. The best next step in management is topical nystatin powder.

This patient has adult polycystic kidney disease (ADPKD). The most common extra-renal complication of ADPKD is -- --

This patient has adult polycystic kidney disease (ADPKD). The most common extra-renal complication of ADPKD is hepatic cysts, which are seen in the ultrasound imaging in Figure D.

homocysteine--b12-->methionine so in b12 def, inc homocysteine inc risk of plac abruption.

This patient has hyperhomocysteinemia in the setting of megaloblastic anemia, likely caused by a nutritional deficiency of folate or vitamin B12. Homocysteine is a product of the catabolism of methionine, which then can be converted back to methionine with the aid of B12 and folate. Hyperhomocysteinemia is a risk factor for placental abruption.

This patient initially presents with classic symptoms of essential tremor and then returns after treatment with abdominal pain, dark urine, and proximal muscle weakness, which suggest a diagnosis of -- --- --. Acute intermittent porphyria is known to be triggered by primidone. An essential tremor is usually a bilateral hand tremor, as in this patient, but could also present as an isolated head tremor. The tremor classically worsens with action, particularly fine motor movements, and is relieved by alcohol. The first-line treatment for essential tremor is propranolol, but its use was contraindicated in this patient in light of her bradycardia. Alternatives include primidone and topiramate, but only primidone is known to precipitate an episode of acute intermittent porphyria (AIP) via it's metabolite phenobarbital.

This patient initially presents with classic symptoms of essential tremor and then returns after treatment with abdominal pain, dark urine, and proximal muscle weakness, which suggest a diagnosis of acute intermittent porphyria. Acute intermittent porphyria is known to be triggered by primidone. An essential tremor is usually a bilateral hand tremor, as in this patient, but could also present as an isolated head tremor. The tremor classically worsens with action, particularly fine motor movements, and is relieved by alcohol. The first-line treatment for essential tremor is propranolol, but its use was contraindicated in this patient in light of her bradycardia. Alternatives include primidone and topiramate, but only primidone is known to precipitate an episode of acute intermittent porphyria (AIP) via it's metabolite phenobarbital.

This patient is a young male presenting with back pain that improves upon activity and decreased mobility suggesting a diagnosis of -- --. The best initial step in management of AS is -- ---to look for sacroiliitis. Ankylosing spondylitis classically presents in a young male who complains of back pain that improves with activity. Patients may experience other complications including uveitis, aortitis, and restrictive lung disease. The best initial step in management is a radiograph of the lumbosacral spine to look for signs of sacroiliitis which is typically the first radiographical finding. The most accurate test is an MRI of the sacroiliac joint.

This patient is a young male presenting with back pain that improves upon activity and decreased mobility suggesting a diagnosis of ankylosing spondylitis (AS). The best initial step in management of AS is radiography of the lumbosacral spine to look for sacroiliitis. Ankylosing spondylitis classically presents in a young male who complains of back pain that improves with activity. Patients may experience other complications including uveitis, aortitis, and restrictive lung disease. The best initial step in management is a radiograph of the lumbosacral spine to look for signs of sacroiliitis which is typically the first radiographical finding. The most accurate test is an MRI of the sacroiliac joint.

This patient is an elderly man with restless legs, unintentional weight loss, and pale sclerae on exam, most likely due to i---) resulting in restless--- .-- cancer is high on the differential and should be ruled out with a colonoscopy.

This patient is an elderly man with restless legs, unintentional weight loss, and pale sclerae on exam, most likely due to iron deficiency anemia (IDA) resulting in restless leg syndrome (RLS). Colorectal cancer is high on the differential and should be ruled out with a colonoscopy.

--- --- --- typically presents in obese female patients and is associated with trauma to the area. It can also be found in patients who suffer from osteoarthritis. Pain and inflammation are usually localized over the pes anserine bursa which is inferior and medial to the patella over the tibia. Treatment is typically rest and NSAIDs as needed for pain management.

This patient is obese and is presenting after athletic activities with pain over the pes anserine bursa suggesting a diagnosis of pes anserine bursitis. Pes anserine bursitis typically presents in obese female patients and is associated with trauma to the area. It can also be found in patients who suffer from osteoarthritis. Pain and inflammation are usually localized over the pes anserine bursa which is inferior and medial to the patella over the tibia. Treatment is typically rest and NSAIDs as needed for pain management.

This patient is presenting after a motor vehicle accident with poor oxygen saturation several hours after the incident, which suggests a diagnosis of a -- -- . Most self-resolve, and supportive therapy is all that is needed.

This patient is presenting after a motor vehicle accident with poor oxygen saturation several hours after the incident, which suggests a diagnosis of a pulmonary contusion. Most pulmonary contusions self-resolve, and supportive therapy is all that is needed.

Patients with Lynch syndrome should be screened for the development of CRC with colonoscopy every 1-2 years beginning at age 25.

This patient is presenting with a cough, fatigue, fever, erythema multiforme (which is associated with Mycoplasma infections), and a chest radiograph demonstrating interstitial infiltrates suggesting a diagnosis of atypical pneumonia most likely caused by Mycoplasma pneumoniae. The treatment for an atypical pneumonia is azithromycin.

decreased MCV, increased iron, decreased TIBC, increased ferritin, and increased reticulocyte count suggesting a diagnosis of beta-thalassemia. inc Fe ferritin and retic= beta thal

This patient is presenting with a decreased MCV, increased iron, decreased TIBC, increased ferritin, and increased reticulocyte count suggesting a diagnosis of beta-thalassemia. Beta-thalassemia typically presents with anemia and pallor in a patient of Mediterranean descent. Laboratory values will demonstrate a profoundly decreased MCV in contrast to other common causes of microcytic anemia (like iron deficiency) which is typically more mild. An increased iron (in particular, if transfusions have been administered) can also be seen, ruling out a diagnosis such as iron deficiency. Other unique findings include a decreased TIBC (secondary to the increased iron level), increased ferritin, increased reticulocyte count (to compensate for the anemia), and increased transferrin saturation. Treatment includes splenectomy, folate supplementation, and transfusions. Symptoms improve with transfusions; however, iron overload is a common problem in these patients.

This child is presenting with a fever and a painful scarlatiniform rash with flaccid blisters suggesting a diagnosis of staphylococcal scalded skin syndrome. Staphylococcal scalded skin syndrome (SSSS) typically presents in pediatric patients with a prodrome of fever, irritability, and skin tenderness. Scarlatiniform (scarlet fever-like) erythema can subsequently be seen which is a painful rash seen in SSSS along with flaccid blisters with a positive Nikolsky sign. Flexural accentuation and perioral crusting are other unique features of this pathology. Mucosal surface sparing (in contrast to Steven-Johnson syndrome and toxic epidermal necrolysis) can be seen in this condition. The pathophysiology is related to the toxin produced which cleaves desmoglein-1. The treatment of SSSS is to administer anti-staphylococcal antibiotics, treat any subsequent infections that arise, and fluid resuscitate the patient given the significant fluid loss in this condition.

This patient is presenting with a fever and a painful scarlatiniform rash with flaccid blisters suggesting a diagnosis of staphylococcal scalded skin syndrome.

This patient is presenting with a fever, headache, and altered mental status with an absence of meningeal findings suggesting a diagnosis of -- -- -- which should promptly be treated with --.

This patient is presenting with a fever, headache, and altered mental status with an absence of meningeal findings suggesting a diagnosis of herpes simplex encephalitis which should promptly be treated with acyclovir.

This patient is presenting with a history of travel, a cough, fever, hilar adenopathy, erythema nodosum, and symptoms that worsen with administration of steroids suggesting a diagnosis of --. The treatment of --- is --- .

This patient is presenting with a history of travel, a cough, fever, hilar adenopathy, erythema nodosum, and symptoms that worsen with administration of steroids suggesting a diagnosis of histoplasmosis. The treatment of histoplasmosis is itraconazole.

Dubin-Johnson syndrome presents with asymptomatic, non-pruritic jaundice, elevated bilirubin levels (with conjugated bilirubin representing > 50% of total bilirubin), and is associated with a hyperpigmented liver. happens for no reason

This patient is presenting with a likely history of chronic obstructive pulmonary disease (COPD) in the setting of decreased breath sounds suggesting a diagnosis of a spontaneous pneumothorax, caused by rupture of an emphysematous bleb. COPD typically occurs in patients with a significant smoking history. Regular exposure to smoke can damage alveoli and degrade the elastic tissue. This causes dilated/distended alveoli which can occasionally burst. This leads to a spontaneous pneumothorax, which presents with severe and sudden shortness of breath. Chest radiography can confirm the diagnosis and placement of a chest tube is the treatment of choice. Small spontaneous pneumothoraces can sometimes be managed conservatively with supportive therapy and will heal on their own.

macrocytic anemia, normal folate b12, dx?

This patient is presenting with a macrocytic anemia in the setting of a normal B12 level and a Pelger-Huet cell seen on peripheral blood smear, suggesting a diagnosis of myelodysplasia. Myelodysplasia presents with a macrocytic anemia, low reticulocyte count, and pancytopenia. The Pelger-Huet cell (a hyposegmented or bilobed neutrophil) is a specific finding for this diagnosis. The differential diagnosis should include vitamin B12 deficiency which can be ruled out with a normal vitamin B12 level. The treatment is transfusions as needed and in some cases chemotherapeutic drugs (lenalidomide and azacitidine).

This patient is presenting with a normocytic anemia, weight loss, and hypercalcemia suggesting an underlying -- as the etiology of his presentation.

This patient is presenting with a normocytic anemia, weight loss, and hypercalcemia suggesting an underlying malignancy as the etiology of his presentation.

This patient is presenting with a period of hyperthyroidism followed by hypothyroidism following an infection suggesting a diagnosis of --- ---. No management is needed for this condition - the patient should be discontinued on any anti-thyroid medications that may have been incorrectly started. Silent thyroiditis occurs after a viral infection and results in autoimmune destruction of the thyroid. This causes a transient hyperthyroid state which can present with sweating, weight loss, and anxiety. It can be mistaken for hyperthyroidism and incorrectly treated. After this initial hyperthyroid state, a short hypothyroid period ensues (with symptoms of weight gain and fatigue) which self-resolves. No treatment is needed as this condition self-resolves.

This patient is presenting with a period of hyperthyroidism followed by hypothyroidism following an infection suggesting a diagnosis of silent thyroiditis. No management is needed for this condition - the patient should be discontinued on any anti-thyroid medications that may have been incorrectly started. Silent thyroiditis occurs after a viral infection and results in autoimmune destruction of the thyroid. This causes a transient hyperthyroid state which can present with sweating, weight loss, and anxiety. It can be mistaken for hyperthyroidism and incorrectly treated. After this initial hyperthyroid state, a short hypothyroid period ensues (with symptoms of weight gain and fatigue) which self-resolves. No treatment is needed as this condition self-resolves.

This patient is presenting with a rapid onset of multiple seborrheic keratoses (Leser-Trélat sign) which is associated with -- -- ---(supraclavicular lymph node enlargement).

This patient is presenting with a rapid onset of multiple seborrheic keratoses (Leser-Trélat sign) which is associated with occult visceral malignancy (supraclavicular lymph node enlargement).

This patient is presenting with a scaly patch of alopecia and post-auricular lymphadenopathy, suggesting the diagnosis of -- --. The most accurate test is a fungal culture in --- ---

This patient is presenting with a scaly patch of alopecia and post-auricular lymphadenopathy, suggesting the diagnosis of tinea capitis. The most accurate test is a fungal culture in Sabouraud liquid medium. Tinea capitis is a dermatophyte infection of the scalp, which classically presents as a dry scaly patchy that develops into alopecia. While the best initial diagnostic test is a KOH preparation, the most accurate test is a fungal culture. Treatment includes oral terbinafine, itraconazole, or griseofulvin.

This patient is presenting with a sudden onset severe headache and physical exam findings suggestive of cerebellar dysfunction suggesting a diagnosis of -- --

This patient is presenting with a sudden onset severe headache and physical exam findings suggestive of cerebellar dysfunction suggesting a diagnosis of cerebellar hemorrhage.

This patient is presenting with a treated exacerbation of congestive heart failure (CHF) with hypokalemia, hypercalcemia, increased fasting blood glucose, and a metabolic alkalosis. This is likely the result of ---

This patient is presenting with a treated exacerbation of congestive heart failure (CHF) with hypokalemia, hypercalcemia, increased fasting blood glucose, and a metabolic alkalosis. This is likely the result of thiazide diuretic use (hydrochlorothiazide).

This patient is presenting with abdominal pain, diarrhea, guaiac postive stools, and oral ulcers suggestive of -- disease. Crohn disease is associated with --- AB

This patient is presenting with abdominal pain, diarrhea, guaiac postive stools, and oral ulcers suggestive of Crohn disease. Crohn disease is associated with ASCA (anti-Saccharomyces cerevisiae antibody) positivity.

This patient is presenting with an initial diagnosis of acute respiratory distress syndrome followed by sudden respiratory decompensation with increasing ventilator support (in particular an increased positive end-expiratory pressure (PEEP)) suggesting a diagnosis of a --.

This patient is presenting with an initial diagnosis of acute respiratory distress syndrome followed by sudden respiratory decompensation with increasing ventilator support (in particular an increased positive end-expiratory pressure (PEEP)) suggesting a diagnosis of a pneumothorax. Intubation is indicated whenever a patient is unable to protect their airway, has impending airway loss, or fails to oxygenate or ventilate on their own. Patients who are intubated and mechanically ventilated should have their respiratory function optimized using the settings on the ventilator. The fraction of inspired oxygen should be kept as low as possible such that the patient is saturating well (with a goal of 95% or greater). In addition, the tidal volume should be optimized to not over-distend the lung, which could cause barotrauma; however, low tidal volumes may predispose the patient to poor ventilation or oxygenation. Another important parameter to titrate on the ventilator is the PEEP. The PEEP should be optimized such that it is not too low, which can predispose the patient to atelectrauma and not too high which can predispose the patient to barotrauma. Patients with high tidal volumes or PEEP levels are at risk of increased pressure within the lung, which can predispose the patient to a pneumothorax. A pneumothorax will present with sudden respiratory decompensation in an intubated patient with no other clear explanation for the change in respiratory status. If the patient demonstrates tension physiology (such as tracheal deviation, jugular venous distension, and hypotension) then immediate needle decompression and chest tube placement are necessary. On the other hand, a spontaneous pneumothorax can be managed with placement of a chest tube. Auscultation of the lungs can support the diagnosis; however, normal lung sounds are often heard in a pneumothorax; thus, it is not a sufficiently sensitive finding. Radiography of the chest can support the diagnosis of a pneumothorax and will demonstrate a collapsed lung with an absence of pulmonary markings traveling to the periphery of the chest wall.

This patient is presenting with anemia and a triphalangeal thumb which is highly suspicious for -- --- ---(associated with an elevated adenosine deaminase level in red blood cells).

This patient is presenting with anemia and a triphalangeal thumb which is highly suspicious for Diamond-Blackfan anemia (associated with an elevated adenosine deaminase level in red blood cells).

This patient is presenting with bleeding from his IV sites and bruises in the setting of liver failure suggesting a diagnosis of --- --- ---. The best treatment for this condition is -- --- --.

This patient is presenting with bleeding from his IV sites and bruises in the setting of liver failure suggesting a diagnosis of depleted clotting factors. The best treatment for this condition is fresh frozen plasma.

chest pain and an EKG suggestive of atrial fibrillation. In the setting of her weight loss and other symptoms of an increased metabolism (such as sweating and anxiety) her atrial fibrillation is likely associated with -- (a -- TSH).

This patient is presenting with chest pain and an EKG suggestive of atrial fibrillation. In the setting of her weight loss and other symptoms of an increased metabolism (such as sweating and anxiety) her atrial fibrillation is likely associated with hyperthyroidism (a decreased TSH).

This patient is presenting with chest pain and borderline elevated cardiac troponins which could suggest a diagnosis of a non-ST elevation myocardial infarction. When given dipyridamole, his chest pain recurs suggesting that -- -- is occurring.

This patient is presenting with chest pain and borderline elevated cardiac troponins which could suggest a diagnosis of a non-ST elevation myocardial infarction. When given dipyridamole, his chest pain recurs suggesting that coronary steal is occurring. Similarly, dipyridamole causes coronary artery vessel dilation. This causes coronary steal by causing non-ischemic parts of the heart to vasodilate thus "stealing" blood from the ischemic parts leading to symptoms of ischemia. COPY

This patient is presenting with chronic fatty, foul-smelling stools, fat soluble vitamin deficiencies, and a history of anaphylaxis with transfusion suggesting a diagnosis of --- disease. The best confirmatory test for --- disease is bowel wall biopsy.

This patient is presenting with chronic fatty, foul-smelling stools, fat soluble vitamin deficiencies, and a history of anaphylaxis with transfusion suggesting a diagnosis of celiac disease. The best confirmatory test for celiac disease is bowel wall biopsy.

This patient is presenting with confusion, hypotension (from diuresis), QT shortening, and focal back pain suggesting a diagnosis of hypercalcemia (likely from malignancy) which often also causes --.

This patient is presenting with confusion, hypotension (from diuresis), QT shortening, and focal back pain suggesting a diagnosis of hypercalcemia (likely from malignancy) which often also causes constipation.

This patient is presenting with cyclical fever, malaise, myalgias, gastrointestinal symptoms, anemia, and thrombocytopenia following travel to West Africa. These are classic findings for-- -- --- which must be treated with --- to eradicate the dormant liver hypnozoite stage of this infection.

This patient is presenting with cyclical fever, malaise, myalgias, gastrointestinal symptoms, anemia, and thrombocytopenia following travel to West Africa. These are classic findings for P. ovale malaria which must be treated with primaquine to eradicate the dormant liver hypnozoite stage of this infection. Although chloroquine is still the treatment of choice for chloroquine-sensitive strains of malaria, it would not be definitive treatment for P. vivax or P. ovale species of malaria as it does not treat the liver hypnozoite stage.

charcot joint in b12 def

This patient is presenting with deformity and pain in his right foot suggesting a diagnosis of neurogenic arthropathy. In the setting of his normal BMI and a plant-based diet, vitamin B12 deficiency is the most likely diagnosis (which is associated with a megaloblastic anemia). Charcot joint (neurogenic arthropathy) occurs secondary to a peripheral neurologic disturbance which results in altered sensation/proprioception of the joint. This neurologic deficit to the joint results in undetected repeat trauma which damages the joints and soft tissues. Vitamin B12 deficiency (or pernicious anemia) can result in dorsal column degeneration and subsequent decreases in proprioception leading to this presentation. It is common in elderly patients and vegans due to dietary deficiency. Other associated complications with vitamin B12 deficiency include a megaloblastic anemia.

This patient is presenting with diffuse muscle/joint pain, Raynaud's phenomenon, and dysphagia suggesting a diagnosis of ------. ---- antibodies are found in this disease.

This patient is presenting with diffuse muscle/joint pain, Raynaud's phenomenon, and dysphagia suggesting a diagnosis of mixed connective tissue disease. Anti-ribonucleoprotein (U1) antibodies are found in this disease. Mixed connective tissue disease can present with a vast array of symptoms. The most common symptoms are Raynaud's phenomenon (vascular hyper-reactivity that presents with burning/pain in the hands where the hands change color from white to blue to red, seen in Figure A), arthralgias, myalgias, dysphagia, and edema of the hands. Anti-U1 ribonucleoprotein antibodies are commonly found in mixed connective tissue disease and are a specific marker. In this disease there is typically an absence of renal involvement. Treatment of this pathology involves steroids.

This patient is presenting with fatigue, weight loss, abdominal pain that has been worsening, and Trousseau's sign which is concerning for a diagnosis of underlying visceral malignancy. The next best step in management is an abdominal CT to evaluate for a source of malignancy.

This patient is presenting with fatigue, weight loss, abdominal pain that has been worsening, and Trousseau's sign which is concerning for a diagnosis of underlying visceral malignancy. The next best step in management is an abdominal CT to evaluate for a source of malignancy. Trousseau's sign (migratory superficial thrombophlebitis) presents with multiple tender nodules (thrombi) that are palpable along the venous system. This physical exam finding is highly concerning for visceral malignancy. Patients who present with this finding, in addition to other symptoms that could suggest underlying malignancy (fatigue, weight loss, and jaundice), should be evaluated with an abdominal CT to search for an underlying malignancy. The most common malignancy that causes migratory superficial thrombophlebitis is pancreatic cancer.

flank pain radiating to the groin and hematuria suggesting a diagnosis of -- (kidney stone). Since the patient is pregnant, the most appropriate next step is to obtain -- ---

This patient is presenting with flank pain radiating to the groin and hematuria suggesting a diagnosis of nephrolithiasis (kidney stone). Since the patient is pregnant, the most appropriate next step is to obtain an abdominal ultrasound.

This patient is presenting with laboratory values suggestive of Paget disease of the bone which is associated with -- and -- skeletal lesions. Paget disease of the bone typically presents in elderly patients with an elevated -- , elevated -- --, and a pathologic fracture. However, another very common presentation for Paget disease of the bone is an asymptomatic patient with an elevated alkaline phosphatase. To ensure that the alkaline phosphatase is coming from bone (since it can come from the GI tract and the skeleton), a gamma-glutamyltransferase (GGT) level can be used to differentiate the origin (as it will be elevated in liver or gallbladder pathology and normal in a skeletal pathology). Paget disease of the bone is associated with lytic and blastic lesions in the bone.

This patient is presenting with laboratory values suggestive of Paget disease of the bone which is associated with blastic and lytic skeletal lesions. Paget disease of the bone typically presents in elderly patients with an elevated calcium, elevated alkaline phosphatase, and a pathologic fracture. However, another very common presentation for Paget disease of the bone is an asymptomatic patient with an elevated alkaline phosphatase. To ensure that the alkaline phosphatase is coming from bone (since it can come from the GI tract and the skeleton), a gamma-glutamyltransferase (GGT) level can be used to differentiate the origin (as it will be elevated in liver or gallbladder pathology and normal in a skeletal pathology). Paget disease of the bone is associated with lytic and blastic lesions in the bone.

This mother is experiencing mastitis and should be counseled to continue breastfeeding from the affected breast, which decreases the progression of mastitis to breast abscess. Mastitis is not a contraindication to breastfeeding.

This patient is presenting with lethargy, vomiting, and seizures at 6 months of age (the age at which infants begin consuming solid/pureed foods) suggesting a diagnosis of hereditary fructose intolerance (aldolase B deficiency).

This patient is presenting with muscle rigidity and fever following medication administration (likely due to metoclopramide), suggesting a diagnosis of --- -- ---The best initial step in management is to administer ---. Neuroleptic malignant syndrome occurs secondary to D2 receptor blockade (e.g., antipsychotics and antiemetics). The treatment of choice is dantrolene which depresses excitation-contraction coupling in skeletal muscle by acting as a ryanodine receptor antagonist and decreasing free intracellular calcium.

This patient is presenting with muscle rigidity and fever following medication administration (likely due to metoclopramide), suggesting a diagnosis of neuroleptic malignant syndrome. The best initial step in management is to administer dantrolene. Neuroleptic malignant syndrome occurs secondary to D2 receptor blockade (e.g., antipsychotics and antiemetics). The treatment of choice is dantrolene which depresses excitation-contraction coupling in skeletal muscle by acting as a ryanodine receptor antagonist and decreasing free intracellular calcium.

This patient is presenting with non-pruritic, hypopigmented patches on her hands and armpits, suggesting the diagnosis of --. Vitiligo can be associated w -- ---, for which a characteristic symptom is --.

This patient is presenting with non-pruritic, hypopigmented patches on her hands and armpits, suggesting the diagnosis of vitiligo. Vitiligo can be associated with pernicious anemia, for which a characteristic symptom is glossitis.

This patient is presenting with periods of inattentiveness in school suggesting a diagnosis of -- seizures. In addition, she also has abnormal, involuntary spasms of her upper extremity suggesting a diagnosis of -- -- --. The best treatment for these conditions is --

This patient is presenting with periods of inattentiveness in school suggesting a diagnosis of absence seizures. In addition, she also has abnormal, involuntary spasms of her upper extremity suggesting a diagnosis of juvenile myoclonic epilepsy. The best treatment for these conditions is valproic acid.

poor glycemic control and cranial nerve III palsy suggesting that her neurologic symptoms are secondary to poorly managed ---- Diabetes mellitus occurs when the demand for insulin exceeds the capacity of the pancreas to produce insulin, resulting in hyperglycemia. Subsequently, the patient's increased blood glucose, if unmanaged, can lead to many vascular complications. Common complications include cardiovascular disease and peripheral neuropathy. The somatic fibers of cranial nerve III are particularly affected, resulting in damage to these fibers causing a "down and out" eye and ptosis. Given the anatomic location, the parasympathetic fibers are intact; thus, the patient will still have a normal pupillary response to light. The best treatments for the complications of diabetes are weight loss, exercise, proper glycemic control, and medication compliance.

This patient is presenting with poor glycemic control and cranial nerve III palsy suggesting that her neurologic symptoms are secondary to poorly managed diabetes mellitus. Diabetes mellitus occurs when the demand for insulin exceeds the capacity of the pancreas to produce insulin, resulting in hyperglycemia. Subsequently, the patient's increased blood glucose, if unmanaged, can lead to many vascular complications. Common complications include cardiovascular disease and peripheral neuropathy. The somatic fibers of cranial nerve III are particularly affected, resulting in damage to these fibers causing a "down and out" eye and ptosis. Given the anatomic location, the parasympathetic fibers are intact; thus, the patient will still have a normal pupillary response to light. The best treatments for the complications of diabetes are weight loss, exercise, proper glycemic control, and medication compliance.

more purulent, days after: gonococcal conjunct less purulent, weeks after: chlamydial conj: rx? se?

This patient is presenting with purulent drainage from his eyes several weeks after a vaginal birth suggesting a diagnosis of chlamydial conjunctivitis. Chlamydial conjunctivitis is best treated with topical erythromycin which increases the risk of pyloric stenosis (which causes non-bilious projectile vomiting). Bacterial conjunctivitis presents with purulent drainage from the eyes. In pediatric patients, the most likely cause of conjunctivitis after birth is secondary to Neisseria gonorrhea or Chlamydia trachomatis. N. gonorrhea typically causes a more severe presentation with profuse purulent drainage from the eyes that occurs days after birth. In contrast, C. trachomatis presents with a less severe purulent discharge that typically occurs weeks after birth. The treatment of choice for chlamydial conjunctivitis is topical erythromycin which is typically administered at birth prophylactically. Both topical and oral erythromycin increase the risk of pyloric stenosis, in particular, in male newborns.

This patient is presenting with recurrent, painful oral and genital aphthous ulcers and erythema nodosum, suggesting the diagnosis of--- --, which may be complicated by the high risk of --- .

This patient is presenting with recurrent, painful oral and genital aphthous ulcers and erythema nodosum, suggesting the diagnosis of Behcet syndrome, which may be complicated by the high risk of thrombosis. BEHCET THINK THROMBOSIS

This patient is presenting with serous otitis media with an absence of infectious symptoms. The most appropriate management is supportive therapy and outpatient referral. Serous otitis media presents with a middle ear effusion and signs of conductive hearing loss without signs of infection (erythema and fever). The tympanic membrane will be hypomobile on otoscopy. In HIV, this typically occurs secondary to either obstructive lymphadenompathy or lymphomas. In an asymptomatic patient without any other symptoms, this patient needs only supportive therapy and outpatient workup.

This patient is presenting with serous otitis media with an absence of infectious symptoms. The most appropriate management is supportive therapy and outpatient referral. Serous otitis media presents with a middle ear effusion and signs of conductive hearing loss without signs of infection (erythema and fever). The tympanic membrane will be hypomobile on otoscopy. In HIV, this typically occurs secondary to either obstructive lymphadenompathy or lymphomas. In an asymptomatic patient without any other symptoms, this patient needs only supportive therapy and outpatient workup. Serous otitis media presents with a middle ear effusion, conductive hearing loss, and an absence of infectious symptoms (fever or erythema of the tympanic membrane).

This patient is presenting with shortness of breath shortly after a blood transfusion suggesting a diagnosis of transfusion-associated lung injury (TRALI). TRALI occurs due to donor antibody binding of recipient --.

This patient is presenting with shortness of breath shortly after a blood transfusion suggesting a diagnosis of transfusion-associated lung injury (TRALI). TRALI occurs due to donor antibody binding of recipient leukocytes.

This patient is presenting with stiffness/pain in his back that is relieved with activity suggesting a diagnosis of ankylosing spondylitis. Ankylosing spondylitis is associated with psoriasis which can present with punctate bleeding spots when scales are removed (Auspitz sign).

This patient is presenting with stiffness/pain in his back that is relieved with activity suggesting a diagnosis of ankylosing spondylitis. Ankylosing spondylitis is associated with psoriasis which can present with punctate bleeding spots when scales are removed (Auspitz sign).

sumatriptan can cause --

This patient is presenting with symptoms of a stroke after taking sumatriptan and dihydroergotamine suggesting a diagnosis of a medication-induced stroke. Triptans and dihydroergotamine are common medications used to treat migraine headache and can induce vasospasm, potentially causing stroke and myocardial infarction.

This patient is presenting with symptoms of anaphylaxis with initiation of a blood transfusion, suggesting a diagnosis of IgA deficiency. IgA deficiency is common in celiac disease, which can present with fat soluble vitamin deficiencies including vitamin E deficiency. Vitamin E deficiency presents with hemolytic anemia and ataxia.

This patient is presenting with symptoms of anaphylaxis with initiation of a blood transfusion, suggesting a diagnosis of IgA deficiency. IgA deficiency is common in celiac disease, which can present with fat soluble vitamin deficiencies including vitamin E deficiency. Vitamin E deficiency presents with hemolytic anemia and ataxia.

This patient is presenting with symptoms of hyperviscosity (fatigue, bl vision, HA, dizzy)fatigue, and organomegaly suggestive of a diagnosis of -- -- . WM is characterized by increased -- production.

This patient is presenting with symptoms of hyperviscosity, fatigue, and organomegaly suggestive of a diagnosis of Waldenstrom macroglobulinemia (WM). WM is characterized by increased IgM production. WM occurs secondary to a clonal B-cell pathologically overproducing IgM. High levels of IgM can lead to symptoms of hyperviscosity including blurred vision, dizziness, headaches, and peripheral neuropathy. Organomegaly is a unique finding in WM and can present with hepatomegaly and splenomegaly. Other non-specific findings include fatigue, anemia, and lymphadenopathy.

This patient is presenting with symptoms suggestive of lichen sclerosus (LS). The best initial step in management is to administer topical--. LS presents with white plaques, atrophy, and scarring secondary to scratching from the patient. The lesions are extremely pruritic and can be very distressing to patients. The best initial step in management with a classic presentation is to administer topical clobetasol. Further diagnostic workup is often unnecessary if the patient responds to treatment.

This patient is presenting with symptoms suggestive of lichen sclerosus (LS). The best initial step in management is to administer topical clobetasol. LS presents with white plaques, atrophy, and scarring secondary to scratching from the patient. The lesions are extremely pruritic and can be very distressing to patients. The best initial step in management with a classic presentation is to administer topical clobetasol. Further diagnostic workup is often unnecessary if the patient responds to treatment.

This patient is presenting with vascular claudication and erectile dysfunction suggesting a diagnosis of aortoiliac occlusive disease. The best initial step in management is an ---

This patient is presenting with vascular claudication and erectile dysfunction suggesting a diagnosis of aortoiliac occlusive disease. The best initial step in management is an ankle-brachial index.

This patient is suffering from acute early pericarditis after a heart attack. -- is the recommended first-line treatment for this condition.

This patient is suffering from acute early pericarditis after a heart attack. High-dose aspirin (650 to 1000 mg three times per day) is the recommended first-line treatment for this condition.

This patient is suffering from massive hemothorax. Greater than 1500 cc of chest-tube output or continued bleeding and chest-tube output greater than 250 cc per hour are indications for open thoracotomy.

This patient is suffering from massive hemothorax. Greater than 1500 cc of chest-tube output or continued bleeding and chest-tube output greater than 250 cc per hour are indications for open thoracotomy.

this middle-aged woman presents with flaccid bullae of the dorsal hands, evidence of fragile skin, increased hair growth (hypertrichosis) of the face, and hyperpigmentation consistent with porphyria cutanea tarda (PCT). Urine testing in a patient with PCT will show elevated levels of several porphyrins including uroporphyrin, coproporphyrin, and hepta-carboxylated porphyrins. char by -- bodies

This patient is suffering from porphyria cutanea tarda (PCT). PCT is characterized histologically by 'caterpillar bodies' - linear, eosinophilic, periodic acid-Schiff-positive globules - found in the blisters and skin lesions of PCT patients. PCT is characterized by a deficiency of the uroporphyrinogen decarboxylase enzyme that plays a role in heme synthesis. Eighty percent of cases are acquired (induced through inhibition of the hepatic enzyme secondary to exposure to certain chemicals) and 20% are inherited in an autosomal dominant fashion. The clinical presentation includes increased skin fragility and blister formation on sun-exposed surfaces - most commonly the face, dorsal hands, forearms, and lower legs. Additionally, hypertrichosis and hyperpigmentation manifests on the temples and cheeks. Phlebotomy, to reduce the excess liver porphyrin stores, cessation of the provoking agent, and hydroxychloroquine, to help clear the skin lesions, are the most common components of treatment.

u/l hear loss sequelae of ---

This patient is suffering from serous otitis media, which is seen in HIV with lymphadenopathy. Serous otitis media is a condition which results from obstruction of the fluid outflow tracts (Eustacian tubes) and consequent buildup of fluid in the middle ear. Of note, the fluid is not infected. This can be caused by many conditions, including lymphomas or lymphadenopathy. On exam, the tympanic membrane is dull and hypomobile on pneumatic otoscopy (as a result of the fluid behind it).

This patient most likely has antiphospholipid antibody syndrome given her recurrent miscarriages and absence of any other symptoms or medical history. Patients with this syndrome can take -- and --- to reduce pre loss risk

This patient most likely has antiphospholipid antibody syndrome given her recurrent miscarriages and absence of any other symptoms or medical history. Patients with this syndrome can take aspirin and low molecular weight heparin to reduce the risk of pregnancy loss. AB to beta2-glycoprotein1. Anti cardiolipin and lupus anticoagulant enzyme.

This patient presents after intensive ballet practice with heel pain notable on dorsiflexion, which is most consistent with -- --. The best next step in management is resting of the foot.

This patient presents after intensive ballet practice with heel pain notable on dorsiflexion, which is most consistent with plantar fasciitis. The best next step in management is resting of the foot.

This patient presents following a vaccum-assisted vaginal delivery with an ecchymotic swelling of the scalp that crosses the midline, which suggests a diagnosis of -- -- . Caput succedaneum is caused by bleeding between the skin and the --- aponeurosis.

This patient presents following a vaccum-assisted vaginal delivery with an ecchymotic swelling of the scalp that crosses the midline, which suggests a diagnosis of caput succedaneum. Caput succedaneum is caused by bleeding between the skin and the galea aponeurosis.

This patient presents with a corneal foreign body. The most common organsim cultured from corneal foreign bodies is ---.

This patient presents with a corneal foreign body. The most common organsim cultured from corneal foreign bodies is coagulase negative staphylococcus.

This patient presents with a firm, movable, nontender cyst with white malodorous exudate. The most likely diagnosis is an-- -- --

This patient presents with a firm, movable, nontender cyst with white malodorous exudate. The most likely diagnosis is an epidermal inclusion cyst.

This patient presents with bilious emesis, delayed passage of meconium, diffuse colonic distension on abdominal radiograph, and a family history of severe pulmonary disease, which is consistent with a diagnosis of -- -- caused by -- --. Sweat testing should be performed to evaluate for underlying cystic fibrosis. Patients with cystic fibrosis are at risk for meconium ileus, which presents with bilious emesis and delayed passage of meconium. Abdominal radiograph may demonstrate dilated loops of bowel, and contrast enema reveals the presence of a microcolon. Cystic fibrosis may also present in the neonatal period with failure to thrive, pancreatic insufficiency, or respiratory symptoms. Sweat testing can be performed as early as the second day of life in neonates with symptoms suggestive of cystic fibrosis.

This patient presents with bilious emesis, delayed passage of meconium, diffuse colonic distension on abdominal radiograph, and a family history of severe pulmonary disease, which is consistent with a diagnosis of meconium ileus caused by cystic fibrosis. Sweat testing should be performed to evaluate for underlying cystic fibrosis. Patients with cystic fibrosis are at risk for meconium ileus, which presents with bilious emesis and delayed passage of meconium. Abdominal radiograph may demonstrate dilated loops of bowel, and contrast enema reveals the presence of a microcolon. Cystic fibrosis may also present in the neonatal period with failure to thrive, pancreatic insufficiency, or respiratory symptoms. Sweat testing can be performed as early as the second day of life in neonates with symptoms suggestive of cystic fibrosis.

This patient presents with dark amniotic fluid, respiratory distress, meconium-stained nails, and hyperinflated lungs with patchy infiltrates, which suggests a diagnosis of meconium aspiration syndrome. Meconium aspiration syndrome is caused by intrauterine passage of meconium.

This patient presents with dark amniotic fluid, respiratory distress, meconium-stained nails, and hyperinflated lungs with patchy infiltrates, which suggests a diagnosis of meconium aspiration syndrome. Meconium aspiration syndrome is caused by intrauterine passage of meconium. Meconium aspiration syndrome (MAS) presents in infants born in meconium-stained amniotic fluid. The mechanisms of pulmonary disease in MAS are thought to include airway obstruction, chemical irritation and inflammation, and interference with surfactant metabolism. Patients typically present with a history of meconium-stained amniotic fluid, marked respiratory distress, and chest radiography demonstrating patchy infiltrates alternating with areas of expansion, hyperinflation, and flattening of the diaphragm. Management of MAS is supportive and involves maintaining adequate oxygenation, ventilation, and blood pressure.

dvt unresponsive to heparin, with normal response being increased PTT, suspect ----. do what?

This patient presents with deep vein thrombosis (DVT) and is not responding to heparin suggesting a diagnosis of antithrombin III deficiency. Management of these patients begins with a direct thrombin inhibitor (argatroban) followed by warfarin. DVTs present with pain, erythema, and inflammation of the lower extremity typically in the setting of a hypercoaguable state (such as a clotting disorder, malignancy, orthopedic surgery, or stasis). Management is heparin bridged to warfarin. In patients who do not respond to heparin (with a normal response being an elevation in PTT), antithrombin III deficiency should be suspected. The appropriate anticoagulation therapy in antithrombin III deficiency patients is a direct thrombin inhibitor (such as argatroban) as a bridge followed by warfarin long term. Figure A demonstrates the right lower extremity (arrow) as enlarged (circumference around the calf) as compared to the left lower extremity suggesting a diagnosis of a DVT.

cleft lip, tet fallot, recurrent sinopulm infxn, hypoplastic thymus

This patient presents with dysmorphic features, cleft lip, signs and symptoms of tetralogy of Fallot, a history of recurrent sinopulmonary infections, and a chest radiograph demonstrating a hypoplastic thymus, suggesting a diagnosis of DiGeorge syndrome. Patients with DiGeorge syndrome have immunodeficiency due to a reduced number of T-cells from hypoplasia of the thymus. DiGeorge syndrome presents with the acronym CATCH-22: Congenital heart defects, Abnormal facies, Thymic Aplasia or hypoplasia, Cleft lip or palate, and Hypocalcemia caused by a deletion in 22q11.2. The degree of immunodeficiency in these patients depends on the degree of thymic aplasia or hypoplasia. The majority of patients with DiGeorge syndrome have a hypoplastic thymus that leads to a reduction in T-cell count and presents with recurrent sinopulmonary infections, including otitis media, sinusitis, bronchitis, and pneumonia. If the thymus is completely absent, patients will have recurrent life-threatening infections. Figure A demonstrates a hypoplastic thymus on chest radiograph. Illustration A demonstrates a normal chest radiograph for comparison, in which the thymic "sail sign" can be seen as a triangular density along the upper right sternal border.

This patient presents with fetal macrosomia, unexplained neonatal hypoglycemia, macroglossia, and hemihypertrophy, which suggests a diagnosis of -- -- --. Patients with BWS should undergo screening of the --, as BWS is strongly associated with --

This patient presents with fetal macrosomia, unexplained neonatal hypoglycemia, macroglossia, and hemihypertrophy, which suggests a diagnosis of Beckwith-Wiedemann syndrome (BWS). Patients with BWS should undergo screening of the kidneys, as BWS is strongly associated with Wilms' tumor.

This patient presents with fever, abdominal pain and cervical motion tenderness, vaginal discharge, and an adnexal mass, suggestive of ---- and possible----. Given her nausea and vomiting and the concern for TOA, the next best step to treat this condition is inpatient admission with intravenous cefotetan and doxycycline.

This patient presents with fever, abdominal pain and cervical motion tenderness, vaginal discharge, and an adnexal mass, suggestive of pelvic inflammatory disease (PID) and possible tubo-ovarian abscess (TOA). Given her nausea and vomiting and the concern for TOA, the next best step to treat this condition is inpatient admission with intravenous cefotetan and doxycycline.

This patient presents with fever, facial pain, double vision, and necrosis of the hard palate in the setting of poorly controlled diabetes mellitus, which suggests a diagnosis of craniofacial --. The treatment of choice for mucormycosis is -- and surgical debridement.

This patient presents with fever, facial pain, double vision, and necrosis of the hard palate in the setting of poorly controlled diabetes mellitus, which suggests a diagnosis of craniofacial mucormycosis. The treatment of choice for mucormycosis is amphotericin and surgical debridement.

This patient presents with flu-like symptoms, polyarthralgias, leukopenia, thrombocytopenia, and transaminitis, which suggests a diagnosis of --.

This patient presents with flu-like symptoms, polyarthralgias, leukopenia, thrombocytopenia, and transaminitis, which suggests a diagnosis of chikungunya. Chikungunya is most commonly found in Central America, South America, and tropical regions in Africa and South Asia. It is transmitted by the Aedes mosquito, which is also the vector for dengue fever, yellow fever, West Nile fever, and Zika virus. Chikungunya most commonly presents with headache, malaise, myalgias, polyarthralgias, maculopapular rash, lymphopenia, thrombocytopenia, and elevated liver enzymes.

This patient presents with globally delayed development, a social personality, dysmorphic features, short stature, strabismus, and hypercalcemia, which suggests a diagnosis of -- syndrome. Williams syndrome is associated with -- --

This patient presents with globally delayed development, a social personality, dysmorphic features, short stature, strabismus, and hypercalcemia, which suggests a diagnosis of Williams syndrome. Williams syndrome is associated with supravalvular aortic stenosis.

hydatid cyst=

This patient presents with hydatid cysts with large cysts enclosing daughter cysts. Treatment is with surgical resection and mebendazole. If cyst contents are spilled, there is a risk for anaphylaxis, which can be minimized by injection of ethanol, which kills daughter cysts, prior to resection. Hydatid liver cysts are caused by infection from the tapeworm Echinococcus granulosus or multilocularis. Contact with sheep is a risk factor and may have contributed to the infection in this farmer. Most often cysts are found in the right lobe of the liver. While small cysts caused by the microorganism are asymptomatic, larger ones may present as right upper quadrant pain and may eventually rupture into the peritoneal cavity causing anaphylactic shock. Large cysts are thus treated with surgical resection preceded by ethanol injection to avoid spilling the contents of the cysts into the abdominal cavity. Mebendazole is given after finishing the resection.

This patient presents with neutropenic fever, hemoptysis, and cavitary lung lesions on imaging consistent with Aspergillosis. The initial treatment of choice for invasive Aspergillus in these patients is systemic antifungals, including ---.

This patient presents with neutropenic fever, hemoptysis, and cavitary lung lesions on imaging consistent with Aspergillosis. The initial treatment of choice for invasive Aspergillus in these patients is systemic antifungals, including voriconazole. and or ampho B

This patient presents with new-onset worsening headaches, polyuria, and polydipsia (concerning for diabetes insipidus), and peripheral vision loss (bitemporal hemianopsia) which are findings suggestive of a -- -- -- The unenhanced head CT showing a suprasellar calcified cystic mass is most consistent with a craniopharyngioma.

This patient presents with new-onset worsening headaches, polyuria, and polydipsia (concerning for diabetes insipidus), and peripheral vision loss (bitemporal hemianopsia) which are findings suggestive of a growing intracranial mass in the sellar region. The unenhanced head CT showing a suprasellar calcified cystic mass is most consistent with a craniopharyngioma.

This patient presents with petechiae, gingival hemorrhage, and coiled body hair, most consistent with ----. The best next step in management upon clinical suspicion of scurvy is supplementation with vitamin C.

This patient presents with petechiae, gingival hemorrhage, and coiled body hair, most consistent with vitamin C deficiency or scurvy. The best next step in management upon clinical suspicion of scurvy is supplementation with vitamin C.

This patient presents with scattered salmon-colored macules on the nape of the neck, which suggests a diagnosis of -- --

This patient presents with scattered salmon-colored macules on the nape of the neck, which suggests a diagnosis of nevus simplex.

This patient presents with short stature, low hairline, and a broad chest, which suggests a diagnosis of Turner syndrome (45, X). Patients with Turner syndrome typically have absent -- --- on buccal smear. Turner syndrome presents in children with short stature and dysmorphic features of a low hairline, webbed neck, and broad chest with widely spaced nipples. Because Turner syndrome is caused by loss of an X chromosome, most patients will have absent Barr bodies on buccal smear. However, some patients with Turner syndrome have 45, X mosaicism or partial loss the X chromosome rather than complete absence, which makes the buccal smear an unreliable test. Patients with indications for screening should undergo karyotype analysis for formal diagnosis.

This patient presents with short stature, low hairline, and a broad chest, which suggests a diagnosis of Turner syndrome (45, X). Patients with Turner syndrome typically have absent Barr bodies on buccal smear. Turner syndrome presents in children with short stature and dysmorphic features of a low hairline, webbed neck, and broad chest with widely spaced nipples. Because Turner syndrome is caused by loss of an X chromosome, most patients will have absent Barr bodies on buccal smear. However, some patients with Turner syndrome have 45, X mosaicism or partial loss the X chromosome rather than complete absence, which makes the buccal smear an unreliable test. Patients with indications for screening should undergo karyotype analysis for formal diagnosis.

This patient presents with tall stature, upward lens subluxation, and a family history of aortic valve disease, which suggests a diagnosis of Marfan's syndrome. Marfan's syndrome is caused by an autosomal dominant mutation in the fibrillin-1 gene on chromesome --

This patient presents with tall stature, upward lens subluxation, and a family history of aortic valve disease, which suggests a diagnosis of Marfan's syndrome. Marfan's syndrome is caused by an autosomal dominant mutation in the fibrillin-1 gene on chromosome 15.

This patient presents with the classic rash of lymphocutaneous sporotrichosis--a reddish nodule that later ulcerates at the site of a thorn prick or skin injury, then spreads along lymphatics, resulting in subcutaneous nodules and ulcers. Sporotrichosis infection is caused by the dimorphic fungus Sporothrix schenckii. Although sporotrichosis is also known as "gardener's disease", any activity with the potential for skin inoculation with soil presents an infection risk. Infection beyond the skin and lymphatics (e.g., osteoarticular, pulmonary, and disseminated sporotrichosis) is rare and typically occurs in immunocompromised patients. rx w?

This patient presents with the classic rash of lymphocutaneous sporotrichosis--a reddish nodule that later ulcerates at the site of a thorn prick or skin injury, then spreads along lymphatics, resulting in subcutaneous nodules and ulcers. Sporotrichosis infection is caused by the dimorphic fungus Sporothrix schenckii. Although sporotrichosis is also known as "gardener's disease", any activity with the potential for skin inoculation with soil presents an infection risk. Infection beyond the skin and lymphatics (e.g., osteoarticular, pulmonary, and disseminated sporotrichosis) is rare and typically occurs in immunocompromised patients. rx w itraconazole

This patient was given an initial dose of radiation therapy to destroy malignant cells which is termed -- --

This patient was given an initial dose of radiation therapy to destroy malignant cells which is termed induction therapy. salvage therapy is given in the case of treatment failure and is a final attempt at eradicating the malignancy.

This patient who is breastfed is presenting with seizures, hypoglycemia, cataracts, and hepatomegaly suggesting a diagnosis of -- . Treatment of galactosemia involves cessation of -- intake.

This patient who is breastfed is presenting with seizures, hypoglycemia, cataracts, and hepatomegaly suggesting a diagnosis of galactosemia. Treatment of galactosemia involves cessation of galactose intake. pts w galactosemia should not be breastfed and no galactose in the diet.

This patient with Marfan syndrome and a family history of possible sudden death from aortic disease most likely has a thoracic aortic dissection, which should be evaluated with --- of the aorta.

This patient with Marfan syndrome and a family history of possible sudden death from aortic disease most likely has a thoracic aortic dissection, which should be evaluated with CT angiography of the aorta.

This patient with a history of kidney transplant 2 months ago is presenting with oliguria, hypertension, peripheral edema, and a biopsy revealing lymphocytic infiltration, which is consistent with --- . pcIntravenous (IV) --- should be administered to arrest any acute immune reaction against the renal graft.

This patient with a history of kidney transplant 2 months ago is presenting with oliguria, hypertension, peripheral edema, and a biopsy revealing lymphocytic infiltration, which is consistent with graft failure due to acute rejection. Intravenous (IV) steroids should be administered to arrest any acute immune reaction against the renal graft. Acute transplant rejection can occur when recipient CD8+ T-cells react against donor antigens or when antibodies are formed status post-transplant. This usually presents within weeks to months after transplant. For kidney transplants, patients may present with oliguria, hypertension, increased creatinine and BUN, and signs of volume overload. A renal biopsy will show lymphocyte infiltration and vascular involvement with intimal swelling. The next best step in management is to continue calcineurin inhibitors (i.e., tacrolimus and cyclosporine) and to add systemic steroids, initially intravenously.

This patient with a history of sickle cell disease is presenting with signs of a stroke (i.e., facial droop and dysarthria), likely caused by a vaso-occlusive sickle cell crisis. Treatment is -- --

This patient with a history of sickle cell disease is presenting with signs of a stroke (i.e., facial droop and dysarthria), likely caused by a vaso-occlusive sickle cell crisis. Treatment is exchange transfusion. The presentation of a sickle cell crisis depends on where the vaso-occlusion occurs. A severe complication of sickle cell disease is a stroke. A stroke in a sickle cell patient presents similarly to an ischemic stroke of the general population, but the treatment is different. In the immediate period, effective oxygenation is important. A simple transfusion can be used to increase hemoglobin levels if very low, but the ultimate treatment is exchange transfusion. The goal of exchange transfusion is to both lower the percentage of sickled hemoglobin and increase the total hemoglobin level.

This patient with chronic steatorrhea, weight loss, low-grade fever, and a positive Periodic acid-Schiff (PAS) stained small bowel biopsy mostly likely has -- -- . Treatment of Whipple disease includes --

This patient with chronic steatorrhea, weight loss, low-grade fever, and a positive Periodic acid-Schiff (PAS) stained small bowel biopsy mostly likely has Whipple disease. Treatment of Whipple disease includes trimethoprim/sulfamethoxazole (TMP-SMX) for 1 year.

This patient with intrusive, violent thoughts and compulsions to confess and recite prayers aloud has obsessive-compulsive disorder (OCD). He has failed first-line treatment with selective serotonin reuptake inhibitors (SSRIs), making --- the appropriate second-line medication choice.

This patient with intrusive, violent thoughts and compulsions to confess and recite prayers aloud has obsessive-compulsive disorder (OCD). He has failed first-line treatment with selective serotonin reuptake inhibitors (SSRIs), making clomipramine the appropriate second-line medication choice.

This patient's bilateral headache, ptosis, proptosis, swelling, papilledema, fever, recent rhinosinusitis infection, and ophthalmoplegia are highly concerning for -- -- --

This patient's bilateral headache, ptosis, proptosis, swelling, papilledema, fever, recent rhinosinusitis infection, and ophthalmoplegia are highly concerning for cavernous sinus thrombosis. orb cellulitis is only one eye. Cavernous sinus thrombosis can result in impingement of the cranial nerves within the cavernous sinus (oculomotor, trochlear, the first and second division of the trigeminal nerve, and abducens nerve).

This patient's clinical presentation is consistent with endometriosis. Patients with endometriosis should avoid ---, as they can promote endometrial growth.

This patient's clinical presentation is consistent with endometriosis. Patients with endometriosis should avoid estrogen-only contraceptive pills, as they can promote endometrial growth.

This patient's clinical presentation is consistent with multiple sclerosis (MS), which is associated with an elevated -- in the CSF.

This patient's clinical presentation is consistent with multiple sclerosis (MS), which is associated with an elevated IgG in the CSF.

To answer this question, it is necessary to have some prior knowledge of the asthma severity categories. There are 4 such categories: intermittent, mild persistent, moderate persistent, and severe persistent. Intermittent asthma is not described as "mild" or "moderate", so answers 1 and 2 are distractors. In order to determine which category is most appropriate for this patient, consider his key pieces in the history: daily symptoms, twice weekly awakenings, daily albuterol use, and an FEV1 of 65%. In mild persistent asthma, patients have symptoms > 2 days/week (but not daily), nighttime awakenings 3-4 times/month, use a SABA > 2 days/week (but not daily), have minor interference with normal activity, and have an FEV1 > 80%. In moderate persistent asthma, patients have symptoms daily, nighttime awakenings > 1 night/week (but not every night), use SABA daily, and have an FEV1 60-80% predicted. In severe asthma, patients have symptoms throughout the day, nighttimes awakenings every night, use a SABA multiple times per day, have extreme interference with normal activity, and have FEV1 < 60%. Based on thes categorizations, the above patient fits in the category of moderate persistent asthma.

This patient's clinical presentation is consistent with rheumatic heart disease with valvular involvement. Patients with this condition should receive penicillin prophylaxis for 10 years, or until age 40 (whichever is longer). Rheumatic heart disease results from autoantigens that form in the setting of oropharyngeal infection with group A streptococcus. Cardiac tissue is one of the major autoantigen targets, which can lead to carditis and valvular abnormalities after resolution of the oropharyngeal, joint, and dermatological symptoms of acute rheumatic fever. Penicillin prophylaxis is recommended to prevent future infections with group A streptococcus, and the duration of treatment is dependent on the presence of carditis and valvular abnormalities.

This patient's clinical presentation is suggestive of post-operative urinary retention. The best next step in management --- followed by ----. Post-operative urinary retention resulting from impaired bladder function can be caused by bladder manipulation and anesthesia (especially spinal anesthesia). An ultrasound bladder scan is a non-invasive method of diagnosing urinary retention. Bladder scans allow for a rapid and accurate method of assessing urinary volumes. Urinary retention is commonly managed with intermittent urethral catheterization or a temporary indwelling Foley catheter followed by a voiding trial.

This patient's clinical presentation is suggestive of post-operative urinary retention. The best next step in management is a bladder scan followed by catheterization. Post-operative urinary retention resulting from impaired bladder function can be caused by bladder manipulation and anesthesia (especially spinal anesthesia). An ultrasound bladder scan is a non-invasive method of diagnosing urinary retention. Bladder scans allow for a rapid and accurate method of assessing urinary volumes. Urinary retention is commonly managed with intermittent urethral catheterization or a temporary indwelling Foley catheter followed by a voiding trial.

This patient's fetus is in breech presentation, requiring ECV. Because ECV can cause small amounts of fetal blood to enter maternal circulation, administration of --- is important for preventing Rh alloimmunization.

This patient's fetus is in breech presentation, requiring ECV. Because ECV can cause small amounts of fetal blood to enter maternal circulation, administration of Rhogam is important for preventing Rh alloimmunization. Rhogam should be given to Rh negative women exposed to external cephalic version, delivery, ectopic pregnancy, molar pregnancy, chorionic villus sampling/amniocentesis, or trauma, all of which which may expose the mother to small amounts of fetal blood. Routine administration of Rhogam is also recommended for all Rh negative women at 28-32 weeks gestation.

This patient's precocious puberty (pubic hair growth and Tanner stage 4 development before age 8), ovarian mass, and elevated estrogen level suggest-- -- --

This patient's precocious puberty (pubic hair growth and Tanner stage 4 development before age 8), ovarian mass, and elevated estrogen level suggest granulosa cell tumor. Granulosa cell tumor is the most common form of malignant stromal tumors of the ovaries. Although it predominately occurs in women over the age of 50, it can also occur in prepubescent children. The tumor often produces estrogen and/or progesterone and presents with postmenopausal bleeding, sexual precocity, or breast tenderness. Of note, histopathological findings of granulosa cell tumors are significant for Call-Exner bodies, which consists of granulosa cells arranged around eosinophilic fluid.

This patient's presentation is consistent with secondary syphilis, given the maculopapular rash on his palms, the white mucous patch on his tongue, and the visualization of spirochetes on dark-field microscopy. The negative RPR/VDRL test is a result of the -- phenomenon, which is caused by excess antibodies leading to an imbalance of the antibody to antigen ratio.

This patient's presentation is consistent with secondary syphilis, given the maculopapular rash on his palms, the white mucous patch on his tongue, and the visualization of spirochetes on dark-field microscopy. The negative RPR/VDRL test is a result of the prozone phenomenon, which is caused by excess antibodies leading to an imbalance of the antibody to antigen ratio.

Myoglobinuria commonly occurs in rhabdomyolysis and presents with blood detected on dipstick urinalysis without RBCs seen on urine microscopy. rhabdo= dipstick ONLY blood, no RBC on urine micro.

This patient's presentation is most consistent with rhabdomyolysis. Myoglobinuria commonly occurs in rhabdomyolysis and presents with blood detected on dipstick urinalysis without RBCs seen on urine microscopy.

This patient's presentation of an ovarian tumor, ascites, and right-sided pleural effusion suggests a diagnosis of -- syndrome. A benign ovarian --- is the most common tumor associated with Meigs syndrome.

This patient's presentation of an ovarian tumor, ascites, and right-sided pleural effusion suggests a diagnosis of Meigs syndrome. A benign ovarian fibroma is the most common tumor associated with Meigs syndrome.

This patient's presentation with arthralgia, abdominal tenderness, and maroon papules (palpable purpura) on the lower extremities following an upper respiratory infection is most consistent with a diagnosis of ---- -- is an uncommon leukocytoclastic vasculitis that is most often seen in children under 10 years of age. Its pathogenesis is related to IgA immune complex deposition in small blood vessels resulting in a leukocytoclastic vasculitis. This produces clinical manifestations such as non-tender, non-blanching palpable purpura on the lower extremities, arthritis, abdominal pain, fatigue, and occasionally low-grade fever. Urinalysis will reveal hematuria due to mesangial IgA immune complex deposits. Patients with HSP are additionally at an increased risk for ---. Treatment for HSP is supportive and consists of hydration and NSAIDs. Glucocorticoids may be used in severe presentations or in cases that fail to respond to initial management.

This patient's presentation with arthralgia, abdominal tenderness, and maroon papules (palpable purpura) on the lower extremities following an upper respiratory infection is most consistent with a diagnosis of Henoch-Schonlein purpura (HSP, also known as IgA vasculitis). HSP is an IgA immune complex-mediated leukocytoclastic vasculitis. HSP is an uncommon leukocytoclastic vasculitis that is most often seen in children under 10 years of age. Its pathogenesis is related to IgA immune complex deposition in small blood vessels resulting in a leukocytoclastic vasculitis. This produces clinical manifestations such as non-tender, non-blanching palpable purpura on the lower extremities, arthritis, abdominal pain, fatigue, and occasionally low-grade fever. Urinalysis will reveal hematuria due to mesangial IgA immune complex deposits. Patients with HSP are additionally at an increased risk for intussusception. Treatment for HSP is supportive and consists of hydration and NSAIDs. Glucocorticoids may be used in severe presentations or in cases that fail to respond to initial management.

This patient's symptoms of fatigue and night sweats point to a diagnosis of APML, which is a cause of disseminated intravascular coagulation (DIC). Promyelocyte blast cells secrete -- ----, also called tissue factor, which activates the coagulation cascade.

This patient's symptoms of fatigue and night sweats point to a diagnosis of acute promyelocytic leukemia (APML), which is a cause of disseminated intravascular coagulation (DIC). Promyelocyte blast cells secrete coagulation factor III, also called tissue factor, which activates the coagulation cascade.

This pregnant/immigrant patient is presenting with shortness of breath, atrial fibrillation, and an apical murmur suggesting a diagnosis of mitral stenosis. The most common cause of mitral stenosis is --- --- which can present with ---- ---- (serpiginous dermatologic macular lesion with a pale center). Mitral stenosis presents with a rumbling apical murmur associated with a diastolic snap. Patients can experience symptoms such as hoarseness and dysphagia as the left atrium enlarges and compresses adjacent structures. Dilation of the atria can disrupt the cardiac conduction system and result in atrial fibrillation. Symptoms of mitral stenosis are typically worsened in expanded volume states such as pregnancy.

This pregnant/immigrant patient is presenting with shortness of breath, atrial fibrillation, and an apical murmur suggesting a diagnosis of mitral stenosis. The most common cause of mitral stenosis is rheumatic fever, which can present with erythema marginatum (serpiginous dermatologic macular lesion with a pale center). Mitral stenosis presents with a rumbling apical murmur associated with a diastolic snap. Patients can experience symptoms such as hoarseness and dysphagia as the left atrium enlarges and compresses adjacent structures. Dilation of the atria can disrupt the cardiac conduction system and result in atrial fibrillation. Symptoms of mitral stenosis are typically worsened in expanded volume states such as pregnancy.

RDS neonatal rx?

This premature newborn presents with very low birth weight, tachypnea, signs of respiratory distress, and chest radiograph with diffuse ground glass opacities and low lung volumes, which suggests a diagnosis of neonatal respiratory distress syndrome. The best next step in management is positive pressure ventilation and surfactant administration. Neonatal respiratory distress syndrome (NRDS) occurs in premature infants and is caused by surfactant deficiency, which results in alveolar collapse and diffuse atelectasis. The incidence of NRDS increases with gestational age with the incidence in patients born at under 28 weeks gestation at 90%. Patients typically present in the first minutes or hours of life with progressive respiratory distress, central cyanosis, and chest radiography demonstrating low lung volumes and diffuse ground glass opacities. Betamethasone should be administered to mothers in preterm labor prior to delivery, and initial management of the newborn involves administration of surfactant and positive pressure ventilation.

This teenager is exhibiting behavioral patterns consistent with obsessive compulsive disorder (OCD). This disorder can be initially treated how?

This teenager is exhibiting behavioral patterns consistent with obsessive compulsive disorder (OCD). This disorder can be initially treated without medications by exposure and response prevention (cognitive-behavioral therapy).

new unilateral neuropathic pain manifesting in her tonsils following tympanoplasty on the same side. This patient most likely has -- --

This vignette describes a patient who has new unilateral neuropathic pain manifesting in her tonsils following tympanoplasty on the same side. This patient most likely has glossopharyngeal neuralgia.

This vignette describes a young boy with fatigue in the setting of pancytopenia and a relative lymphocytosis, concerning for acute lymphoblastic leukemia (ALL). The most specific assessment is what?

This vignette describes a young boy with fatigue in the setting of pancytopenia and a relative lymphocytosis, concerning for acute lymphoblastic leukemia (ALL). The most specific assessment, a bone marrow biopsy, would reveal > 25% lymphoblasts. BM BX, > 25% BLASTS

27 yo F rapid viriliz, elev testos, large L adnexal mass, solid on US

This young woman presents with rapid virilization, elevated testosterone, and a large left adnexal mass that is primarily solid on ultrasound, most consistent with an ovarian androgen-secreting tumor. Sertoli-Leydig cell tumors are the most common androgen-secreting tumor of the ovaries and classically result in quick virilization of affected females. Testosterone levels are often over 200 ng/dL, and 17-hydroxyprogesterone may also be slightly elevated as it is an androgen precursor. The androgen excess may result in symptoms such as facial and abdominal hirsutism, oligomenorrhea, deepening voice, and acne. its not CAH bc the Adnexal tenderness mentioned, meaning duh its ovarian tumor pathology

--- is associated with increased remission rates in patients with myasthenia gravis.

Thymectomy is associated with increased remission rates in patients with myasthenia gravis.

What is visible in radiograph in infants and changes response in infxns

Thymus

-- --- often presents with a scaly, erythematous patch on the scalp that may develop into alopecia with residual black dots. The best initial test for diagnosis is a KOH prep, the most accurate test is a fungal culture, and treatment is oral terbinafine, itraconazole, or griseofulvin.

Tinea capitis often presents with a scaly, erythematous patch on the scalp that may develop into alopecia with residual black dots. The best initial test for diagnosis is a KOH prep, the most accurate test is a fungal culture, and treatment is oral terbinafine, itraconazole, or griseofulvin.

-- --- often presents with a scaly, erythematous patch on the scalp that may develop into alopecia with residual black dots. Treatment consists of-- terbinafine, itraconazole, or griseofulvin. Topical anti-fungals cannot be used to treat tinea capitis, but may be used to prevent transmission.

Tinea capitis often presents with a scaly, erythematous patch on the scalp that may develop into alopecia with residual black dots. Treatment consists of oral terbinafine, itraconazole, or griseofulvin. Topical anti-fungals cannot be used to treat tinea capitis, but may be used to prevent transmission.

Showers of floaters and scotomata. Fine lacy vessels, preretinal and intraretinal hemorrhages, cotton wool spot and exudates.

Tractional retinal detachment- curtain drawn down

-- ---- of the newborn presents with worsening tachypnea within two hours of delivery, and it is caused by pulmonary edema resulting from inadequate clearance of alveolar fluid.

Transient tachypnea of the newborn presents with worsening tachypnea within two hours of delivery, and it is caused by pulmonary edema resulting from inadequate clearance of alveolar fluid.

-- -- presents with cleft lip and palate, polydactyly, rocker-bottom feet, holoprosencephaly, PCKD, omphalocele or umbilical hernia, and is most likely due to maternal meiotic nondisjunction.

Trisomy 13 (Patau syndrome) presents with cleft lip and palate, polydactyly, rocker-bottom feet, holoprosencephaly, PCKD, omphalocele or umbilical hernia, and is most likely due to maternal meiotic nondisjunction.

which RTA has recurrent nephrolithiasis

Type 1 RTA, also assoc w other Autoimmune disorders

Mcc ETEC

Unwashed fruits and veg

watery diarrhea, hypokalemia =

VIPoma WDHA watery diarrhea, hypokalemia, achlorhydria.

can't use what med in si pts?

Varenicline is a nicotinic receptor partial agonist, smoking cessation agent that should be avoided in patients with unstable psychiatric disease or suicidal ideation, as this medication may worsen these symptoms.

Chronic alcoholism can lead to cerebellar dysfunction presenting with gait instability, truncal ataxia, dysdiadochokinesia, intention tremor, and hypotonia.

Vascular insufficiency ulcers typically present with painful and shallow ulcers that have a pale base and a lack of granulation tissue that occurs at very distal locations, such as the fingertips and toes. Treatment includes smoking cessation and graded exercise.

Biphasic stridor that improves with neck extension

Vascular ring

-- -- classically present in patients less than 12 months of age with biphasic stridor that improves with neck extension, and they are associated with cardiac abnormalities.

Vascular rings classically present in patients less than 12 months of age with biphasic stridor that improves with neck extension, and they are associated with cardiac abnormalities. so get echo

--- --- is a potential complication of myocardial infarction that can lead to -- ---

Ventricular aneurysm is a potential complication of myocardial infarction that can lead to systemic thromboembolism.

cluster headache ppx?

Verapamil

-- -- presents with diarrhea, bullous skin lesions, and hemochromatosis.

Vibrio vulnificus presents with diarrhea, bullous skin lesions, and hemochromatosis.

abd pain, constip, polyuria, polydipsea

Vit D toxicity, DM symptoms, with normal glucose

how to monitor res status in GBS?

Vital Cap Do spirometry to assess pt resp status

--- toxicity occurs in patients with eccentric dietary habits and supplement use and presents with symptoms of nausea, vomiting, headache and blurry vision.

Vitamin A toxicity occurs in patients with eccentric dietary habits and supplement use and presents with symptoms of nausea, vomiting, headache and blurry vision.

---- deficiency presents with dermatitis, neuropathy, stomatitis, and mood changes and can occur in patients with inadequate dietary intake or sudden weight loss.

Vitamin B6 (pyridoxine) deficiency presents with dermatitis, neuropathy, stomatitis, and mood changes and can occur in patients with inadequate dietary intake or sudden weight loss.

thrombocytopenia, recurrent infxns, eczema:

WAS, dec CD43

Hypertension can be secondary to well-understood pathologies (such as pheochromocytoma, Cushing disease, or renal artery stenosis) or can be idiopathic, which is referred to as benign essential hypertension. The best initial and most effective treatment for benign essential hypertension is weight loss. In order, the most effective lifestyle interventions for hypertension include: 1. Weight loss (5-20 mmHg/10kg) 2. DASH diet 3. Exercise 4. Sodium restriction 5. Alcohol intake reduction

WDESA

Whereas CML and CLL typically present with leukocytosis that can exceed 100,000/uL, AML and ALL more often present with ----.

Whereas CML and CLL typically present with leukocytosis that can exceed 100,000/uL, AML and ALL more often present with cytopenias.

xanthomas related to

Xanthomas and xanthelasma are deposits consisting of lipid-laden macrophages. They are associated with familial hypercholesterolemia as well as any disease which leads to cholestasis such as primary biliary cirrhosis. PBC

parathyroid pituitary, now has epigastric pain, suspect

ZES, men 1

meta analyses

a systematic method of evaluating statistical data based on results of several independent studies of the same problem. funnel plot

acute sob, back/chest pain, signs of hemolysis (jaun, elev BR, LDH, dec Haptoglobin) s/p transfusion:

abo incompat

absence sz vs complex partial sz

absence: no post octal state complex: post ictal.

centrilobular necrosis

acetaminophen tox

acidotic effect on Ca

acidotic states cause H to displace Ca from albumin, causing free Ca but not total Ca to increase. This is in contrast to alkalosis, which causes H always divorces Ca and albumin, increasing free ca Bicarb in alkalosis loves Ca and eats it, decreasing free ca

atrial myxoma can cause

acute art occlusion

acute bacterial rhinosinusitis (ABRS). --- is the most common predisposing risk-factor for ABRS.

acute bacterial rhinosinusitis (ABRS). Viral upper respiratory infection is the most common predisposing risk-factor for ABRS.

globular uterus uniformly enlarged

adenomyosis, dysmenorrhea, menmorhagia, chronic pelvic pain.

mid 40s, multiparous woman, dysmenorrhea, menorrhagia, dyspareunia, urinary sx, enlarged uterus size (12 weeks or so). cause?

adenomyosis, endomet grows into myometrium. menorrhagia is seen in adeno. remember, pain w bowel mvmts is more assoc w endometriosis (rectouterine septum nod) Adenomyosis is characterized by the presence of endometrial glands and stroma in the uterine myometrium. Adenomyosis presents with dysmenorrhea, menorrhagia, and a tender, boggy, and enlarged uterus. The pathophysiology of adenomyosis is related to implantation of endometrial glands within the myometrium. In contrast, a leiomyoma presents with an enlarged, firm, and irregularly shaped uterus - this is an important differential diagnosis when working up adenomyosis

hellp syn rx

administer betamethasone and schedule imm delivery

baby has axillary hair, cause?

adrenal glands prematurely activated

low ACTH, high cortisol, negative high dose dex supp test obtain ---

adrenal imaging think abt it, if its inhib by dex, then pit source. if its ectopic then acth is elev, bc the ectopic source releases ACTH not cortisol directly. when ACTH low, its adrenals

scotoma central vision loss

age related Mac degeneration

se carbamazepine

agranolocytosis

Edematous, erythematous, bogginess, watery rhinorrhea

all rhinitis, rx intranasal fluticasone

urinary retention caused by -- agonists

alpha agonists, phenylephrine, tighten sphincter, can't pee

glom bm thin split

alport syn

best initial step for acute otitis media

amox

Rx cryptococcal meningitis

ampho B, flucytosine acutely fluconazole for life or until cd4 > 100

Rx cryptococcal meningitis

ampho b, flucytosine fluconazole later

apple green birefringence, pos congo red stain

amyloidosis

bv garnerella is

anaerobe

pruritus, urticaria, sob, wheeze, hypotension =

anaphylaxis

anti dna topoisomerase ab

anti SCL 70 Answer 3: Anti-DNA topoisomerase I (or Anti-Scl-70) autoantibodies are associated with diffuse scleroderma. Disease presentation is related to sclerosis of certain organ systems, which can range from hardening of the skin and fingertip pitting, to renal, digestive, pulmonary, and musculoskeletal symptoms.

cryo used to reverse

anticoag and rx DIC Pro clotting, use with FFP

bullous pemphigoid ab

antihemidesmosome ab This patient is presenting with tense blisters and a linear pattern on immunofluorescence, suggesting the diagnosis of bullous pemphigoid, which is associated with anti-hemidesmosome antibodies. Bullous pemphigoid (BP) is an autoimmune blistering disorder that causes tense bullae that is predominantly in the elderly. Unlike pemphigus vulgaris, there is a negative Nikosky sign (blistering following horizontal pressure on the skin). It is associated with anti-hemidesmosomes antibodies, which cause subepidermal cleavage. On immunofluorescence there is a linear IgG pattern along the basement membrane, as shown in Figure A. Treatment involves long-term topical or systemic glucocorticoids.

Wegener's granulomatosis with polyangiitis dx what AB

antiproteinase 3

ab in hashimotos

antithyroid peroxidase

This patient likely has chronic mesenteric ischemia causing food aversion, leading to his significant weight loss. Chronic mesenteric ischemia is also known as "abdominal angina" - it is caused by mesenteric atherosclerotic disease, leading to insufficient splanchnic blood flow immediately following meals. It is typically found in patients with a history of smoking, peripheral vascular disease, or coronary artery disease. Patients often present with chronic weight loss, food aversion, and dull, crampy abdominal pain within the first hour after a meal. Diagnosis can be made with duplex ultrasonography, or CT- or MR-angiography.

ao old ppl, aversion to eat consider chronic mes ischemia The critical difference is that steroid-induced myopathy, unlike PMR or inflammatory myositis, is associated with normal ESR and CK respectively.

unequal bp L vs R arm seen in

aortic diss

aortic regurgitation. -- --- -- -- is the most common cause of aortic regurgitation for young patients in developed countries.

aortic regurgitation. Congenital bicuspid aortic valve is the most common cause of aortic regurgitation for young patients in developed countries. Myxomatous degeneration is the most common cause of mitral valve prolapse and mitral regurgitation in developed countries, not aortic regurgitation.

sz can cause --- pna

aspiration!

severe RSV requiring hospitalization means inc risk of dev

asthma, by over 50%.

rx pertussus

azithro

gastric bypass/sleeve causes

b12 def

hypotension, bradycardia, hypoglycemia, rx:

bb od, so rx w glucagon

pos pressure mech vent is bad in tamponade

bc it dec CO, inc IT pressure dec Cardiac filling

rx chagas ds

benznidazole or nifurtimox

IgA, HSP,s/p viral URI

berger iga

Hg A2 inc in inc hg S in

beta thal minor SC in hbS

______ should be administered to patients < 34 weeks gestation, or in any case < 36 weeks gestation with a L/S ratio < 2.0. The L/S ratio suggests fetal lung maturity and should be greater than 2.0. For patients with a L/S ratio < 2.0, betamethasone should be administered to accelerate fetal lung maturity and increase surfactant production.

betamethasone

how to slow hear loss in paget bone

bisphosphonate

no vit K, fat Malabs issue or homeless malnutrition means that

bleeding, bc vit K klot

acrocyanosis:

blueness of the extremities, seen in cryoglobulinemia

mid shaft clap fx, which NV structures and blood supply at risk

brachial plexuss subclav art

nulliparity inc risk of what ca

breast and ovarian

ethylene glycol intox causes what on kidneys

ca oxalate crystals

rx prolactinoma

cabergoline or bromocriptine

Envelope shaped crystals in urine... which ds pts are at inc risk?

calcium oxalate stones Crohns patients Crohn disease causes loss of fat in the stool, which can bind calcium leading to increased oxalate absorption and increased risk for calcium oxalate nephrolithiasis.

Haloperidol is the most common antipsychotic to elicit EPS, especially when used for schizophrenia. The EPS include, acute dystonic reactions (four hours), pseudoparkinsonism (four days), akathisia (four weeks), and tardive dyskinesia (four months).

can use PCN to rx Pneumococcal PNA in elderly

Why give carbidopa with levodopa?

carbidopa is DOPA DC inheb dec excess peripheral dopamine to lessen se of levo.

concentric thickening ventricular walls

cardiac amyloid s patient's clinical presentation is consistent with cardiac amyloidosis (CA), which is diagnosed by cardiac biopsy. CA should be suspected in patients presenting with congestive heart failure, concentrically thickened ventricles, and non-elevated blood pressures. Decreased voltage on EKG in the setting of thickened ventricles on echocardiogram further point towards a diagnosis of CA. Patients frequently present with evidence of non-cardiac amyloid deposition, including tingling in the hands and feet (neural deposits) and foamy urine (renal deposits). Treatment is aimed at symptom management (diuretics) and decreasing further amyloid deposition with drugs such as cyclophosphamide and dexamethasone.

this worsens prog for sarcoid

cardiac involvement

halos seen in

cataract and glaucoma

painless, bl vision, halos around lights, trouble night driving, NO RED REFLEX

cataracts: hard lens, lens opacification

asylum motor weakness, saddle anes, hyporeflexia, late onset bowel bladder dysfn

cauda equina, start steroids and then mri

OCD initial rx?

cbt & ssri

rx whipple ds?

ceftriaxone and tmpsmx

Villous atrophy with crypt lengthening and intraepithelial lymphocytes

celiac

sudden onset painless loss vision, cherry red spot:

central ret art occlusion, massage globe, pt breathe in paper bag retina is pale, fovea dark red This patient with a history of atrial fibrillation, is presenting with acute-onset, painless, monocular vision loss and a fundoscopic exam positive for retinal whitening. This suggests a diagnosis of central retinal artery occlusion. Acute management includes ocular massage and hyperbaric oxygen. Symptoms of central retinal artery occlusion (CRAO) include sudden, painless, complete, and normally unilateral vision loss. Fundoscopic exam can show pallor of the optic disc, diffuse ischemic whitening of the retina (Figure A), cherry-red macula (Illustration A), retinal edema, and constricted/bloodless retinal arterioles. The center of the macula appears red because the photoreceptor layer at the fovea is perfused by the posterior choroidal vasculature, not the central retinal artery. Treatment involves surgical decompression of the retinal anterior chamber, ideally within one hour of the occlusion. If surgical intervention is not readily available, the next best step in management involves ocular massage (to dislodge the embolus) and either carbogen therapy or hyperbaric, high-flow oxygen.

chronic SDH mcc by?

cerebral atrophyThe patient is presenting with recurrent spontaneous abortions, a history of thrombosis, and a prolonged aPTT, which suggests the diagnosis of antiphospholipid syndrome (APLS). APLS is associated with a false-positive VDRL.

OCPs increase the risk of what cancer

cervical liver breast ocps inc TBG and total thyroid levels while free t4 t3 normal

lateral medullary syn, PICA

change in swallow, loss pain temp pistil face/cl body, horner., ataxia.

rx CF PNA in childhood vs adult

child: Vanc - S aureus adult: Amikacin - pseudomonas amikacin, ceftazidime, ciprofloxacin, or piperacillin-tazobactam.

intranuc inclusions, nuc grooves, pap fronds, psammoma body. this thyroid ca has hx of

childhood radiation, its pap thyroid CA orphan annie is white, grooves are the purple line through.

chronic ear infxns, drainage, inflamed bulging TM, augmentin doesn't do anything: --- dx?

cholesteatoma A cholesteatoma is a cystic structure typically located within the middle ear or mastoid process. It is the result of keratinizing squamous epithelial cells and may cause symptoms of otorrhea, hearing loss, and dizziness. It also has the potential to continue to grow and invade surrounding structures. A cholesteatoma may be congenital or acquired. Acquired cholesteatomas are most common among those who have chronic otitis media; however, it is also seen in those with tympanic perforation or unintentional surgical placement of skin into the middle ear.

prolonged OH, abd tenderness, steatorrhea=

chronic pancreatitis, CT abd w con

anca to myeloperoxidase is what dx

churg strauss

proteus UTI looks like

coffin shaped, alkaline UTI

case control

compare people with and without disease to find common exposures. think if they had past exposure.

otosclerosis is --- hear loss Presbycusis is -- hear loss (pres) is SN hear loss Conduction deafness = Rinne: BC > AC (in damaged ear); Weber: lateralization to damaged ear. Sensorineural deafness: Rinne: AC > BC (in damaged ear); Weber lateralizes to healthy ear.

conductive. so rhinne is b>a on affected ear, then the weber lat to the same one thats messed up. in SN hear loss, the weber lat to normal sounding one.

b/l corneal ulceration dx and rx?

contact lens inf keratitis - gatifloxacin eye drops

thiazides can cause --- --- --- rx

contraction met alk, IVF

sudden onset severe back pain perianal anesthesia, symmetrical motor weakness, hyperreflexia, early onset bowel bladder dysfn

conus medullaris

vertigo, hear loss tinn

could be labrynthitis vs menieres ds

lung findings: crepitus hyperres dec br sound, ddec tact frem, dull to percussion

crep: PTX hyperres: emphysema PTX pl eff: dec br sound, ddec tact frem, dull to percussion Tactile fremitus refers to the vibration intensity felt on the chest wall when asking the patient to speak.

hacking cough:

croup (parainflu), dexamesth, racemic epic, observation

WAGR assoc w

cryptorchidism

HIV diarrhea, acid fast organism dx rx?

cryptospor parvum, rx nitazoxanide

low wells score means get a

d dimer

pain swelling pus lacrimal duct, dx?

dacrocystitis inflamm of lacrimal sac

rx derm herpetiformis

dapsone

rx pemphig vulgarios (vulgarity comes from LIPS):

dapsone

preg moms ulnar dev w flexed thumb dx

de quervain tenosynovitis rest nsadis

labs in rickets

dec Ca, Phos, colecalciferol, inc alk phos, PTH.

crest syn, manometry shows??

dec LES tone and peristalsis in distal esoph The components of CREST syndrome include calcinosis, Raynaud's, esophageal dysmotility (secondary to LES sclerosis), sclerodactyly, and telangiectasias; it is a form a systemic scleroderma (sclerosis). The disease involves excessive deposition of collagen and is most common in females between the ages of 30-50. Raynaud's is typically the fist symptom to present, followed by finger puffiness that later progresses to thickening of the skin of the fingers.

how to reduce risk of ca ox stone formation

dec Na intake

how to decrease resp quotient?

dec carb, inc fat intake carb = 1.0, fat = 0.7

ocp dec which CA

dec colorecrtal, endometrial, ovarian CEO inc risk of breast and cervical.

hyperthyroidism effect on SVR -

decrease SVR

DLCO in emphysema

decreased DLCO is normal in Chronic bronchitis and asthma Normal ABC (asthma, chronic bronchitis) COPD has dec DLCO

SVR in hyperthyroid?

decreased.

Vit K deficiency

def factors 2 7 9 10, inc pt and ptt.

fever, HA, retroorbital pain, myalgia, arthralgia, leuko and thrombocytopenia:

dengue fever, dx w serology test Dengue fever is a mosquito-borne viral infection endemic to tropical Asia, Central and South America, and the Caribbean. It is characterized by fever, headache with retro-orbital pain, muscle and joint pain, rash, leukopenia and thrombocytopenia. It may progress to hemorrhagic fever or shock. The diagnostic test of choice is PCR in the acute phase of the illness and serologic testing in the later stages.

Kartagener syndrome (primary ciliary dyskinesia)

dextrocardia, recurrent sinusitis, URIs, bronchiectasis, situs inversus. BUS bronchiectasis, URI, situs inversus sinusitis

azathioprine se

diarrhea hepatotoxic

subendothelial deposits

diffuse proliferative GN, diffusely proliferatively cum endothelial

blurry vision, diarrhea, PVCs: dx?

dig tox, renal cleared. Chronic digoxin toxicity is characterized by blurry vision, gastrointestinal symptoms, and life-threatening cardiac arrhythmias. Because digoxin is primarily excreted by the kidneys, digoxin toxicity may be precipitated by acute kidney injury or renal failure. Patients with heart failure, particularly those on loop diuretics like furosemide, are at particular risk because the hypokalemia caused by the furosemide predisposes to digoxin toxicity.

rx acute dystonia

diphenhydramine or benztropine. Acute dystonia presents with muscular contractions in the setting of antipsychotic use and should be treated initially with diphenhydramine or benztropine.

turn pts head, dizzy nausea, nystagmus, dx rx

dix hallpike, epley note bppv has no hear loss tinnitus or ataxia, changes w position tho

transient loss of vision, transient monocular vision loss ischemia to the retina or optic nerve, retinal emboli from --- artery

do carotid US

intractable epistaxis

do electrocautery of Kiesselbach plexus

pcn allergy pt with dog bite rx w

doxy clinda, could also use TMPSMX clinda

syndrome after bypass

dumping, eat smaller portions, dec carbs

vertigo, hearing loss, and horizontal nystagmus dx?

dx is menieres ds, rx low salt diet. Symptoms will include chronic relapsing and remitting episodes of vertigo, sensorineural hearing loss, tinnitus, and nausea. First-line treatment is dietary modifications, specifically a low-salt diet, avoidance of alcohol, nicotine, and caffeine, and a thiazide diuretic. If symptoms persist, ultimate therapy may involve vestibular nerve ablation or a labryinthectomy.

scleroderma pt with AKI and HTN: dx and rx?

dx scleroderma renal crisis, rx ACE-I

diff in amebic abscess vs echinococcus

echinococcus has fever, where amebic abscess does not

Intercellular edema of the epidermis

eczema, atopic derm

low hcg, afp, estriol:

edwards During weeks 16-20, do amniocentesis. CVS is done 10-12 weeks. CVS 10-12, before Amnio 16-20

can't start Li when?

elev Cr

rx seb derm

emollients

schiller-duval bodies, inc AFP

endodermal sinus tumor

chronic pelvic pain worse b4 mestrual period, vag tenderness, lat displacement cervix =

endometiosis. after failing w OCP, do laparoscopy. ocp and nsaids first. Presence of endometrial glands and stroma outside the uterus describes endometriosis. Patients with endometriosis classically present with recto-vaginal tenderness, uterosacral ligament abnormalities such as nodularity or thickening, lateral displacement of the cervix, and possible adnexal enlargement due to an endometrioma. This patient most likely has endometriosis, causing dysmenorrhea, dyschezia, and dyspareunia as well as infertility. The most effective method of treating infertility in a patient with endometriosis is laparoscopic resection of endometrial implants. PCOS can induce ovulation w clomiphene to conceive, not in PCOS.

endometrial hyperplasia with atypia, which can be treated with --- therapy in patients who are hoping to become pregnant.

endometrial hyperplasia with atypia, which can be treated with progestin-only therapy in patients who are hoping to become pregnant.

dec bowel sounds, uterine tenderness post op, dx, rx?

endometritis, rx gent clinda

boy drank house cleaner (base), what now?

endoscopy

before dx gastroparesis, do what?

endoscopy to r/o mech cause obstruction

5 yr old, mass in 4th ventricle, dx?

ependymoma Medulloblastoma, a cerebellar tumor, may compress the fourth ventricle. Ependymomas are also commonly found in the fourth ventricle and may cause an obstructive hydrocephalus. They have a very poor prognosis. ME IN 4TH VENTRICLE

inc blood flow

epididymitis preens sign, dec pain when elevate testicle

abd pain, epigastric, pain radiate to shoulder, hypotensive, tachy what study to get pt will have rebound tender, guarding, hypoactive bs, sudden severe abd pain, perit signs.

epigastric pain acute: PUD perf! get CXR 1st- pneumoperitoneum

drooling, thumbprint sign, dx?

epiglottitis tripod position, fever, stridor, tachypnea,

what tremor is worse when point to something, rx?

ess tremor, rx propranolol/primidone

rx premat ovarian insuff

estradiol patch with PO medroxyprogesterone. Labs will show high FSH, low estradiol at start of foll phase. Patients require estrogen supplementation until around age 50, as low estrogen increases the risk of coronary artery disease, osteoporosis, and sexual dysfunction. Vaginal gel or a transdermal patch is the first-line option, but both must be used in conjunction with medroxyprogesterone to avoid unopposed estrogen and the risk of endometrial hyperplasia.

ferning also seen in ovulation, there are high -- levels

estrogen

how to dx vitiligo

examine under wood lamp

Calculate GCS

eyes, motor, verbal

This infant is most likely experiencing breastfeeding failure jaundice, as evidenced by poor milk intake and the onset of jaundice within the first week of life. Breast milk jaundice does not typically become apparent until 6-14 days of life.

f before m, feed failure before milk jaundice

angiokeratomas assoc w what?

fabry ds

which malaria subtype has cerebral involvement?

falciparum

causes of vit k def

fat malabs (panc, crohns), malnut (homeless, elderly), AB.

uremic pericarditis has

fever

no doxy for lyme ds in preg pts or children < 8

first line doxy then amox if pcn allergy then erythromycin

PCWP indicates how much pressures in L heart, for ex in card shock it will be high. low in hypovolemic shock

flat JVD, delay cap refill > 2 = hypovolemia

retinal detachment

floaters and flashes of light. curtain vision emergency

type4 RTA has altos resistance, in diabetic, rx w

fludricortisone

1st line rx SIADH

fluid rest

what is seen in crohns ds

focal ulcerations w granuloma

highly processed foods cause -- def

folate

vit a def patients havre

follicular hyperkaratosis

Rx of ethylene glycol OD?

fomepizole

Klinefelter syndrome results in primary hypogonadism and presents with tall stature, gynecomastia, small testes, a small phallus, hypospadias, and cryptorchidism.

from meiotic nondisjn, guy will be tall, have boobs, small balls, and high FSH and LH

HIV

fsgs

for boerhaave use

gastrograffin

se cyclosporin

ging hypertrophy

linear deposition type 4 collagen in GBM

goodpasture type 2 HS, lung and kidney) type 4 collagen, linear GBM

call exner bodies found in

granulosa cell tumors

call exner body

granulosa theca cell tumor

labrynthitis has --- --- vest near has does not have

hear loss

before dx autism get

hearing exam

vitamin E deficiency has

hemolytic anemia, as well as symptoms of B12 def (neuro sx)

lichen plans assoc w

hep c

aplastic crisis in parvo b19 (slapped cheek rash) w dec hg s and normal hg a2 has hx of

her spherocytosis

ARDS Rx

high peep, low TV, high CO2 permissive hypercapnia redues o2 tox

his patient is presenting with a thyroid mass, hoarseness, and hypocalcemia suggesting a diagnosis of --- Medullary carcinoma of the thyroid can occur in MEN IIa and MEN III syndromes which can be associated with pheochromocytoma (episodic hypertension and headaches).

his patient is presenting with a thyroid mass, hoarseness, and hypocalcemia suggesting a diagnosis of medullary carcinoma of the thyroid. Medullary carcinoma of the thyroid can occur in MEN IIa and MEN III syndromes which can be associated with pheochromocytoma (episodic hypertension and headaches).

what infxn in mom is CI to breastfeeding baby?

hiv

+FAST + seatbelt sign, concern for

hollow viscus injury, watch for free air under diaphragm

intellectual disability + marfan =

homocystinuria. Vit B6 (pyridoxine) dec cardiac comp risks. supplement b6,9,12. The diagnosis of homocystinuria is based on serum blood testing demonstrating increased levels of methionine and homocysteine.

hard painful lid nodule

hordeolum

polycythemia vera rx

hydroxyurea

bradycardia and short QT =

hyperCa, rx IV NS

bone pain, abd pain, trouble focus:

hypercalcemia from 1 hyperpth. Primary hyperparathyroidism is the most common cause of hypercalcemia and occurs secondary to increased secretion of parathyroid hormone (PTH). Hypercalcemia presents with "bones, stones, moans, and groans" which corresponds to symptoms of bone pain, renal stones, abdominal pain, and psychic symptoms (like trouble focusing). Patients should initially be stabilized with IV fluids as hypercalcemia can lead to diuresis. Definitive treatment is a parathyroidectomy which can subsequently lead to "hungry bones syndrome", which is hypocalcemia secondary to decreased activity of PTH and increased deposition of calcium in the bones. Symptoms of hypocalcemia include tetany, paresthesias, and cardiac arrhythmias.

ascending weakness, palpitations, and abdominal pain, missed HD appts, dx?

hyperkalemia

tmp-sms el effect

hyperkalemia

oral leukoplakia is what kind of plasia

hyperplasia

pretibial myxedema seen in

hyperthyroid, graves

oh abuse, weakness, occ palpit, brisk DTR, prolonged QT cause

hypo Mg

--- can cause post-op ileus

hypoKalemia pt has been vomiting a lot

risk for dig tox

hypokalemia

post op ileus suspect...

hypokalemia! pt will have vomit hx

pt has prolonged QT, hx of thyroidectomy, dx?

hypoparathyroidism dec Ca

chronic alcoholic which EL def in referring is common

hypophos

fluphenazine side effect:

hypothermia - disrupts thermoregulation and body's shivering mechanism

no 1 cause mortality burn pts:

hypovolemia, rx w IVF

mcc intuss

idiopathic The best first step in management of intussusception is pneumatic (air) enema, which is both diagnostic and therapeutic.

when can you do brain surgery on a met

if its 1 location of brain met, then radiation

rx hirschsprungsq

ileostomy with appendecetomy

lateral fem epicondyle pain

iliotibial band syndrome

child honey crust lesion =

impetigo, can cause PSGN. A rare, but possible complication of impetigo caused by S. pyogenes is PSGN which can present with dark urine, periorbital edema, and hypertension.

how to transport avulsed teeth?

in milk..

thyroid hormone effects:

inc HR, contractility, dec SVR

chronic OH use results in what effects on nmda and gaba ch

inc NMDA channels, downreg GABA. cerebellar dysfn, gait instab, truncal ataxia, dysdiadokokinesia.dec transketolase activity.

hypovolemia effect on SVR?

inc SVR in hypovolemia

maternal hyperglycemia w dm inc risk of

inc risk of neural tube defects, heart defects Neonatal hyperglycemia is not seen in IDM, but hypoglycemia is. This is due to crossing of glucose from the mother to the fetus via the placenta, and subsequent hyperinsulinemia and ß-cell hyperplasia from the fetus. After delivery, the high insulin levels cause hypoglycemia because there is no longer an overabundant source of glucose. Hyperglycemia in infants is much less common and can be due to glucose infusion or neonatal sepsis.

scihzos brain imaging

inc vent size, dec cerebral, hippocampal, temp masses. NO CHANGE IN CEREBELLAR MASS

best study for incidence and prevalence?

incidence: prospective cohort prevalence: cross sectional

endomysial inflammatory with basophilic rimmed vacuoles within muscle fiber sarcoplasm

inclusion body myositis Inclusion body myositis typically affects distal forearm and arm muscles and may cause bulbar symptoms of dysphagia.

lupus anticoagulant causes ---- on ptt, and causes

increases PTT, lends to clotting, not bleeding.

acute gout rx

indomethacin (renal failure) inject steroid colchicine, (watch hep renal fail)

rifampin affects cyp450

induces it, and so don't use for pts on warfarin

Intrauterine fetal demise managed with

induction of labor bc it can cause DIC

OCP moa

inhibitors/delay ovulation

3-year-old boy is brought to his pediatrician by his mother when he developed redness, burning, itching, and exquisite pain all over his arms, lower legs, neck, and face.

initiate PO beta carotene, dx erythropoietic protoporphyria

pure sensory pure motor stroke location

int capsule

social anxiety disorder

intense fear of social situations, leading to avoidance of such rx is CBT

pathophys eczema atopic derm

intercell edema epidermis This patient is presenting with pruritic, scaly lesions on his hands suggestive of a diagnosis of atopic dermatitis (eczema). Atopic dermatitis is associated with the histologic finding of intercellular edema in the epidermis. Atopic dermatitis typically occurs in patients who have a history of atopic disease (such as asthma or seasonal allergies). They will present with intensely pruritic, erythematous, and scaly lesions typically of the flexural surfaces. Histologic findings of atopic dermatitis include intercellular edema of the epidermis. The treatment of choice is avoidance of irritants and topical steroids for acute flares.

haptoglobin dec in which hemolysis?

intravascular hemolysis: dec haptoglobin

Cl and bicarb relation

inverse. as cl is dec, there is less bicarb secretion and bicarb goes up. Metabolic alkalosis is commonly generated and maintained by volume, chloride, and potassium depletion; volume resuscitation is indicated to increase bicarbonate excretion.

after dx endometriosis, try OCP, if fail then

laparoscopy w bx for def dx. empirically manage w ocp nsaids

rx for glaucoma? acutely? how to dx?

laser periph iridotomy, more acutely timolol drops rx long term w acetazolamide, CA inhib The best initial test for acute angle-closure glaucoma is ocular tonometry, while the most accurate test is gonioscopy.

resp sx, gi sx, hyponatremia: dx?

legionella, commonly on cruise ships.

intermesntrual bleeding, pelvic pressure BUT PAINLESS, irreg shaped, enlarged uterus:

leiomyomata

rx aspirin induced asthma

leukotriene

Tremor bradycard ataxia neph DI, med OD

li tox

rx for CCB perish edema?

lisinopril

metformin CI in renal failure, sepsis, severe heart failure, liver dysfn, COPD not CI for metformin

liver heart kidney problems means no metformin

normal AST ALT chronic, elev PT and PTT, dec Albumin, dec Ca severe cases, definite therapy? dx and rx?

liver transplant- fulminant hep failure

ca2+ in crush injuries

low

no breasts means estrogen level is so in pt with primary amenorrhea, inc FSH, do ---

low so in pt with primary amenorrhea, inc FSH, do karyotype

wavy lines

mac degen. MD see wavy. FM opposite of retina, fovea in center around is macula which will have drusen. rx multivavit and antiox. rx Zn.

u/l nipple dc, do

mammogram

how to dx DES?

manometry

tall, joint hypermob, tearing chest pain:

marfan, dissection. Pts with marfan have MVP, mid sys click. MARFANS AR MVPs

Neonatal polycythemia causes

maternal HTN, DM, preeclampsia, smoking

first stage of labor

max cervical dilation 2nd deliver infant 3rd deliver placenta

mcv and relic count in her sphero

mcv normal, inc mchc, inc retic

how to crrect Ca?

measured Ca + 0.8 (4-albumin)

child w nephrotic syn and hep B, dx?

memBranous GN

spike and dome w supepithelial deposits

memb GN

hpv rec up to age -- for men -- for women

men: 21 women 26

vertigo, hear loss, and tinnitus

menieres ds, rx diet mod, low salt, no OH, nicotine, caffeine

funnel plot triangl is

meta-analysis, combine results of several studies together, increasing power of findings of indiv studies

wilms tumor does not cross midline, derived from?

metanephros WAGR: wilms, aniridia, growth abnorm, retard. Neural crest cell-derived abnormality describes a neuroblastoma, which presents with an abdominal mass that crosses the midline and is associated with elevated urinary metanephrines.

heroin user, vomiting, diarrhea, restless, mydriasis, lacrimation, rx

methadone, rx for heroin withdrawal

folate is depleted in OH pts and is cofactor for what enzyme?

methionine synthase

rx for trichomonad, green yellow discharge

metro for pt and partner

mild hip pain after a viral infection suggesting a diagnosis of -- --. This pathology only requires conservative therapy (NSAIDs and rest) and tends to self-resolve.

mild hip pain after a viral infection suggesting a diagnosis of transient synovitis of the hip. This pathology only requires conservative therapy (NSAIDs and rest) and tends to self-resolve.

renal injury, met all, hyperCa

milk alkali syndrome

rx seb derm

mineral oil daily scalp wash

angel mans, whose chromosomes are missing?

moms are missing in angel man, so all dad disomy of chromosome 15 Angelman syndrome is caused by paternal uniparental disomy in which both copies of chromosome 15 are inherited from the father, or from maternal deletion of 15q11.

Compression of the forefoot with concomitant pressure on the interdigital space reproduces the pain on the plantar surface between the third and fourth toes and produces an audible click. What is the cause of this patient's condition?

morton neuroma

do --- on elderly dementia to figure out wtf is going on

mri

osteolytic lesions of

multiple myeloma, repeat infxns lack of AB diversity

pelter huet think

myelodysplasia

hypocellular and fibrotic bone marrow

myelofibrotic disorders

rx TCA OD

na bicarb

Hepatocyte swelling and necrosis with macrovesicular fat, Mallory bodies, and neutrophilic infiltration

nafld

psoriasis assoc w what nail changes

nail pitting

decreased orexin

narcolepsy

rx for RSV bronchiolitis

nasal suctioning, o2, IVF

TRACHOMA = --- eye and ultimately blindness.

neovascularization

asx hypocalcemia, DVT dx

nephrotic syn, piss protein, hyperlipidemia ensues, and hypercoag state lose AT3 and protein c/s which cause bleed.however, ionized free Ca remains same, so no sx of hypoCA

acyclovir se

nephrotox

amp B se

nephrotox

new sz, mri head multiple fluid filled cysts dx?

neurocystercicosis rx albendazole from tania sodium, meat.

hartnup, has low levels of

niacin

anorexia,dpressed,whatnottoprescribe?

no bupropion since it lowers SZ threshold

how is hsv encephalitis different than classic meningitis"

no meningeal signs

Anti-GBM syndrome (goodpasture) is a glomerular disorder marked by the clinical triad of crescentic glomerulonephritis, pulmonary hemorrhage, and IgG anti-glomerular basement membrane antibodies. Common presentations include fever, muscle pain, renal failure, hemoptysis, cough, and dyspnea. Initial treatment in anti-GBM syndrome is emergent plasmapheresis in conjunction with steroids and cyclophosphamide. Plasmapharesis clears circulating auto-antibodies and decreases the risk of progression to dialysis. Steroids and cyclophosphamide are used to suppress antibody production, but the effect is less immediate than that with plasmapheresis. This is in contrast to another pulmonary renal syndrome - granulomatosis with polyangiitis (GPA - formerly Wegener's granulomatosis), in which steroids and cyclophosphamide (or rituximab) are used without routine administration of plasmapheresis. Most patients with GPA are c-ANCA/PR3 positive. While renal biopsy in anti-GBM reveals linear IgG deposits along the basement membrane, biopsy in GPA will show necrotizing vasculitis with and granulomatous inflammation.

no plasmapheresis in Wegeners GPA. Wegeners wants steroids and cyclophos.

nocardia vs acintomyces which is acid fast branching filaments

nocardia, also causes cavitation in lung

hematuria, worried abt pyelo get ---

non con CT, pyelo

increasing cutoff point, effect? dec " " "

non ds L, ds R inc cut off L --> R: dec Sensitivity, inc PPV think about it now if you are +, you def have this shit dec cut off point R --> L: inc sens, inc NPV remember sens and spec are opp the closer to L the higher the sens (inc NPV), more R means higher spec (inc PPV)

pyloric stenosis is --

nonbilious

MM often found after

nontraumatic fx in mid age pts, monoclonal immunoglobulin m spike suggests it

DLCO in asthma

normal

borborygmi = GI stasis can occur in scleroderma and diabetes, leads to

normal flora overgrowth Decreased GI motility can occur in pathologies such as diabetes mellitus (decreased neural input to the bowel) or scleroderma (fibrosis of the bowel wall). When the bowel is unable to undergo peristalsis, overgrowth of the normal flora can occur. This bacterial overgrowth can lead to symptoms such as abdominal pain, bloating, diarrhea, and flatulence. Treatment includes antibiotics (such as rifaximin or amoxicillin-clavulanate) and pro-kinetics (such as erythromycin or metoclopramide).

g6pd pt with PCP

normally its tmpsmx, but bc g6pd, do pentamidine

how to confirm RSV bronchiolitis dx?

nucleic acid nasal aspirate

post sz, lactic acidosis, rx?

observe, repeat labs in 2 h

Strabismus Rx

occlusion of normal eye, so the messed up one starts to work normal

This patient with Jervell and Lange-Nielson syndrome presents with congenital sensorineural deafness, syncope and prolonged QT interval on EKG. Prolonged QT can be seen HYPOcalcemia, NOT HYPERcalcemia.

old women have low Ca and are hella cute!

neonatal tremor, sleep disturbance, sz, hyperactive reflex, sweat, temp instab, YAWNING:

opiate withdrawal. rx w morphine for sx control

is a 1-day-old full-term child who has not been feeding or sleeping well and has diarrhea. When you examine the child, you note an irritable sweaty baby with a high pitched cry, sneezing, and mottled ski

opioid withdrawal, neonatal absintence syndrome, mom did opioids in preg. give morphine

12 yo exercise induced knee pain, tenderness tibial tubercle =

osgood schlatter ds

Cisplatin SE

oto and nephrotoxicity same as aminoglycosides

loops cause

ototox

anti mullerian hormone indicates

ovarian reserve, so its low in turners syn pateints.

elev Alk phos, hip pain, elderly pt, think ca, phos, PTH normal

paget ds bone: assoc osteosarcoma

weight loss, fatigue, malaise, + OH: dx

pancreatic CA, repeated pancreatitis and alcoholism, no jaundice if in tail of pancreas

dark urine, jaundice, wt loss

panic CA

what thyroid ca caused by radiation?

pap thyroid CA

symmetry arthritis, rash hands and feet, dx? recent illness

parvo b19

teen with anterior inferior tenderness to patella =

patellar tendonitis

rx a fib with no structural heart ds?

per echo is pt heart structurally intact, rx w class 1C drugs ie flecainide or propafenone.

v/q scan findings in PE

perfusion defect without ventilation defect

pericardial calcification, suggests? dx? MCC

pericardial calcified equals constrictive pericarditis. MCC is TB Constrictive pericarditis is a subtype of pericarditis characterized by diastolic heart dysfunction and possibly right heart failure. Common causes include radiation, recent cardiac surgery, viral illness or idiopathic, and TB. It may present with chest pain that is relieved by leaning forward, dry cough, dyspnea, and eventually ascites, peripheral edema, and hepatomegaly. Constrictive pericarditis due to extrapulmonary TB is often nonspecific in the beginning, with systemic symptoms developing before other symptoms appear. Classic findings in constrictive pericarditis include Kussmaul's sign (lack of an inspiratory decline in jugular venous pressure), friction rub, pericardial knock, and most commonly elevated jugular venous pressure. Echocardiography will show pericardial echogenicity representing calcification (Illustration A). If TB is suspected as the cause for constrictive pericarditis, evaluation should include looking for TB in other parts of the body (sputum culture and examination for acid-fast bacilli) and possible pericardiocentesis. TB-related cases do not tend to self-resolve, so anti-TB therapy and corticosteroids are needed, with pericardiectomy if refractory.

cardiac scratch sound=

pericardial friction rub: pericarditis

uvular deviation, drooling, hot potato voice:

peritonsillar abscess

sore throat, fever, drool, cervical LAD, muffled voice, uvular dev:

peritonsillar abscess, ct shows rx w needle aspiration then iv AB

Anti-intrinsic factor antibodies

pernicious anemia b12 def

anteromedial tibia pain=

pes anserine bursitis

meds that cause folate def

phenytoin, primidone, phenobarbital, trimethoprim, pyrimethimine, MTX

doxy se

photosensitivity

rx antichol tox

physostigmine

chest pain, dysphagia after taking meds=

pill esophagitis

prior c section + painless vag bleeding =

plan previa

blue cells are

plasma cells, overgrowth in MM

rx ttp

plasma exchange

aldolase level checked when suspecting

poly, dermatomyositis

poor glycemic control in the mother (fasting glucose greater than 95 mg/dL and postprandial glucose greater than 140 mg/dL) can cause hypoglycemia in the infant due to -- and hyperinsulinemia in the fetus. The effect of insulin persists just after birth, resulting in hypoglycemia that can present as hypothermia, tachypnea, lethargy, poor feeding, jitteriness, and ultimately seizures.

poor glycemic control in the mother (fasting glucose greater than 95 mg/dL and postprandial glucose greater than 140 mg/dL) can cause hypoglycemia in the infant due to glucose crossing the placenta and triggering ß-cell hyperplasia and hyperinsulinemia in the fetus. The effect of insulin persists just after birth, resulting in hypoglycemia that can present as hypothermia, tachypnea, lethargy, poor feeding, jitteriness, and ultimately seizures.

obesity predisposes to -- -- inc insulin causes inc androgen. inc estrogen and testos,

precocious puberty

ribavirin CI during --

preg

dvt pe more likely in --

pregnancy

what causes cerebral palsy, risk factor?

prematurity

Membranoprolif GN

presents with diffuse menangial cell prolif with cap wall thickening

dec prevalence affects sens and spec how?

prev doesn't affect em

monthly cramping abd, back pain

primary dysmenorrhea

pts with schizo are inc risk of dev

primary polydipsia

se macrolides

prolong QT

fem neck fx, elderly pt, displaced fx, rx w

prosthesis

shistocytes, recent sx, elev INR:

prosthetic heart valve placement.

warfarin skin necrosis worse in pts w

protein c def

noonan different from turners bc it has

pulmonic valve stenosis and septal defects.

how often to reposition ulcers

q2h

vesicles ext aud canal, ear pain, ipsil facial paralysis: what is the syndrome

ramsay-hunt syndrome

confounding minimized by

randomization

PCN use in mono can cause

rasha mono has POST CERV LAD mono rx w supporitve ther avoid contat sports (splenomeg)

hypopyon (pus bottom of eye), leukocyes in ant chamber, cause is what

recent cataract sx

MM has predisposition to

repeat infxns

fever, neck pain, CERVICAL LAD, fullness in pharynx, midline uvula, no stridor

retroph abscess 6 mo - 6 yrs watch out for acute mediastinitis!

sz, cocaine, exercise can all cause

rhabdo causing hyperKalemia

pediatric rx FBAspirtation

rigid bronch

red rash checks and nose, worse w coffee 50 yo women: dx and rx?

rosacea, rx topical metro

rx RTA 1

rta 1 rx w Nabicarb replacement RTA 2 rx HCTZ

3 day measles, begins with a fever followed by a 3 day descending rash, starting at the head and working its way down (just like measles). Infamous intrauterine infection, affecting heart (Patent Ductus Arteriosus), eyes (cataracts), and CNS (microcephaly, sensorineural deafness). Noted for postauricular and cervical lymphadenopathy in the mom.

rubella

rash face and trunk. before this had low grade fever, post cerv LAD. dx

rubella german measles

get what titers before pregnancy?

rubella, variecella Because of this, the CDC recommends that pregnant women not be administered a live-attenuated vaccine such as MMR, varicella, or live-attenuated influenza. If a woman receives a live-attenuated vaccine, she should be counseled to avoid becoming pregnant for one month after the vaccination. If a patient without demonstrated immunity by titer is already pregnant or wishes to become pregnant in the next month, then she should be counseled on the risks of congenital rubella syndrome and the importance of being vaccinated as soon as they are no longer pregnant. If a pregnant patient is accidentally vaccinated or if a woman becomes pregnant one month after receiving a live-attenuated vaccine, she should be counseled on the theoretical risk to her fetus, but no changes to her prenatal care are necessary.

cold + dec br sounds =

ruptured bleb

steroids guy comes to ED, hypotensive, suspect:

ruptured hep adenoma

indications for rx Hep C?

rx w sofosbuvir, indic are age>18, RNA level, compensation liver ds

facultative anaerobic g+ cocci clusters

s clusters is staph, chains is GAS

delayed onset prosthetic joint infxn, organism is

s epidermis

EKG finding in PE

s1q3t3, rbbb

sc pt severe bone pain,fever

salmonella osteomyelitis

tamox and raloxifine used in osteoporosis, se?

se are hot flash and dvt

test to confirm ZES?

secretin stim test

lead poisoning can cause

sid anemia w basophilic stippling, high ferr, low TIBC, high erythro protoporphyrin.

OH causes what anemia

sideroblastic, inc Fe, normal TIBC, ringed sideroblasts on Prussian blue stain, OH, ISNZ, lead poisoning.

anti ro la ab

sjogren DRY The patient is presenting with dyspareunia and dental caries suggesting a diagnosis of Sjogren's syndrome. Sjogren's syndrome is an autoimmune disorder that targets salivary and lacrimal glands. The most common symptoms that patients present with are dry eyes (keratoconjunctivitis sicca), dry mouth (xerostomia), arthritis, and dyspareunia. Biomarkers that are indicative of the disease include anti-Ro (anti-SSA) and anti-La (anti-SSB). Anti-nuclear antibodies (ANA) and rheumatoid factor (RF) may also be positive.

dry eyes, dry mouth, dental caries, and dyspareunia.

sjogren, anti-Ro ab

risk factors for oral leukoplakia

smoke, OH

causes of AIUGR

smoking, DM, HTN

Hyponatremia classically occurs in elderly patients on 'tea and toast' diets, alcoholics with 'beer potomania', and anorexic patients with very low calorie intake. Euvolemic hypotonic hyponatremia occurs because the clearance of free water is linked to excretion of solute in the urine. Urine osmolality is less than 100 mOsm/kg in these patients.

so its from poor solute intake

This patient's radiograph shows bilateral bowing of the tibia and femur, suggesting inadequate bone mineralization due to metabolic disease. His lab values are consistent with Vitamin D-Resistant Rickets (aka Familial Hypophospatemic Rickets). Familial Hypophosphatemic Rickets is the most common form of rickets. It is often caused by an x-linked dominant mutation in the PHEX gene, leading to an inability of the renal tubules to absorb phosphate (GFR remains normal). This leads to inadequate mineralization of bone and commonly presents as bilateral tibial bowing between the ages of 6 months and 2 years. Key lab values for the diagnosis are low serum phosphorous and elevated alkaline phosphatase. Serum calcium is usually normal or low normal. High dose vitamin D3 and corrective surgery is currently the standard of care for hypophosphatemic rickets. Although phosphate replacement is the intuitive solution, current literature regarding the benefit of phosphate replacement for these patients is controversial.

so just choose vitae's d res rickets normal Ca, low Pos, high all pos, normal PTH, normal 1,25 dihydroxy

the PT is not abnormal until 80% liver function is lost.

so rapidly imrpving AST/ALT with worsening PT indicates fulminant hepatitis

low back pain improves lean fwd, dx?

spinal stenosis, get MRI lump spine

smoke colored urine

split glom bm, alport Syndrome! hematuria with SN deafness

contact dermatitis type 4 HS rxn, histology shows?

spongiosis

how to confirm preeclampsia dx

spot urine protein cr ratio

long term rx panic attack

ssri

varicoceles worse w

standing. pain improves upon lying down. they may cause test atrophy.

acute MS flare rx

steroids

rx ITP

steroids

rx poly and dermatomyositis

steroids

rx for anterior uveitis?

steroids topical red eye, super bloody looking, consensual photophobia

acute rx ITP in children

steroids, IVIg

positional straining urine leak:

stress incont w valsalva, urethral hyper mobility

multiple seb keratoses

supraclav ln enlargement, visceral malig.

Juvenile nasopharyngeal angiofibroma is a benign and vascular tumor that must be further evaluated with head imaging, such as CT scan or magnetic resonance imaging (MRI). rx is

surg resection

meds, double bubble, get

surgery eval

dysphagia, was in South america

suspect chagas, achalasia, do ba esoph.

symmetric intrauterine growth retardation is caused by --.

symmetric intrauterine growth retardation is rubella infection.

knee pain w fever, 1st step

synovial fluid analysis via arthrocentesis

mva, dec upper extrem str and sensation: dx?

syringomyelia

ha, wt loss, arthralgia, ASYMM pulses: --- rx is

takayasu arteritis steroids

crowns affects --- ---, causing --- def

term ileum, b12

testicular ischemia in there is no

test torsion crem reflex The best treatment of testicular torsion is surgical detorsion and bilateral fixation.

elevated hg a2?

thalassemia

chf, periph neuropathy in OH abuse patient rx give them

thiamine, b1, beri beri

"colors more bland looking" vision loss, eye pain

think optic neuritis, internuc ophthalmoplegia also seen

older man arthralgia, wt loss, fever, diarrhea foul smelling, abd pain dx?

think whiles ds, PAS+ lamina propria of small intestine on bx and rx w AB

in resp alk, as pH rises affin of albumin to Ca risen

thus there is temporary hypocalcemia and pt has numbness parasthesias

indirect hyperbili, lethargy, hypotonia, large protruding tongue, dx in 6 wk old?

thyroid dysgenesis

rx open angle glaucoma

timolol eye drops

rx seb dermatitis

topical antifungals, -azoles

rx candida vaginitis in preg

topical miconazole

rx for localized impetigo

topical mupirocin

rx nonbullous impetigo

topical mupirocin

tine corporis rx

topical terbinafine and azoles

when to dc tb treatment?

transaminitis

AR wide pulse pressure, water hammer pulse. causes?

trauma, dissection, inf endocarditis.

rx bed bug bites?

triamcinolone topical steroids

rx migrant, moa?

triptan, 5ht1 b/d agonist

tender mass F runner lateral greater trochanter

trochanteric bursitis

insensate elbow lesion could be

tuberculoid leprosy

ob pt, diabetes, 32 weeks gestation, what to start?

twice weekly non stress tests until delivery starting at 32 weeks! Pregnant women with diabetes are at an increased risk of sudden intrauterine death. Intrapartum fetal assessment, consisting of a nonstress test, biophysical profile, or combination of these twice per week until delivery, should begin at 28-32 weeks gestation. A nonstress test (NST) is an evaluation where external ultrasound monitors are placed on the mother's abdomen to monitor the fetal heart rate. A reactive NST is defined as 2 or more accelerations in 20 minutes, where an acceleration is at least a 15 bpm increase in fetal heart rate for at least 15 seconds. A biophysical profile is a scoring assessment of fetal well-being; it includes 5 components: 1) nonstress test, 2) ultrasound evidence of fetal breathing movements, 3) ultrasound evidence of fetal gross body movements, 4) US evidence of fetal muscle tone, and 5) qualitative assessment of amniotic fluid volume.

h pylori produces

urease in small intestine

radiolucent stones are

uric acid stones, myeloprolif ds can release the uric acid to cause them. can also happen from HGPRT def leech nyhan.

Adenomyosis is caused by abnormal endometrial tissue within the uterine myometrium. Patients present with dysmenorrhea, menorrhagia, and chronic pelvic pain. The uterus would be UNIFORMLY enlarged rather than irregularly (irreg = fibroids)

uterus is fixed and retroverted: endometriosis

hemangioblastomas: there is high risk of which CA

vHL. high risk of RCC. PTH-rp secreted by: Squamous cell CA lung, RCC, breast CA.

how to stop smoking?

varenicline and nic patches

PROM, painful vag bleed, fetal bradycardia, materal HD instab: rx?

vasa previa, requires imm C section

vasc insufficiency can present with

vascular claudication pain in legs w walking and art insuff ulcers.

only vertigo, no pos change

vest neuritits, rx meclizine Vest neuritis Vertigo only

fever, malaise, chest pain

viral dilated CM This patient's exertional dyspnea, orthopnea, bibasilar crackles, S3 heart sound, and the laterally displaced cardiac apex is suggestive of a dilated cardiomyopathy (DCM). The patient's recent viral infection suggest that an enterovirus, such as coxsackie B virus (an enterovirus), caused the myocarditis that eventually resulted in DCM. The most common cause of myocarditis is viral infection, which can eventually result in DCM. Parvovirus B19, human herpesvirus 6, coxsackie B virus, and influenza virus are examples of cardiotropic viruses that can result in a myocarditis. The mechanism by which myocarditis occurs is via a direct cytotoxicity and adverse autoimmune response in virally infected cells. Viruses, such as coxsackie B virus, can result in a myocarditis that leads to dilated cardiomyopathy.

hyper viscosity, syn?

waldenstrom

best lifestyle intervetion for HTN

weight loss

newborns eye issues, if its within 5 days of birth: if its weeks later it is:

within 5 days gonococcal. Got be immediate and puruelent. rx IM ceftriaxone Chlamydia is weeks later.

schiller duval body

yolk sac tumor

which neck zone gets surgery

zone 2, remember top to bottom is 3 2 1


Kaugnay na mga set ng pag-aaral

English II (H) Hero's Journey (POPLAW)

View Set

ACCT 4301 Chapter 13 Concepts Pt. 1

View Set

Gero test chapter 2 caring and aging

View Set

MKTG Chapter 9: Segmentation, Targeting, and Positioning

View Set